UWORLD 2

¡Supera tus tareas y exámenes ahora con Quizwiz!

CMV rarely causes disease in the immunocompetent. Typically responsible for subclinical infection. When primary CMV infection does result in clinical evident illness, affected individuals appear to have mono-lke syndrome characterized by fever, malaise, myalgia, atypical lymphocytosis and elevated liver transaminases. In contrast to EVB, where heterophil antibodies are not usually present in patients with CMV, thus they will have a negative Monospot test. a/d: odynophagia (painful swallowing) associated with linear esophageal ulcers d/t CMV infection is seen in patients with HIV. Retinitis is one of the most common manifestations of CMV infection in patients with HIV. b. disseminated CMV can cause diarrhea and hepatitis in the immunocompromised. In this patient, the combination of positive Hep B surface antibody with a negative hep B surface antigen and a negative hep B core antibody reflects immunization against Hep B, not prior hep B infection. e. A vesicular skin rash is seen in associated with herpes simplex or herpes zoster virus infection f. cmv pneumonitis typically occurs after organ transplantation.

1592 Dude has positive lab results for Hep B surface antibody but not surface antigen or core antigen, and CMV. Which of the following did he most likely had in the past? a. esophageal ulcers b. hepatitis c. mono d. retinitis e. vesicular skin rash f. viral pneumonia

S. aureus can cause right sided endocarditis in IV drug users. These bacteria settles on the valve leaflets d/t blood flow turbulence at these sites. They can cause perforation in the heart valves, rupture the chord tendineae, and send septic emboli to the lung (with right heart endocarditis) or to the brain and systemic circulation (with left heart endocarditis). This patient with fever is an IVDU with holosystolic murmur that increases inspiration (likely tricuspid regurgitation) and multiple lung opacities (likely septic emboli). a. candida endocarditis is rare and typically a severe manifestation of candidemia, with septic emboli to the brain, extremities and GIT tracts. Patients who are IVDUs, have prosthetic heart valves, or have indwelling lines are at increased risk of infection. b. cultured negative endocarditis d/t Bartonella, Coxiella, Mycoplasma, Histoplasma Chlamydia or HACEK (Haemophilius, actinobacillus, cardibacteriu, eikenella, and kingella), among others, would be much less likely than S. aureus endocarditis c/d/e/h: S. pneumonia and K. pneumonia typically cause pneumonia; Klebsiella can also lead to UTIs. Nocardiosis typically affects the lung (cavitary lesions), brain (brain abscess), or skin and is seen mostly in immunocompromised patients. Peptostreptococcus causes anaerobic infections (dental, cutaneous, intraabdominal). g. S. bovis now known as S. galolyticus is part of the normal flora of the colon and bacteremia or endocarditis caused by these guys is associated with colon cancer (happens in about 25% of the cases) I. S. viridian is the most common etiologic agent in subacute bacterial endocarditis following dental work. SBE occurs most frequently in patients with preexisting valvular abnormality (eg, rheumatic heart disease, congenital heart malformations). The presentation is typically sub acute (over weeks rather than days).

729 A 23 y.o ha fever, chills, dyspnea, and cough for the past several days. Temp is 102, HR 109. PE shows 3/6 holosystolic murmur heard best at the lower sternal border that increases on inspiration. Chest imaging reveals scattered bilateral peripheral lung opacities. HIV testing is negative. She also has needle track marks on both arms. Blood cultures will most like show which of the following organisms? a. candida b. eikenlla cordons c. K. pneumonia d. Nocardia e. peptostreptococcus f. s. aureus g. S. bovis aka S. gallolyticus h. S. pneumonia i. viridan group of strep

ACALCULOUS CHOLECYSTITIS: is acute inflammation of the gallbladder in the absence of gallstones. -Most commonly occur in critically ill patients and is associated with high mortality. Pathology, gallbladder stasis and ischemia, which cause inflammation of and injury to the gallbladder wall. Presentation: -fever, RUQ pain, positive Murphy's sign, leukocytosis and mild elevation in LFT. -PE: jaundice and palpable RUQ mass. Diagnostic study of choice is an ultrasound, which may show signs of acute cholecystitis (eg, an edematous and enlarged gallbladder) and no gallstones a. choledochal cysts are congenital dilations of the common bile duct that typically present during childhood (age <10) with recurrent abdominal pain and jaundice b. chronic cholecystitis results from repeated mild attacks of acute cholecystitis, which leads to thickening of the gallbladder wall. Ultrasound typically demonstrates a shrunken, fibroses gallbladder c. Liver fluke infection usually affects individuals from endemic areas (eg, southeast asia) and presents acutely with fever, RUQ pain, jaundice and eosinophilia. Infection of the biliary tree is associated with the formation of brown pigment gallstones, not acute calculus cholecystitis. e. pigment gallstones are categorized as black (typically secondary to intravascular hemolysis) or brown (typically secondary to biliary infection). f. porcelain gallbladder is usually found incidentally on abdominal radiograph as a rim of calcium deposits outlining the gallbladder. The condition associated with chronic cholecystitis and may increase the risk of gall bladder carincoma.

81 A 75. yo man has abdominal pain and fever d/t necrotizing fasciitis of the gluteal region and severe sepsis. He was treated appropriately. For the past 2 days, the patient has had right upper quadrant abdominal pain, nausea and recurrent fever. PMH HTN, type 2 DM and COPD. Temp is 102. PE shows marked tenderness over the right subcostal area. Leukocyte count is 18k compared to 9k 2 days ago. a. choledochal cyst b. fibrotic and shrunken gallbladder c. fluke infection of the biliary tree d. inflamed and enlarged gallbladder e. pigment gallstones f. porcelain gallbladder

clinical features of cardiac tamponade -etiology: malignancy/radiation therapy, infection , drugs and connective tissue disae (eg, SLE and RA) -C/Ps: beck's triad (JVD, hypotension, diminished heart sound). Pluses paradoxes (abnormally large inspiratory decrease in systolic BP > 10 mm Hg) -Diagnosis: ECG (low voltage QRS complexes, electrical alternates) and chest x-ray (enlarged water bottle shaped heart, clear lungs) This patient presentation of hypotension, tachycardia, jugular venous distention with clear lungs and pulses paradoxes (manifesting as loss of palpable pulse during inspiration) is consistent with cardiac tamponade. - this is most likely d/t patient's recent viral illness causing viral pericarditis with significant pericardial fluid accumulation. Normally there is a <10 mm Hg decrease is systolic blood pressure during inspiration. pulses paradoxes refers to an abnormal exaggerated decrease in systolic blood pressure of over 10 mm Hg on inspiration, a common finding in patients with large pericardial effusions causing tamponade. Inspiration causes an increase in systemic venous return, resulting in increased right heart volumes. Under normal conditions, this results in expansion of the right ventricle into the pericardial space with little impact on the left side of the heart. However, in conditions that impart expansion into the pericardial space (eg, cardiac tamponade), the increased right ventricular volume that occurs with inspiration leads to bowing of the intraventricular septum toward the left ventricle. This leads to decrease LV end diastolic volume and forward stroke volume, with a resultant decrease in systolic pressure during inspiration. In acute MI, presentations include substernal discomfort, diaphoresis and/or dyspnea, and can progress to cariogenic shock (hypotension, tachycardia, JVD and pulmonary edema). patients can have weak or undetectable peripheral pulses; however, pulses paradoxes is typically not seen. Acute viral myocarditis can lead to severe LV systolic dysfunction and present with acute CHF and cariogenic shock. Patients with severe LV failure can have pulses alternates - a variation in pulse amplitude with alternate beats. Pulses paradises is typically not seen. Pulses paradoxes can also occur in patients with asthma, COPD and constrictive pericarditis. Constrictive pericarditis is caused by scarring and loss of elasticity of the normal pericardial sac: it takes several weeks to months to develop and only rarely occurs after recurrent episodes of acute pericarditis the intimal stage of septic shock is hyper dynamic circulatory state with lowered systemic vascular resistance and increased CO (warm shock). weak pulses and pulses paradoxes would not be expected In tension pneumothorax, breath sounds are typically absent on the affected side with hype resonance to percussion. Other features include tachypnea, tachycardia, hypotension, distended neck veins, and tracheal deviation to the contralateral side,

96 A 34 y.o man comes into the emergency complaining of fatigue and lightheadedness. He had an upper respiratory infection last week, and since then his energy level has been low with shortness of breath on mild exertion. His BP is 80/60, pulse is 120 and regular. His pulse becomes undetectable to palpation during inspiration. the lungs are clear to auscultation, but the jugular veins are distended. Which of the following is the most likely diagnosis? a. acute MI b. acute viral myocarditis c. cardiac tamponade d. constrictive pericarditis e. septic shock f. tension pneumothorax

Dude has anal squamous cell carcinoma given the duration of pain, itching and rectal bleeding gin addition to the visible ulcerative mass (anal cancers are ulcerative in over 50% of cases). Anogenital SCC and they precursors, squamous intraepithelial lesion, have been linked to HPV. The HPV 16 and 18 serotypes are the guys that are strongly correlated with invasive carincoma. Immunodeficiency states increase host susceptibility to HPV infection and more severe infection HIV-positive men who have sex with men are at increased risk of developing anal squamous cell carcinoma (anal intercourse is hypothesized to be related), and HIV positive women are more prone to developing cervical SCC. a. adenoviruses can cause severe upper respiratory illnesses, pneumonia and disseminated infection in immunosuppressed patients b/e: HIV patients often experience reactivation of latent EBV infection. EBV replication in such patients is associated with certain lymphomas and with oral fairly leukoplakia, which typically manifests as white plaques on the lateral tongue margins (this patient's white buccal plaques on initial diagnosis were more likely thus d/t Candida). c. C. trachoma's causes lymphogranuloma venereum, characterized by ulcerative or vesicular lesion on the external genitalia follows y significant regional lymphadenopathy. Proctitis with tenesmus and bloody discharge can be seen with both LGV and anal carcinoma; however, LGV has a less indolent clinical course, and painful lymphadenopathy is a cardinal feature d. HIV positive patients with less than 100 CD4 cell count are at significantly increased risk of developing CMV infection, which most frequently causes retinitis in AIDS patients. CMV involvement of the GIT typically manifests as esophageal ulcers or colitis, not anal masses. f. although AIDS increases the prevalence of HSV-2 infection and the frequency of symptomatic genital herpes recurrences, HSV-2 is not typically associated with anogenital carcinoma. A large, hard mass in not characteristic of herpetic session (classically painful vesicles or ulcers).

1723 A 34 y.o man has white plaques on his buccal mucosa. After appropriate workup, he has HIV with CD4 count of 280. He returns 3 years later d/t several months of pain and itching in the perirectal area. He also has intermittent rectal bleeding and often sees bright red blood on the tissue after wiping. PE shows a single, hard mass with superficial ulceration measuring approximately 2x2 cm is noted in the anal canal. No hemorrhoids are present. There is no palpable regional lymphadenopathy. Which of the following pathogens is most likely responsible for this patient's current anal pathology? a. adenovirus b. candida albicans c. chlamydia trachoma's d. CMV e. EBV f. HSV g. HPV

Characteristic of gram negative rods. Lactose fermenters (pink colonies) -Fast fermenter are: KEE (Klebsiella, E. coli and enterobacter) -Slow fermenter are: CS (Citrobacter and serratia) Lactose nonfermenters (white colonies) -Oxidase positive: pseudomonas aeruginosa -Oxidase negative 1) No H2S = shigella 2) H2S (black color) = Salmonella and Proteus Gram negative bacilli account for 5% of all community-acquired pneumonia. However, they are responsible for the majority of cases of nosocomial pneumonias and have a high mortality rate. Pneumonia caused by gram negative rods is rare in health individuals and usually occurs in patients with immunocompromisation (especially neutropenia) and impaired host defenses (eg, infants, the elderly, alcoholics). The patient's pneumonia is most likely caused by Klebsiella, a gram negative lactose fermenting bacillus. The presence of a thick capsule is seen as a clear zone on gram stain and also causes the carahceristic mucoid growth in culture. Klebsiella pneumonia classically affects alcoholics. The pathophysioogicla mech involves colonization of the oropharynx followed by micro aspiration of upper airway secretions. As the aspiration usually occurs while supine, the upper lobes are commonly affected. Hallmark of Klebsiella pneumonia is thick, mucoid, blood tinged sputum (currant jelly sputum) and lung tissue necrosis with early abscess formation. a/d: Chlamydia pneumonias and Mycoplasma pneumonias are common causes of atypical community acquired pneumonia. These organisms are not visible on gram stain and do not grow on conventional cultures. c. legionella pneumoophila is a gram neg bacillus that can cause pneumonia, general following aerosolization from a contaminated water supply. HOWever, it is not encapsulated organisms and does not produce mucoid colonies. P. aeruginosa commonly causes a nosocomial infection that is generally seen in patients with structural lung disease, immunocompromisation or intensive care hospitalization. S. aureus can cause pneumonia,. but it is gram positive and so is S. pneumoniae.

9989 A 65 y.o man ha fever, chills, chest pain and productive cough for the last 2 days. He drinks 6-10 beers daily and has a 45 pack year smoking history. His temp is 104, BP 100/70, HR is 94 and RR is 28. There are crackles and decreased breath sounds in the right upper lobe Chest x-ray shows a large right upper lobe infiltrate. Sputum examination shows encapsulated gram negative bacilli. Sputum culture grows pink colored mucoid colonies on MacConkey agar. Which of the following organisms is the most likely cause of this patients condition? a. Chlamydia pneumonia b. Klebsiella pneumonia c. Legionella pneumonphila d. Mycoplasma pneumonia e. Pseudomonas aeruginosa f. S. aureus g. S. pneumoniae

Juvenile pilocytic astrocytomas usually arise in the cerebellum, brainstem, hypothalamic region or optic pathways. Microscopically, pilocytic astrocytomas are well differentiated neoplasms comprised of spindle cells with hair like glial processes that are associated with micro cysts. these cells are mixed with Rosenthal fibers and granular eosinophilic bodies. b. GBMs occur dos often in the subcortical white matter of the cerebral hemisphere When a tumor in the frontal cortex spreads across the corps callous into the contralateral hemisphere, it creates the appearance of a bilateral symmetrical lesion, here the term "butterfly glioma." GBMs are composed of poorly differentiated, often pleomorphic astrocytic cells with marked nuclear atypia and brisk mitotic activity. Necrosis is an essential diagnostic feature, and prominent microvascular proliferation is common. c. medulloblastoma (primitive neuroectodermal tumor) is the second most common posterior fossa tumor in children. ON microscopic examination cells are small and poorly differentiated, with scant cytoplasm and little storm. A high mitotic index is common. Classic Homer-Wright rosettes can be seen in one fifth of cases d. ependymomas account for 10% of posterior fossa tumors in children. Intracranial lesions usually occur infratentiorially, arising from the roof too the fourth ventricle in children, while spinal ependymomas typically occur in adults. histologically, ependymal pseudo rosettes with glial fibrillar acidic protein (GFAP)-positive processes tapering toward blood vessels characterize ependymomas. e. Neuroblastomas are the most common extra cranial solid tumors occurring in children. Undifferentiated neuroblastoma are small, round, blue cell tumors. A neuritic process also called neuropil, is apathy pathogonomic feature of neuroblastoma cells. NSE< chromagranin, snaptophysin and S-100 immunohistochemical stains are usually positive. More than 90% of patients have elevated urianry levels of homovanillic acid (HVA) and/or vanillylmandelic acid (VMA). The most important biologic marker is MYCN (N-MYC) of chromosome 2.

1153 A 14 y.o kid as nausea, vomiting, generalized headache, ataxia and visual complaints. MRI shows a cystic tumor in the cerebellum. Biopsy showed Rosenthal fibers and granular eosinophilic bodies. a. pilocytic astrocytoma b. glioblastoma multiform c. medulloblastoma d. ependymoma e. neuroblastoma

68/95/99.7

1172 What is the rule for Gaussian distribution?

Bacterial vaginosis: white/fishy odor. No inflammation. -Labs: pH over 4.5. Clue cells. Positive whiff test (amine odor with KOH) -Treat with metronidazole or clindamycin Trichomoniasis: yellow-green malodorous, frothy discharge. Inflammation. -Labs: pH over 4.5. Motile trichomonads -treat with metronidazole (also sexual partner) Candida vaginitis: thick, cottage cheese. Inflammation -Labs: pH 3.8-4.5. Pseudohyphae Treat fluconazole b. a wet mount with epithelial cells and rare leukocytes is consistent with normal vaginal discharge (leukorrhea). c. Gonorrhea cervicitis is characterized by cervical eryth/friability, purulent vaginal discharge, and a Gram stain that shows gram neg intracellular diplococci. d. a wet mount with leukocytes and pear shaped organisms describes trichomonad vaginitis. e. pseudohyphae with leukocytes are seen in candida vaginitis.

11765 Sexually active 15 y.o girl is having grayish white, fishy smelling vaginal discharge. Associated with mild pruritus. She has one male partner who is inconsistent with condom use. Cervix is not inflamed a. epithelial cells covered with gram variable rods b. epithelial cells with rare leukocytes c. gram negative intracellular diplococci d. leukocytes and pear shaped organisms e. pseudohyphae with leukocytes

Temporomandibular disorder (TMD) is characterized by a constellation of symptoms that may include unilateral facial pain that worsens with jaw movement, headache and ear discomfort. Symptoms can originate from temporomandibular joint (TMJ) derangement, pathologic contraction of the muscle of mastication and/or hypersentitivy of the nerves that supply the jaw The mandibular nerve is the largest branch of the trigeminal nerve and contains both motor and sensory components. It supplies sensation to the TMJ and manidublar teeth as well as the floor of the mouth, inside of the cheeks, anterior tongue, and much of the skin of the lower part of the face. The motor fibers innervate the muscle of mastication (medial and lateral pterygoid, master, temporals), muscles of the floor of the mouth (i.e., mylohyoid), tensor vili palatine and the ensor tympani in the middle ear. As a result, patient with TMD involving the mandibular nerve can have both jaw pain and otological symptoms. a. cervical spine disease causes neck pain, referred pain to the shoulder and arms, and upper extremity motor weakness b. facial nerve, which is primarily responsible for facial expression, carries somatic signal from the external auditory canal and the pinna but is not involved in the sensory innervation of the TMJ or the middle ear. d. maxillary division of the trigeminal nerve carries mainly sensory fibers from the cheek, nares, upper lip and teeth, the pharyngeal palate and the maxillary sinuses. e. the vestibulocochlear nerve provides auditory and position sense but is unrelated to the TMJ and not involved in TMD

11663 A 26 yo woman has pain and muffled hearing in left ear She also describe a sensation of jaw clicking when chewing food. PMH is significant for episodic headaches and left sided facial pain for several years that are worse when moving the jaw. She grinds her teeth at night while she is sleeping. On PE she is unable to open her mouth fully and develops pain with passive movement of the jaw. The external auditory canal and tympanic membrane are normal. Ring and Weber are normal. Left medial pterygoid muscle dysfunction is suspected. Involvement of which of the following nerves is causing her ear symptoms? a. cervical spinal nerve b. facial c. mandibular division of trigeminal d. maxillary division of trigeminal e. veestibulocochlear

Lab abnormalities in cirrhosis: Indicators of liver function: Impaired biosynthetic capacity (elevated PT and hypoalbuminemia). Impaired transport and metabolic capacity (elevated bilirubin). Indicators of liver injury: markers of hepatocyte injury (elevated AST > ALT). Markers of cholestatiss (elevated ALP and GGT) Others: thrombocytopenia (d/t splenic sequestration and of platelets) Alcohol-associated hepatic injury evolves through he stages of alcoholic steatosis (reversible), alcoholic hepatitis (reversible), and alcoholic cirrhosis (irreversible). Lab findings in cirrhosis reflect both hepatocellular/biliary injury and loss of hepatic function Hepatocyte injury causes a release of intracellular enzymes and an increase in serum transaminases; biliary injury is reflected by increases in ALP and GGT. Although these lab studies are indicative of ongoing hepatobiliary injury, they do not provide information on the liver's functional reserve, a key determinant of prognosis in patients with cirrhosis. Serum albumin levels and PT are better indicators of liver's biosynthetic function, and its ability to transport and metabolize organic anions is reflected by serum bilirubin level. Hypoalbuminemia, elevated bilirubin, and prolonged PT are signs of inadequate liver function (eg, liver failure) and indicate a poor prognosis in cirrhotic patients. For this reason, they are included in multiple scoring systems used to assess the severity liver failure and need for transplantation. a. AST indicates hepatocellulalr injury and relate ion intracellular enzymes into the blood. AST is classically elevated to more than 2 times the level of SLT in alcoholic liver disease. Transaminase elevation do no reflect liver function and do not predict outcomes; these enzymes can be transiently elevated by a number of self-limited processes. b. fibrinogen is a coagulation factor and an acute phase protein. Liver fibrinogen synthesis is increased in response to infection or acute inflammation and diminished in liver failure. Therefore, low fibrinogen levels would be expected in liver failure. When the ALP is elevated, serum GGT measurement can differentiate whether the cause is biliary disease or an alternate cause such as bone disease. GGT elevations are more specific biliary injury The BT is measurement of platelet function (not liver synthetic function) and is often prolonged in severe alcoholic liver disease, although with a fair degree of variance. Thrombocytpoenia in alcoholism develops d/t both direct toxic effects of alcohol and the bone marrow and hypersplenism with splenic sequestration of platelets.

1423 Dude has liver cirrhosis from being a heavy alcoholic for 40 years. Which of the following lab finding would be most indicative of a poor prognosis for this patient? a. high aspartate aminotransferase b. high fibrinogen levels c. high gamma glutamyl transferase d. prolonged BT e. Prolonged PT

Symptoms of digoxin toxicity: -cardiac: life treating arrhythmias -GI: anorexia, nausea, vomiting and abdominal pain -Neurologic: color vision alterations, confusion, fatigue, weakness Patient has aFib and CHF is having digoxin toxicity. Although calcium channel blockers and metal blockers are the preferred treatment for patients with aFib with rapid ventricular response, digoxin is a common second line treatment that is particularly useful in patients with underlying systolic cardiac dysfunction. Its major effects are: 1. increased vagal tone, causing slowing of conduction through the AV node (rate control affect) 2. Na-K-ATPase inhibition, causing increased intracellular sodium and calcium (increases cardiac contractility) Digoxin has a narrow therapeutic index, making toxicity relatively common. Symptoms of digit intoxication are fairly nonspecific and include nausea, abdominal pain, fatigue, dizziness, confusion, blurred vision nd abnormal color perception. Toxicity can cause a wide range of cardiac arrhythmias, including brady cardia and junctional escape beats d/t increased AV nodal block. -hypokalemia can precipitate toxicity by increasing digoxin binding to Na-K-ATPase. However, ELEVATED POTASSIUM is a sign of digoxin toxicity as inhibitor of the pump increases extracellular potassium. a. amiodarone can cause pulmonary toxicity, thyroid dysfunction, cardiac arrhythmias, elevated liver enzyme and visual disturbances. However, it doesn't cause hyperkalemia b. acute aspirin overdose presents with vertigo, tinnitus, vomitng and diarrhea, whereas severe intoxication can lead to coma, hyperpyrexia, pulmonary edema and death d/f: beta blocker and/or calcium blocker overdose usually cause profound bradycardia and hypotension (absent in the patient), In addition,nausea and vomiting are more common in digoxin overdose than in beta blocker overdose, which typically causes bronchospasm and hypoglycemia. E: furosemide overdose causes volume depletion, along with electrolyte abnormalities such as hypokalemia and hypomagnesemia. g. spironolactone can cause hyperkalemia, gynecomastia, impotence and decreased libido h. Valsartan is an angiotensin II receptor blocker. Overdose usually causes hypotension, renal failure and hyperkalemia.

147 A 76 y.o woman has lethargy and confusion over the past 2 days. She has persistent atrial fibrillation and CHF and is on multiple meds. Recently, she has had nausea and decreased appetite. Today, she had 3 episodes of vomitng. The patient also report vision difficulties. labs shows hyperkalemia ECG shows junctional escape rhythm at a rate of 48/min with occasional ventricular premature beats. BP is 133/80, T is 98. Increased blood level of which of the following medications is most likely responsible symptoms? a. amiodarone b. aspirin c. digoxin d. diltiazem e. furosemide f. metoprolol g. spironolactone h. valsartan

Long distance running and other forms of aerobic exercise cause increased oxidative metabolism of glucose and fatty acids in skeletal muscle. The skeletal m muscles increase their rate of both oxygen consumption and CO2 production. These increases are balanced by increases of the cardiac output/skeletal muscle perfusion and ventilation, respectively. Homeostatic mechanisms maintain arterial O2 and CO2 contents and arterial pH near normal resting values but there are significant change in the venous blood O2 and Co2 connects and pH. Because exercising muscles extracts additional O2, the venous blood O 2 content is decreased. Th venous blood CO2 content is increased d/t increased CO2 production. The venous blood pH is decreased. The arterial pH is usually normal in moderate exercise but may decrease during strenuous exercise d/t lactic acidosis Physiologic dead space corresponds to regions of alveolar ventilation that do not engage in gas exchange d/t relatively low alveolar perfusion and/or alveolar hyper aeration. The hyperventilation that accompanies exercise may slightly increase anatomic dead space by stretching the conduction fairways with fuller inspirations, but doesn't significantly increase the physiologic dead space. In healthy individuals, exercise causes a uniform simultaneous increase in both ventilation and perfusion of individual alveoli.

1590 Dude runs five miles. Which of the following do you expect to increase at the peak of his exertion? a. CO2 content in the arterial blood b. O2 content in the arterial blood c. CO2 content in the mixed venous blood d. pH of the arterial blood e. physiologic dead space

this dude is experiencing paroxysmal supra ventricular tachycardia. Treatment can be with vagal stimulation , such as carotid sinus massage or the valsalva menacers. The maneuvers increase vagal tone increase the refractory period of the AV node and help prevent a entrant circuit from conducting. If Valsalva measures fail, intravenous administration of adenosine is recommended. The valsalva maneuver is executed by forcibly exhaling against a closed glottis. This is done by taking full inhalation, closing the glottis (i.e. holding one's breath) and subsequently bearing down without exhaling as one would during a bowel movement. The rectus muscles are recruited in this process, and they play the largest role in development of the resultant elevated intraabdominal and intrathoracic pressure during this maneuver. b. Iliac muscle, a flexor of the hip that lies over the iliac fossa c. gluteus minimum is innervated by the superior gluteal nerve and has the subtle, but important function of preventing the contralateral (non weight bearing) side of the pelvis from dipping when that leg is elevated of the ground, as when one walks. d. gluteus medium has the same innervation and function as gluteus minus e. gluteus maximus is innervated inferior gluteal nerve and is the major extensor of the thigh at the hip.

1730 Dude BP 100/70 and HR is 160. Physician wants him to do the valsalva maneuver. Why and what is the most important muscle in performing the Valsalva maneuver? a. rectus b. iliac c. gluteus minimus d. gluteus medium e. gluteus maximus

CYP450 Inducers: barb, carb, rifampin, phenytoin, griseofulvin, St. John's wort, modafinil, cyclophosphamide Inhibitors: Amiodarone, cimetidine, fluoroquinolone, clarithromycin, azoles, grapefruit juice, isoniazid, ritonavir This patient is suffering from a thromboembolic stroke, likely the result of a dislodge left atrial thrombus that formed d/t an adverse drug interaction leading to inadequate anticoagulation. CYP450 interactions should be considered when selecting and dosing medications as changes in enzymes activity affect the rate of metabolism. Alterations in plasma drug levels are especially important when considering medications that are vital to a patient's health or have a high propensity for toxicity, such as warfarin. An example is warfarin can interact with St. John's wort, an over the counter medicinal herb known for both its anti inflammatory and antidepressantproeperties. It induces hepatic CYP450 resulting in increased warfarin metabolism, decreased drug levels and inadequate anticoagulation. ABCDE: are inhibitor of CYP450 enzymes and increase warfarin levels, leading to an increased risk of bleeding complications. Oral penicillin V and warfarin do not have any significant interaction. Antibiotics in general reduce the intestinal bacterial load, which reduces vitamin K synthesis and could potentiate warfarin's anticoagulant effects.

1777 A 65 y.o man has sudden onset of right side weakness and difficulty speaking. He has a history of paroxysmal atrial fibrillation and has been taking warfarin for the past several years with a stable PT. He started taking a new drug. PE right hemiplegia, right hemisensory loss, expressive aphasia and right homonymous hemianopia. MRI shows middle cerebral artery infarction. Transesophageal ACC reveals a small thrombus in the left atrium. a. amiodarone b. cimetidine c. ciprofloxacin d. clarithromycin e. fluconazole f. penicillin V g. St John's wort

Patient has excessive VIP secretion, most likely from a VIPoma that results in (WDHA): -watery diarrhea -hypokalemia -achloryhydria VIP stimulates HCO3 and Cl secretion via binding the secondary messenger cAMP causing sodium, chloride and water secretion into the bowel (secretory diarrhea) Somatostatin (octreotide) decreases the production of many GI hormones (eg, VIP, gastrin, glucagon, cck), its inhibitor of vip production by this lipoma is responsible for the resolution of this patient's symptoms a. cck causes increased secretion of pancreatic enzymes and bicarb, gallbladder contraction and inhibitor of gastric emptying. It however doesn't cause WDHA syndrome. It is produced by I cells of the proximal small bowel mucosa in repose to fatty acid and amino acids. b. gastric is produced by G cells in the stomach mucosa. It stimulates gastric acid production and growth of the gastro mucosa. Gastronomes classically cause intra table peptic ulcer disease (Zolliner_ellison syndrome). c. Gherkin is produced in the stomach and regulates food intake. Levels increase in fasting states and fall after eating. Antagonist of gherkin are being investigated as drug targets. d. glucagonoma ins apnacreatic aha cell tumor the thypersecretes glucagon and can cause secondary DM and necrolytic migratory erythema of the skin. Motion is produced by duodenal mucosa na stimulates smooth muscle contract in the upper git. antibiotic erythromycin acts as agonist at motion receptors in the stomach and duodenum, contributing to the drug's GI side effects.

1938 A 32 y.o woman has persistent tea colored, odor less watery diarrhea. Sampling of her gastric contents show a total lack of gastric acid secretion. Somatostatin treatment promptly relieves the symptoms. An excess of which of the following is responsible for her condition? a. cck b. gastrin c. gherkin d. glucagon e. motion f. vasoactive intestinal peptide

Deglutition (swallowing) is a complex process that involves 3 phases: 1. In the voluntary oral phase, the food bolus is collected in the back of the mouth and lifted upward to the posterior wall of the pharynx. 2. This initiates the pharyngeal phase, which consists of involuntary pharyngeal muscle contractions that propel the food bolus to the esophagus. 3. During the esophageal phase, food stretches the walls of the esophagus, stimulating just above the site of distention and moving the food downward. Relaxation of the lower esophageal sphincter (LES) follows, allowing the food bolus to enter the stomach. Abnormal spasm or diminished relaxation of the CRICOPHARYNGEAL MUSCLE during swallowing is thought the be the underlying mechanism of Zener diverticulum formation. This process results in early oropharyngeal dysphagia with a feeling of food obstruction at the level o f the neck and coughin/chocking. Increased oropharyngeal intraluminal pressure eventually results in herniation of the pharyngeal mucosa through a zone of muscle weakness (false diverticulum) in the posterior hypo pharynx (Killian triangle). Presentations: 1. halitosis/regurgitation. 2. pulmonary aspiration of diverticular contents may also lead to recurrent pneumonia. Complications: -As diverticulum enlarges, it may become palpable as a lateral neck mass, and dysphagia can worsen d/t luminal narrowing caused by extrinsic esophageal compression. d. degenerative changes of the myenteric plexus with impaired LES relaxation result in achalasia. Barium swallow typically shows esophageal dilation with esophagogastric junction narrowing giving the characteristic "bird's beak" appearance. c. mucosal tears around the gastroesophageal junction can be caused by increased intraluminal pressure in the stomach during prolonged or recurrent tretchign/vomiting (Mallory-Weiss syndrome). d. retention cysts form d/t accumulation of trapped secretions following obstruction of a gland's duct. Chronic rhino sinusitis frequently causes mucus retention cysts in the maxillary sinus. e. mediastinal lymphadenitis (eg, d/t Tb, fungal infections)) can cause scarring/traction of the mid portion of the esophagus, resulting in the formation of true diverticular (i.e., containing all gut wall layers).

279 a 74 y.o man dysphagia, choking spells and cough. PMH recurrent episodes of pneumonia, HTN and osteoarthritis. Has foul smelling breath, but his oropharyngeal and neck examinations are normal. barium swallow study reveals a diverticulum in the upper esophagus. a. cricopharyngeal motor dysfunction b. degenerative changes of the myenteric plexus c. increased intraluminal pressure in the stomach d. retention cyst d/t duct obstruction e. scarring and traction of the esophagus

This patient has acute Hep A. Morphology -ballooning denudation (hepatocyte swelling with wispy/clear cytoplasm) -Councilman bodies (eosinophil apoptotic hepatocyte) -Mononuclear cell infiltrates Presentation: -fever, anorexia, malaise, nausea/vomitng and RUQ abdominal pain -After several days, prodromal symptoms lessen as patients develop signs of cholestatiss such as jaundice, pruritus, dark colored ruine (d/t increased conjugated bilirubin levels), and echoic stool (lacks bilirubin pigment). Prognosis: self limiting a/c: chronic cholestatic processes (eg, primary biliary cirrhosis, primary sclerosing cholangitis) may present initially with prolonged pururits and fatigue. As these conditions progress, patients can develop echoic stool and malabsorption of fat soluble vitamins (d/t reduced bile flow to the SI). This cause decrease in vitamin D and osteomalacia with bone pain. d. skin pigmentation and DM are more suggestive of hemochromatosis, an inherited disease associated with iron overload in the live,r pancreas, heart and other organs. e. upper GI bleeding and ascites are more suggestive of portal HTN, a condition that typically develops secondary to liver cirrhosis.

366 A 43 y.o man has history of traveling to South America for missionary trips PMH is insignificant. Father is alcoholic complicated by cirrhosis and portal HTN. Biopsy of patients liver shows spotty hepatocyte necrosis and inflammatory cell infiltration. Which of the following is clinical presentation of the patient? a. clay colored stool and diffuse bone pain b. fever anorexia and dark colored urine c. prolonged pruritus and fatigue d. skin pigmentation and DM e. upper GI bleeding and ascites

Atherosclerosis is a chronic inflammatory and fibroproliferative disease. Endothelia cells, vascular smooth muscle cells (VSMCs), and leukocytes play an important role in the development and progression of this disease. The process is initiated by chronic hemodynamic stress and hyperlipidemia, which cause ENDOTHELIAL CELL INJURY. Leads to expression of surface vascular cell adhesion molecules (VCAMs) that all adherence and migration of MONOCYTES and T lymph into the intima. The infiltrating leukocytes and dysfunctional endothelium release cytokines and growth factors (eg, platelet-derived growth factor, fibroblast growth factor, endothelia 1, IL-1) that promote MIGRATION AND PROLIFERATION of VSMCs within the intima. VSMCs are also stimulated to synthesized EXTRACELLULAR MATRIX PROTEINS (eg, collagen, elastin, proteoglycans) that form the fibrous cap typical of mature atheroma. Disruption of the fibrous cap with luminal thrombosis (ATHEROSCLEROTIC PLAQUE RUPTURE) has catastrophic clinical consequences and is responsible for the majority of acute coronary syndrome cases and sudden cardiac death. a. endothelial cell dysfunction is the earliest pathophysiologic change that precedes the formation of atherosclerotic lesions. Endothelial injury results in increased lipid permeability, leukocyte (monocytes, T cells) and platelet recruitment, and migration and proliferation of VSMC from media to the intra. However, the VSMCs are directly responsible for the synthesis of new collagen and extracellular matrix b. fibroblasts are found infrequently in the tunica initma of BVs and are not significantly involved in atherosclerosis pathogenesis c/e: activated macrophages and T lyme secrete growth factors that recruit VSMCs reasonable for forming the fibrous cap, but they are not directly responsible for the dense deposition of extracellular matrix and collagen. macrophages also produce matrix metalloproteinases and tissue factors that degrade the extracellular matrix, causing the formation of large, soft lipid rich core with thinning of the fibrous cap Such vulnerable plaques have an increased chance for rupture.

443 A guy was found dead in his home. Pathological examination shows complete thrombotic occlusion of the left main coronary artery and diffuse atherosclerotic vascular disease characterized by multiple atheroma. Along with a lipid core, these atheroma have a fibrous cap with dense deposition of extracellular matrix and collagen. Which of the following cells are directly responsible for the latter finding? a. endothelial cells b. interstitial fibroblasts c. macrophages d. smooth muscle cells e. t lymphocytes

Humoral Response: Type 1: IgE Type 2: IgG and IgM (autoantibodies) = complement activation Type 3: immune complexes deposition = complement activation Type 4: no humoral response Cellular components: Type 1: basophils and mast cells Type 2: NK, eosinophils, neutrophils and macrophages. Type 3: neutrophils Type 4: T-CELLS, MACROPHAGES (unique)

567 List the 4 types hypersensitivity reactions.

Patient is having peri-infarction pericarditis that improve with supportive measures, those with sufficient or persistent symptoms are typically treat with aspirin in combination with colchicine. Acetylsalicylic acid - is an NSAID that irreversibly inhibits COX-1/2 by acetylation, preventing the conversion of archindonic acid to prostaglandins, prostacyclin and thromboxane. COX-1 acetylation inhibits generation of TXA2, in platelets (antithrombotic effect). COX-2 acetylation blocks prostaglandin production in inflammatory cells (eg, activated lymphocytes and neutrophils), resulting in anti-finlammatory, antipyretic and analgesic effects. a. tylenol is an analgesic and antipyretic agent that reversibly inhibit COX primarily in the CNS. It lacks the anti-inflammatory properties d/t its weak inhibition of COX in peripheral tissue. c. budesonide is a glucocorticoid agent with potent anti-inflammatory activity. Corticosteroids reduce COX-2 expression and decrease synthesis of arachidonic acid via phospholipase A2 inhibition. d. Celecoxib is a selective COX-2 inhibitor that provides anti-inflammatory benefits without interfering with the physiological functions of COX1. In contrast to to non selective NSAIDs, selective COX2 inhibitors have no significant effect on platelet function and are associated with lower incidence of GI bleeding. However, selective COX2 inhibitor is associated with an increased in thrombosis. e. colchine exerts its anti inflammatory effects by inhibiting polymerization of beta-tubulin into microtubules, preventing migration and activation neutrophils. f,g,h: diclofena, ibuprofen and indomethacin are NSAIDs that reversibly inhibit COX1/2

714 A 52 y.o man recovering from an acute MI develops chest pain that is received by leaning forward. On PE, a friction rub is present at the left sternal border. Th patient is treated with a medication that inhibits COX-1 & COX-2 enzymes via irreversible acetylation, resulting gin significant relief of his symptoms. a. acetaminophen b. aspirin c. budesonide d. celecoxib e. colchicine f. diclofenac g. ibuprofen h. indomethacin

Statins are the first line therapy for hypercholesterolemia and are useful in primary and secondary prevention of acute coronary events. MOA: inhibiting HMG-COA reductase, the rate limiting enzyme in the synthesis of cholesterol. It lowers cholesterol, LDL and TAGs levels. Primary side effects: -myopathy -haptitis Statin associated myopathy is usually characterized by mild muscular pain and resolves with discontinuation of the medication. However, some patients will develop severe myopathy with striking elevation in CK levels and occasional rhabdomyolysis. The risk of severe myopathy is increased when statins are given concurrently with vibrate (especially gemfibrozil), which impair the hepatic clearance of statins and lead to excessive blood levels. An increased risk of statin myopathy is also likely with concurrent use of niacin or ezetimibe, but to a lesser extend. a. concurrent use of bile acid sequestrates with statins doesn't increase the risk of myopathy. However, bile acid sequestrates can reduce GI absorption of statin d. vibrates can increase drug levels of ezetimibe but this don'ts have significant impact on its side effects, which are generally mild and not dose related. Ezetimibe doe not alter the pharmacokinetics of vibrates and doesn't substantially influence their adverse effects. e/f. the primary adverse effects of niacin include flushing, hyperglycemia and hepatotoxicity. There are no significant interactions with the concurrent use of niacin and ezetimibe or gemfibrozil.

778 A 68 y.o man has muscle pain and fatigue. PMH HTN, mixed hyperlipidemia, and CAD with an acute MI 2 years ago. PE shows diffuse tenderness in the proximal muscles of the upper and lower extremities. Serum creatine kinase activity is elevated. Which of the following drug combinations is most likely responsible for this patient's condition? a. atorvastatin and cholestyramine b. atorvastatin and ezetimibe c. atorvastatin and gemfibrozil d. gemfibrozil and ezetimbie e. niacin and ezetimibe f. niacin and gemfibrozil

the qrs complex duration is typically slightly reduced during exercise in response to the increase in cardiac conduction velocity the accompanies faster HRs. However, this patient's QRS duration is normal at rest and prolonged at near maximal HR. This suggests that the drug used to treat his afib lengthens the qrs duration in a rate dependent manner, which is consistent with a drug exhibiting strong use dependence such as flecainide. Flecainide is class 1c antiarrhthmic that is typically used to treat supra ventricular tachycardias such as those caused by afib. Class 1c binds strongly to the fast sodium channels responsible for phase 0 depol of cardiac myocytes, blocking the inward sodium curren and prolonging qrs complex. They are the slowest of the class 1 agents to dissociated fro Na channel. This results in a phenomenon known as use dependence, in which their sodium blocking effects intensity as HR increases d/t less time between AP for the medication to dissociate form receptor a. selective beta 1 blockers such as atenolol od not significantly affect qrs duration. However, their use during exercise ECG testing can limit the max hr that is achieved b. digoxin can cause false positive st segment depression during exercise egg testing as well as slower the max achievable hr. it doesn't significantly affect qrs duration c. dofetilide is a class iii natiarrhthmic drug that blocks the outward potassium current during repeal. as a result, it increases qt interval duration but doesn't affect qrs duration. class III cents demonstrate reverse use dependence (the slower the hr, the more qt interval is prolonged). e. verapamil class iv is a Ca channel blocker that slows the SA and AV nodal conduction. It also increases coronary blood flow and reduces myocardial oxygen demand, which can mask ischemia during stress testing. verapamil doesn't affect qrs duration.

8869 a 45 y.o man is started on meds for paroxysmal afib undergoes stress ECG testing. the patient exercises on a treadmill for 9 minutes and reaches 98% of age predicted maximal HR whiteout chest pain or ST-segment changes. His pre testing resting QRS and QT are normal. However, at his maximally achieved hr, his qrs complex increase. Base on the test, which of the following meds is most likely being used to treat his atrial fibrillation? a. atenolol b. digoxin c. dofetilide d. flecainide e. verapamil

Listeria monocytogenes is a facultative intracellular, gram +, that produces a very narrow zone of beta hemolysis on blood agar, resembling pattern produced by group B strep. -immotile at 37 c but has tumbling motility at 22 c, a fact that can be used to identify potential Listeria isolates. -It can multiple at temp as low as 4c, which allows the bacteria to contaminate refrigerated food (eg, meats, unpasteurized milk, soft cheese, raw vegetables). Intracellular bacteria are protected against circulating immune factors such as Abs and complement but must develop survival mechanism in order for them to survive inside of macrophages, such as: -fusion of phagosomes with lysosomes (eg, salmonella and Tb) -inhibiting phagolysossome acidification (Tb) -or escaping from the phagosome into the cytosol (eg, Literal and Shigella). Listeria is able to lyse the vacuolar membrane through the action of Listeriolysin O, a pore forming toxin that is selectively activated within codified phagosomes. In healthy people, Listeria infection stimulates the production of cytokines (IFy, TNF-b, IL-12) that induce cell mediated immune redone leading to macrophage activation and killing of intracellular Listeria. However, in pateitnets that are immunocompromised, elderly and pregnant woman, the organism survives and can cause serious infections. Neonates can acquire Listeria transplacentally or during delivery; they are especially vulnerable to developing meningitis and septicemia because their cell mediated immunity is not yet fully developed. b. eosinophil activities are important against helminths c. Paitents with pure humoral immune defects (eg, X-linked agammaglobulinemia of Bruton) are at increased risk of bacterial infections by STrp and Staph but have normal responses to infection combated by cell mediate immunity (eg, lister, virus and fungi) d. Mast cells contribute to defuse against parasites and play an important role in allergic response. They do not confer immunity to intracellular pathogens e. patients with inherited def of the terminal complement components (eg, C5b-C9( are unable to form the membrane attack complex and are predisposed to recurrent Neisseria infections.

1391 A 54 y.o man has fever, chills, and malaise for the last 24 hours. He was recently diagnosed with Hodgkin's disease. Temp is 103 and BP 102/61. Blood cultures grown motile gram-positive rods that produce a very narrow zone of beta-hemolysis on blood agar. the bacteria are also found to grow well at refrigeration temp. Which of the following processes is most important in eliminating these bacteria from the body? a. cell mediated immunity b. eosinophil action c. Ig secretion d. mast cell activation e. terminal complement cascade

Lynch syndrome (HNPCC) is an AD disease caused by defective DNA mismatch repair. DNA replication occurs with a high degree of fidelity because mismatched nucleotides are repaired through the proofreading activity of DNA pol delta and epsilon. However this proofreading functionality is not infallible; base substations and small insertion and deletion mismatches occur d/t errors in base pairing every 10^6 bases on average. Function of the DNA mismatch repair system to fix these errors shortly after the daughter stands are synthesized. The mismatch repair system involves several genes, including MSH2 and MLH1, with code for component of the human MutS and MutL homoglos. Mutatons of these 2 genes account for around 90% of cases of lynch syndrome Mismatch repair begins with MutS homolog deleting a mmistmatch on the new created daughter strand, which is distinguished from the parent by occasional nicks in the phosphodiester bonds. Mute homolog is then recruited, and the resulting complex slides along the DNA molecule until 1 of the taught strand nicks in encounters. At this point, exonuclease 1 is loaded onto and activated by the repairs complex. The daughter strand is then degraded backward past the initial mismatch point, leaving a variable gap of ssDNA that is stabilized by ssDNA-binding protein. The complex then dissociated while DNA pol delta load the 3' end of the discontinuity and begins synthesizing a new daughter strand segment. Finally, DNA ligase I seals the raining nick to complete the repair process. b. exposure to UV light can cause pyrimidine (usually thymine) dimers to form d/t covalent joining of adjacent pyrimidines. Pyrimidine dimers interfere with DNA replication and are removed by nucleotide excision repair c/d: several types of insults can alter the DNA bases. For example, NO can deaminase C, A and G. There are also spontaneous changes, such as deamination of C to U and the constant low level loss of purines via thermal disruption. Glycosylases are enzymes that detect and remove abnormal bases from DNA, creating an empty sugar phosphate residue that is subsequently removed and replaced by the correct nucleotide (base excision repair). e. exposure to ionizing radiation causes double stranded DNA breaks that are repaired by end joining repair mechanism. Non homologs end going, the main mechanism in primates, is more prone to cause mutations than homologous recombination.

2028 Patient has lynch syndrome. FH is significant for colon endometrial and ovarian cancer. Which of the following is most likely responsible for the development of colon cancer in this patient? a. nucleotide mismatches that escape repair b. covalent bonds between adjacent pyrimidines c. insertion of abnormal bases (eg, uracil) into DNA d. empty sugar phosphate residues in the DNA molecule e. DS breaks in DNA

Benzo work at the GABAa, is the main NTs in the CNS and synthesized from glutamate using glutamate decarboxylase. GABAa receptor complex consists of 5 subunits and central chloride ion channel, with different binding sites for GABA and various drugs (eg, benzodiazepines, barbiturates). Bentos act by biding to the bento binding site, which allosterically modulates the binding of GABA, resulting in an increased frequency of chloride ion channel opening. The influx of Cl ions into the neurons causes neuronal hyperpolarization and inhibitor of the action potential. a. GABAb receptors mediate their actions using G proteins. The skeletal muscle relaxant baclofen is an example of a drug that works as a GABAb receptor agonist. Benz work at the GABA receptor. Benzos do not alter GABA metabolism. Certain anti-epileptic medications including valproic acid and vigabatrin, function to reduce gaba catabolism. Benzodiazepines do not function through direct activation of GABAa receptor. Instead, they work as positive allosteric modulators that promote an increased frequency of channel opening in response to GABA stimulation. Benzodiazepines do not function through the blockage of GABAa receptor lumen. Examples of drugs that directly block the lumen of ion channels include amiloride (which affects epithelial sodium channels in the distal portions of the nephron) and verapamil (a calcium channel blocker). Prolonged benzos usage has been shown to cause down regulation of GABAa receptors and may be one of the mechanisms by which benzodiazepine tolerance develops.

937 Bitch has a panic disorder. She receives medication and feels better within an hour. Which of the following best describes the MOA of the medication she received? a. G protein signal transduction b. alters GABA metabolism c. binds allosterically to th eGABA receptor d. competes with GABA at its biding site e. physical blocks the ion channel lumen f. up regulates GABA receptor density

ECG useful in diagnosing MI and can help localize the area of infarction. ST elevation typically represent acute MI< whereas prominent Q waves are suggestive of old MI. ST elevation in leads I and aVL = acute lateral MI from occlusion of the left circumflex artery. Because the chest leads V5-6 are also placed laterally, they may also show ST elevation during a lateral infarction. a/d. the LAD supplies the anterior aspect of the LV and inter ventricular septum (septal branches), which corresponds to the anterior chest leads (V1-V4). -proximal = all 4 -distal = V1-2 Left main coronary gives rise to the LAD and left circumflex, there ore, left main coronary occlusion would show ST elevation in the anterior (eg, V1-V4) and lateral (V5-V6, I and aVL) leads. The right coronary artery typical supplies the RV and inferior aspect of the left ventricle. Occlusion of this vessel results in an inferior MI, which corresponds to the ST elevation in the inferior leads (II, III, and aVF)

10467 Dude has MI where ECG shows ST elevation in leads I and aVL. Cardiac enzymes are elevated. Which of the following artery is most likely to be occluded? a. distal LAD b. left circumflex c. left main coronary d. proximal LAD e. right coronary artery

ACL originates on the lateral femoral condyle, and courses anterior and medially to insert on the anterior intercondylar area of the tibia. The primary function of the ACL is to prevent anterior motion of the tibia with respect to the femurs, though it also plays a role in stabilizing the knee against rotator and varsus/valgus forces. It is the most commonly injured in non contact injuries involving sudden decelerations and pivots on an extended knee. Following an CL tear, the knee will hoo laxity with the tibia able to easily pulled forward relative to the femur (eg, Lachman test, anterior drawer test) the middle geniculate artery provides the primary blood supply to the ACL, an injuries are characterized by rapid-onset hemarthrosis and knee swelling.

1697 Girl landed on her right foot with her toes rotated medially and her knee extended, and says she felt a popping sensation before falling to the ground. PE shows excessive anterior translation of the tibia relative to the femur on stability testing. What is wrong? a. anterior cruciate b. fibular (lateral) collateral c. oblique popliteal d. patellar e. posterior cruciate f. tibial (medial) collateral

The following disorders and their CVS developmental defects Down = endocardial cushion defects (eg, osmium premium atrial septal defects, regurgitant atrioventricular valves) DiGeorge = Tetralogy of Fallot and Interrupted aortic arch Friedrich ataxia = hypertrophic cardiomyopathy Kartagener syndrome = situs inverses Marfan = cystic medial necroses (eg, aortic dissection, aneurysm). Mitral valve prolapse Tuberous sclerosis = valvular obstruction d/t cardiac rhabdomyomas turner = aortic coarctation and bicuspid aortic valve. This patient most likely has congenital coarctation of the aorta, which typically affects the region of the aorta just distal to the left subclavian artery. Although traditionally classified into pre ductal and post ductal types, the signs and symptoms of aortic coarctation depend more on the age of presentation, which varies according to the severity of the stenosis. Severe coarctation usually presents in infancy with differentials cyanosis affecting the lower extremities as long as the ductus arterioles remain patent. On ductal closure, these neonates can develop signs of heart failure and shock. More moderate can develop signs of heart failure and shock. More moderate stenosis often presents in childhood or adolescence with symptoms of lower extremity claudication (eg, pain and cramping its exercise), BP discrepancy between he upper and lower extremities, and delayed or diminished femoral pulses Continuous murmur and pulsatile intercostal collateral scan also develop secondary to restricted circulation. Congenital aortic coarctation occurs in up to 10% of patients with Turner syndrome (45 XO) DiGeorge is associate with interrupted aortic arch, a mor extreme anomaly than aortic coarctation in which the aortic arch is atretic or a segment of the arch is absent. Affected patients have poor lower extremity pulses, but respiratory distress, variable cyanosis and signs of CHF will also develop during the first days of life. Down is associated with endocardial cushion defects that result in osmium premium atrial septal defects and regurgitant atrioventricular valves. Friedrich ataxia is marked by spinocerebellar degeneration with predominantly spinal ataxia. This can produce difficulty walking, but femoral pulses are not affected. This AR condition is associated with hypertrophic cardiomyopathy. Kartagener syndrome, cilia are immotile d/t a micro tubular dyne arm defect. Infertility, recurrent sinusitis, and cronchiectasiss occur as a result. This syndrome is associated with situs inversus. Marfan syndrome is associated with cystic medial necrosis of the aorta, which may result in aortic dissection and aortic valve incompetence. Mitral valve prolapse can also occur d/t the same degenerative process. Tuberous sclerosis is an AD syndrome characterized by cutaneous angiofibromas (adenoma sebaceous), seizures, and mental retardation. Pathological lions include CNBS hamartomas and benign neoplasms, renal and other visceral cysts, and cardiac rhabdomyomas.

30 A 12 y.o girl has pain in her legs. PE shows pulsatile vessels within the intercostal spaces and diminished femoral pulses relative to brachial pulses. This patient's symptoms are most likely associated with which of the following conditions? a. digeorge b. down c. friedreich ataxia d. Karatgener e. marfan f. tuberous g. turner

The SMA leave the aorta at the level of L1 and supplies the intestine from the duodenum and pancreas to the left colic flexure. The transverse portion of the duodenum lies horizontally at the level of L3, between the aorta and SMA.. Normally, the SMA and aorta form an approximately 45 degree angle. If this angle diminishes to less than 20 degrees, the transverse portion of the duodenum can get entrapped between the SMA and aorta leading to symptoms of partial small bowel obstruction. This condition is called SMA syndrome. Narrowing of the aortomesenteric angle can occur with any condition that causes diminished mesenteric fact, including low body weight, recent weight loss, severe burns or other inducers of catabolism, and prolonged bed rest. It can also occur with pronounced lordosis or after surgical correction of scoliosis, as this procedure lengthens the spine resoling in decreased mobility of the SMA

303 A 32 y.o female has severe nausea and recurrent bilious vomiting. Symptoms initially began as postprandial epigastric pain and early satiety, but have progressed over the last 2 weeks. She works as an actress, and tells you that her symptoms only started when she landed a role in a soap opera and was inspired to lose 25 pounds. On PE, her abdomen is tender and slightly distended with high pitched bowel sounds. Concerned about small bowel obstruction she is admitted to the hospital. Laparotomy is performed, and it is observed that the angle between her superior mesenteric artery and her aorta is significantly decreased. Which of the following structures is most likely to be obstructed by the artery? a. ascending portion of the duodenum b. descending portion of the duodenum c. duodenal bulb d. duodenaljejunal flexure e. gastric antrum f. transverse portion of the duodenum

This guy has nephrotic syndrome characterized by edema and marked proteinuria (over 3.5 g/day). With the underlying malignancy, this patient most likely have membranous glomerulopathy. Up 85% of cases are idiopathic. It is also the most common causes of nephrotic syndrome in adults. The remainder occur secondary to the following things: 1. systemic disease: DM, said tumors (lung/colon), and immunologic disorder s(such as SLE) 2. Certain drugs - gold, penicillamine and NSAIDs 3. Infections - hep B, hep C, malaria and syphilis. His biopsy shows uniform, diffuse thickening of the glomerular capillary wall on light microscopy without an increase in cellularity, are consistent with a diagnosis of membranous glomerulopathy. EM will reveal thickening caused by irregular, dense deposits lead between the basement membrane and the epithelial cells. These protrusions resemble "spikes" when stained with silver. IF microscopy reveals that these granular deposits contain IgG and C3 d. membranoproliferazive glomerulonephritis, shows large hyperceullalr glomeruli (hence, "proliferative"). In membranous glemerulopathy, diffuse thickening of the capillary wall is seen without the increase of cellularity. Berger is painless hematuria in children and young adults a few days after an upper respiratory infection. IgA deposits mesangium are found in IF. Minimal change disease is the most common cause of nephrotic syndrome in children. NO abnormalities are found on light or IF microscopy. However, effacement of the foot processes of podocytes are seen on EM.

382 A 56 y.o patient has colon cancer and is suffering from generalize edema. His urine protein excretion is 4.5 g over 24 hours. Kidney biopsy shows glomerular capillary wall thickening without an increase in cellularity. When the sample is stained with silver methanamine, irregular spikes protruding form the glomerular basement membrane are seen. This patient most likely suffers from which of the following ? a. anti-GBM disease b. ANCA-assoicated glomerulonephritis c. membranous glomerulopathy d. membranoproliferazive glomerulonephritis e. postinfectious glomerulonephritis f. berger g. FSGS H. minimal change disease I. alport J. thin basement membrane syndrome K. amyloidosis L. multiple myeloma m. malignant HTN

Time after MI and predominant light microscopic changes: 0-4 hours: no visible changes 4-12: wavy fibers with narrow, elongated myocytes 12-24: myocyte hypereosinophilia with pyknotic (shrunken) nuclei 1-3ds: coagulation necrosis (loss of nuclei and striations) and prominent neutrophilic infiltrate. 3-7ds: disintegration of dead neutrophils and myofibers. Macrophage infiltration at border areas 7-10ds: robust phagocytosis of dead cells by macrophages. Beginning formation of granulation tissue at margins 10-14ds: well developed tissue with neovascularization 2weeks-2months: progressive collagen deposition and scar formation Sudden cardiac death 12 days after an MI is most likely d/t ventricular arrhythmias originating from the infarcted myocardium. The microscopic diagnosis of MI depends on the presence of necrotic myocardium, with areas of acute inflammation and necrosis separated from viable myocardium. After the initial event, several characteristic microscopic changes occur in the infarcted zone in a specific temporal sequence with some overlap in different stages. During the second week after MI, the damaged tissue is replaced by granulation tissue and neovascularization is found in the infarct zone. a. increased collagen deposition and decreased cellularity in the zone of infarcted myocardium generally become evident beginning at 2 weeks post infarction. Fibrosis continues during weeks 2-8, resulting in a dense collagenous scar at about 2 months post infarction. b. prominent interstitial neutrophil infiltration is found t the border zone of infarcted myocardium 1-3 days after MI onset. This acute inflammatory infiltrate becomes increasingly prominent over the next few days after receding around days 5-7. Edema and wavy myocyte fibers with normal appearing nuclei can be detected in infarcted myocardium by light microscopy approximately 4-12 hours following MI. Cytoplasmic hypereosinophilia is a characteristic finding of early MI and occurs 12-24 hours after onset. Macrophage phagocytosis of infarct myocardium begins at about 3 days post infarction. The macrophages become laden with degenerating myoglobin pigment and hemosiderin (not foam cells, which are indicative of extensive lipid engulfment). By day 10, the accumulated macrophages begin to be replaced by granulation tissue. Infarcted myocardium usually appears normal under light microscopy up to 4 hours after the onset of MI.

41 A 60 y.o man has acute substernal chest pain, nausea and diaphoresis. ECGS shows STEMI. Reperfursion therapy is not performed d/t underlying cirrhosis and history of vatical bleeding. The patient is eventually discharge from the hospital and conservative mangement. 12 days later he died. Light microscopy would most likely show which of the following changes in the myocardium of the inferior wall? a. dense collagen scar b. dense interstitial neutrophil infiltrate c. edema and wavy myocyte fibers d. granulation tissue with neovascularization e. hypereosniophilic myocytes f. macrophage infiltration with foam cell formation g. normal myocardium

The URT (eg, nasal passages, mouth pharynx and larynx) accounts for about half of the total airway resistance. the remainder derives from the LRT, which begins at the trachea and consists about about 23 generations of airways. Although resistance, within the trachea and mainstream bronchi is relatively high, it increases in the medium sized bronchi because of highly turbulent airflow. Resistance is maximal between bronchial generations 2-5, but drops in subsequent generations (eg, small bronchi, bronchioles) because the summated cross-sectional area massively increases. This greatly slows airflow velocity, allowing low-resistance laminar airflow to predominate in airways less than 2 mm in diameter.

481 Describe the resistance in the respiratory track

Patient has Conn syndrome - a very common cause of secondary HTN. Primary hyperaldosteronism secretes high amount of mineralocorticoids from bilateral nodular hyperplasia of the adrenal zona glomerulosa or an aldosterone producing adrenal adenoma. The main effect of aldosterone is to stimulate the absorption of sodium and excretion of potassium and hydrogen ions in the renal collecting tubules. Aldosterone secretion from the glomerulosa is normally regulated by angiotensin II and potassium levels. Overpoduciton of aldosterone can result in sodium retention, hypertension and feedback suppression of the RAAS system (i.e., very low renin activity). Some patients also develop metabolic alkalosis and hypokalemia , which can exacerbated by increased distal bubble sodium delivery (eg, diuretics, increased sodium intake). Clinical presentations: muscle weakness, cramps, occasionally rhabdomyolysis and cardiac arrhythmias. a/b. overactivity chromaffin and extra adrenal paraganglion cells will give you pheochromocytoma a catecholamines secreting tumors. Typical presentation is severe hypertension. But it is also associated with tachycardia and symptoms of catecholamines excess (eg, sweating, palpitations, headaches). c. JG cells will give you hypertension but also high renin levels. This is typically seen in renal artery stenosis with decreased renal blood flow. d/f. zona fasciculate and reticular will give cushing and hyperandrogegism, respectively. Cushing syndrome can cause hypertension and hypokalemia but is usually associated with weight gain and Cushingoid body habits (eg, central obesity and moon face).

547 Dude has recurrent episodes of muscle weakness. His BP is 190/110 mmm Hg supine and 195/110 standing. HR is 70 and 72, supine and standing, respectively. Labs shows very low plasma renin levels. Overactivity of which of the following structures is most likely? a. chromatin cells of the adrenals b. extra-adrenal paraganglion cells c. JG cells of the kidney d. Zona fasciculate of the adrenals e. zona glomerulosa of the adrenals f. zona reticularis of the adrenals.

amyloid is abnormally folded (insoluble) fibrillar protein that deposits in the extracellular space of tissues. When stained in Congo red and viewed under polarized light, these deposits have a characteristic of apple green birefringence d/t to their beta-sheet structure Amyloid deposition can affect multiple organs system, such as those involved in plasma cell tumors (eg, amyloid light chain protein (AL)) and chronic inflammatory disease (eg, amyloid-associated protein (AA)). However, in Alzheimer disease, amyloid deposits are seen exclusively in brain tissue. these deposits contain *beta-amyloid* (A-beta), which formed by cleavage of amyloid precursor protein ( a transmembrane glycoprotein). Early in the disease, neuritic (senile) plaques can be found in the medial temporal lobe (eg, hippocampus, amygdala, entorhinal cortex) and are composed of central ABeta core surrounded by dystrophic neuritis. Beta deposition can also occur in the media/adventitia of cerebral vessels (amyloid antipathy) and may cause vessel wearing with intracranial hemorrhage. Huntington, atrophy of the caudate and microscopy reveals intranuclear inclusions containing aggregates of hungtington protein Parkinson has intracellular eosinophilic inclusions composed of alpha-synuclein (lewy bodies). Pick (frontotemporal dementia) silver stationing shows round cytoplasmic inclusions contiatning aggregates of tau protein (Pick bodies) Neuritic plaques and amyloid antipathy is not seen in pseudo dementia Vascular dementia typically present its sudden/stepwise cognitive decline in patients with cardiovascular risk factors and ischemic stroke.

592 Woman's worsening memory problems. PMH significant for longstanding HTN, diabetes and depression. She died two years later from a massive MI. Congo red stationing of brain samples obtained from the hippocampus and cerebral arterioles reveal patchy red deposits that turn yellow-green under polarized light. This patient most likely suffered from which of the following conditions? a. alzheimer b. hunting ton c. parkinson d. pick e. pseudo dementia f. vascular dementia g. B12 def

Patient is under vaccinated and has septic arthritis caused by H. influenzae B (Hib), secondary to hematogenous dissemination following his recent episode of otitis media. H influenza is a small pleomorphic, gram neg coccobacillus that is part of the normal flora of the URT. It is blood loving organism that requires both X factor (hematin) and V factor (NAD) to grow. Because these factors are found within erythrocytes, optimal concentrations are present only in lysed blood agar (chocolate agar). H. influenza can be either encapsulated or unencapsulated (zontypble) w/ encapsulated strains divided into 6 serotypes (a-f) based on the poly saccharide structure of the capsule. Type B capsular material consists of a ribose and ribitol phosphate polymer called polyribitol phosphate (PRP). It is the only serotype that contains pentose monosaccharides rather than hexose sugars as the carbohydrate component of the capsule. The PRP capsule prevents phagocytosis and intracellular killing of neutrophils, allowing the organism to invade the vasculature, persist in the bloodstream, and spread hematogenously to distant sites. Antibodies against the type B capsule provide immunity by promoting opsonization and complement fixation. H. influenza type B used to be a major cause of severe, invasive infections including epiglottis, meningitis and bacteremia. However, since the advent of the conjugate Hib vaccine, most H influenzaeinfections are do non-type B strains that cause noninvasive disease such as sinusitis, bronchitis, otitis media and conjunctivitis. B. there are no strains of H influenza known to produce an exotoxin of any kind. FImbriae are not present in this bacteria. These are proteinaceous projections from bacterial cells mediate attachment to target issues during the process of establishing infection. This mehod of attachment to tart tissues during the process of establishing infection. This method of attachment is used by N. gonorrhoeae, N. meningititidis and E. coli. Hemolysis are not secreted by H influenza Hyaluronidase a enzyme used by bacteria to digest extracellular ground substance and enhance their ability to spread. Hyalurondiase is not produced by H influenza. It is produced by staphylococci, group A strep, and Clostridium difficile most notably.

964 A 3 y.o boy has fever, malaise, and a swollen right knee. He is hypotensive and tachycardic. PMH is significant for a recent episode of otitis media and several vaccinations that are not up to date. Arthrocentesis of his right knee shows cloudy synovial fluid. Grant stain of the aspirate reveals pleomorphic. Gram-negative coccobacilli. Cultures performed a blood agar plate supplemented with a disk containing hematin and NAD grow colonies only near the disk. To pathogenicity of the organism responsible for this patient's condition is most likely related to which of the following virulence factors? a. capsule b. cytotoxic exotoxin c. timbre d. hemolysis e. hyaluronidase

Dude has nephritic syndrome. -linear deposit of Ig along the glomerular basement membrane, a finding characteristic of anti-glomerular basement membrane disease. Deposits are composed of IgG and C3. The target is collagen type IV a component of glomerular basement membrane, leading to subsequent complement deposition. Glomerular crescent composed of proliferating parietal cells with an infiltration of monocytes and macrophages are seen on light microscopy Anti-GBM antibodies may cross react with collagen type IV in pulmonary alveolar basement membrane and cause pulmonary hemorrhage. The combined presentation of renal failure and pulmonary hemorrhage in patients with anti-GBM antibodies is known as good pasture syndrome. Basement membrane splitting is seen on light microscopy using silver or PAS stains in MPGN. The glomeruli of MPGN have a lobular appearance with proliferating mesangial cells and increased mesangial matrix. Immunofluorescence shows granular deposits. FSGS, IgM and C3 deposits may lead to sclerotic areas of the glomeruli on immunofluorescence d/t nonspecific trapping of these components. Normal glomeruli are found on light microscopy in minimal change disease, a condition affecting mostly children that presents as a nephrotic syndrome. These are no deposit on immunofluorescence.

9 A 43 y.o man has shortness of breath and fatigue. He has little energy even to get out of bed. He recently has weight gain and ankle swelling. BP is 162/93. He ha bilateral lower extremity pitting edema that extends up to his knees. Urinalysis reveals 2 protein and elevated WBC and RBC. He also has hemoptysis. What would light microscopy show? a. basement membrane splitting b. capillary wall thickening c. crescent formation d. FSGS e. normal

Carbamazepine is DOC for treating trigeminal neuralgia. It is effective in pain reduction in up to 80% of patients. Like phenytoin, it inhibits neuronal high frequency firing by reducing the ability of Na channels to recover from inactivation. It can cause aplastic anemia, so CBC should be monitored regularly. Carbamazepine is P450 inducer that increases the metabolism of many other medications, thereby decreasing their effectiveness. a/g: baclofen and valproic acid can be used for trigeminal neuralgia, but are not first line choices. Haloperidol is a neruleptic med used for treatment of schizophrenia, acute psychoses, acute mania and Tourette syndrome. It is not not indicated in trigeminal neuralgia. Propranolol has various uses, including migrate prophylaxis and the reduction of portal venous pressure to prevent vatical bleed. But not use for trigem neuralgia. Phenobarbital is effective for the management of generalized tonic-clonic seizures.

Dude has severe, right sided facial pain. Describes that pain as a knife stabbing my face. This usually occurs several seconds after the a meal or teeth brushing. How would you treat? a. baclofen b. haloperidol c. diazepam d. carbamazepine e. propranolol f. phenobarbital g. valproic acid

This patient is experiencing pleuritic chest pain d/t acute lower lobe bacterial pneumonia. Pleuritic chest pain is characterized by sharp, localized, often severe pain that is exacerbated by coughing, breathing or changing position. It can result from any condition that causes inflammation of the pleura (eg, infection, pulmonary embolism, uremia). The pleura is dived into the segments as follows: -Visceral pleura: The visceral (pulmonary) pleura covers all surfaces of the legs, including the surfaces within the pulmonary fissures. The visceral plea doesn't carry pain fibers. -Parietal pleura: Forms the outer boundary of the pleural space and can be subdivided as follows: a. costal pleura: covers the thoracic wall, including the ribs, sternum, intercostall spaces, costal cartilages and sides of the thoracic vertebrae. b. mediastinal pleura: covers the mediastinum c. diaphragmatic pleura: covers the surface of the diaphragm located within the thoracic cavity d. cervical pleura: extends with the apices of the lung into the neck. The phrenic nerve, which is derived from the C3-C5 nerve roots, delivers motor innervation to the diaphragm and carries fibers from the diaphragmatic and mediastinal pleura. Irritation of the pleura in either area will cause a sharp pain worsened by inspiration that will be refereed to the C3-C5 distribution at the bas e of the neck and over the shoulder, Sensory innervation of the remainder of the parietal pleura are accomplish by the intercostal nerves and typically felt closer to the source of the pain. The spinal accessory nerve is motor innervation of the SCM and traps. The long thoracic nerve is the winged scapula causing lesion by the serrates anterior The vague nerve is parasympathetic to the viscera of the best and the foregot.

Patient has productive cough associated with chills and fever. sputum is purulent with faint pink streaks of blood. Sharp pain over the right shoulder and neck area brought on by deep inspiration. Temp is 101 and respirations are 22. PE shows crackles and dullness over the right lower pulmonary lobe. While listening with the stethoscope over the right mid back, the examiner has the patient say the letter "E," and the sound is perceived by the examiner as a loud letter "A." The pain experienced by this patient is most likely carried by which of the following nerves? a. accessory b. intercostal c. long thoracic d. phrenic e. vagus

This patient has McArdle disease (GSD type V). -caused by deficiency of myophosphorylase - an isoenzyme of glycogen phosphorylase present in muscle tissue. Deficiency of this enzyme leads to decreased breakdown of glycogen during exercise, resulting in poor exercise tolerance, muscle cramps and rhabdomyolysis. Prognosis is generally good and symptoms can be improved by consuming simple sugars before beginning physical activity. During glycogenolysis, glycogen phosphorylase shortens glycogen chains by cleaving alpha-1,4-glycosidic linkages between glucose residues, liberating glucose 1-phospahte in the process. This occurs until 4 residues remain before a branching point (the limit dextrin). At this point, the debranching enzyme performs 2 enzymatic functions: 1. Glucosyltransferase cleaves the 3 outer glucose residue of the 4 that are left by glycogen phosphorylase and transfer them to a nearby branch 2. the enzyme alpha-1,6-glucosidase removes the single remaining branch residue, producing a free glucose and a linear glycogen chain that ca be further shortened by glycogen phosphorylase.

1032 A 15 y.o boys is easily fatigue during exercise. After several repetition on the bench his arms feel like jelly. He also has severe muscle cramping and urine discoloration after periods of intense straining. Further evaluation reveals that his exercise toelrcn can be greatly improved by drinking an oral glucose solution before beginning a strenuous activity. This patient is most likely deficient in what enzyme?

Chediak-higashi syndrome is an AR inherited disease that can affect the CNS, skin color and immune system. Neurologic defects associated with this condition include: nystagmus, peripheral and cranial neuropathies. Immunodeficiency in this syndrome results from a defect in neutrophil phagosome lysosome fusion. This causes abnormal giant lysosomal inclusion that are visible on light microscopy of a peripheral blood smear. The immunodeficiency leads to recurrent pyogenic infections most commonly caused by Staph and Strep. Abnormal melanin storage in melanocytes causes partial oculocutaneous albinism. a. neutrophils in patients with CGD are unable to kill catalase-producing organisms, d/t the deficiency of NADPH oxidase. Phenylketonuria is caused by def of phenylalanine hydroxyls. Impaired conversion of phenylalanine to tyrosine leads to CNS abnormalities. The patients have features of albinism and musty body odor. DiGeroge syndrome is an immunodeficiency resulting from a delation on chromosome 22 leading to maldevelopment of the 3rd and 4th pharyngeal pouches. It is characterized by thyme and parathyroid phypoplasia, abnormal facies and cardiac defects. Thyme hypoplasia causes T cell defect that manifests with recurrent viral, fungal and protozoal infections. Wiskott-Aldrich syndrome is an X-lniked disorder characterized by immunodeficiency, eczema and thrombocytopenia. The immunodeficiency present in Wiskott-Aldrich syndrome is a combined B-lymphocyte and T-lymphocyte disorder Albinism is a genodermatosis that results from defects in the production of melanin by melanocytes. In this disease melanocytes are normal in number location; it is the production of the pigment itself that is defective. Complete albinisms is manifest by white hair, blue eyes and pink or white skin and results from a complete absence of tyrosinase.

1132 A 4 y.o white kid is being evaluated for recurring skin and respiratory infections. He has light skin and silvery hair. Horizontal nystagmus is present on eye examination. PB shows giant cytoplasmic granules in neutrophils and monocytes. The patient most likely suffers from which of the following conditions? a. chronic granulomatous disease b. phenylketonuria c. disgorge syndrome d. wiskott aldrich synod e. chediak higashi syndrome f. albinism

Patient has dysentery by Shigella. The bacteria is transmitted via fecal oral and is never a component of the intestinal flora. People who are at risk are children, men who have sex with men and adults in skilled nursing facilities. It is non lactose fermenting organism that produce acid (not gas) during glucose fermentation (in contrast to E.coli). Furthermore, Shigella species are non-motile and do no produce H2S (In contrast to Salmonella) Mucosal invasion is the essential pathogenic mechanism for infection. It infiltrates the M cells that overlie Peyer's patches. After cell entry, it is able to lyse its containment vacuole and enter the cytosolic compartment. It then can induce apoptosis of the host cell and spread to adjacent cells via protrusions created during shot cell actin polymerization. Shigella invasion triggers a robust host inflammatory response that is largely mediated by neutrophils. Some stars have shiga toxin that messes with the 60S ribosomal subunit, halting cellular protein synthesis. However, shiga toxin plays a minor, nonessential role in the disease process as notoxigenic strains also cause shigellosis. Intestinal colonization is not necessary as transmission of as few as 10-100 organism is sufficient to cause disease. If Shigella is slated from a stool culture, an active infection is taking place. Proliferation in lymph nodes is more characteritiscs of Salmonella type and Yersinia entercoliticia (grows at low temp like Listeria) can gain access to lymphatics and proliferate in the mesenteric lymph nodes. Y enterocolitica can cause inflammation and enlargement of the lymphoid tissue around the appendix an detrital ileum ("pdseudoappedicitis"), leading to right lower quadrant pain that can be confused with acute appendicitis. Shigella infections rarely case bacteremia, as the bacterium is readily phagocytose and destroyed after entering the blood stream. Enteric bacteremia is more likely to be cased by E coli, S phi (typhoid fever), Klebsiella and Proteus

1135 Kid has bloody diarrhea. Stool studies shows fecal leukocytes and occult blood. fecal cultures grow non-lactose fermenting, gram neg rods of MacCOnkey agar. the bacteria ferment glucose without gas production, do not generate hydrogen sulfide when grown on triple sugar iron agar, and cannot replicas at refrigeration Temp. Which of the following bacterial factor sis the most important during the pathogenesis of this patient's disease? a. exotoxin production b. intestinal colonization c. mucosal invasion d. proliferation in lymph nodes e. survival in the circulation

Shigella can cause disease with as little as 10 organisms. They are able to invade tissue and is highly adapted to surviving the acidity of the stomach and the bacteriostatic action of bile. Incubation = 1-3 days. Disease begins with watery diarrhea nd progresses to fever, abdominal pain, dysentery and tenesmus (painful rectal spasm associated with the urge to defecate but little passage of stool) A much large inoculum of Slamonella (approximately 10^7) is required for successful infection in a suscepti bye host. other organisms that can cause diarrhea with only a small inoculum include -Campylobacter jejuni (500) -Entamoeba histolytic (1-10) -Giardia lambda (1-10)

1136 Volunteer studies are used to determine the infectious dose of Salmonella required to cause gastro enteritis. On a graph measuring the % of people infected and number of bacteria there are two curves. Curve one requires less than 100 to cause infection. Curve 2 is represents salmonella which requires over 100,000 to cause infection. Which of the following bacteria represents line 1? a. clostridium perfringens b. enterotoxigenic E. coli c. Shigella flexneri d. V. cholera e. V. parahaemolyticus

Pentad of TTP-HUS (thrombocytopenia thrombotic purpura-hemolytic uremic syndrome): 1. fever 2. neurologic symptoms 3. renal failure 4. anemia 5. thrombocytopenia These share common clinical pathologic features including: -platelet activation in arterioles and capillaries -diffuse microvascular thrombosis (most commonly affecting the brain, kidneys and heart) -microangiopathic hemolytic anemia with schistocytes -thrombocytopenia Unlike DIC, in which coagulate cascade activation leads to prolongation in coagulation studies (PT and activatedPTT), TTP is almost always characterized by normal PT and PTT. a. focal segmental glomerulosclerosis, including its collapsing variant, commonly manifests as heavy proteinuria. b. crescentic or rapidly progressive glomerulonephritis typically presents with macroscopic hematuria, HTN and progressive renal failure. It is classified as anti-glomerular basement membrane (with hemoptysis in Goodpasture syndrome), immune complex mediated (eg, systemic lupus erythematous), or pace-immune (with pulmonary, upper respiratory and kidney involvement in granulomatosis with polyangiitis). c. PSGN is typically in childhood that can follow strep pharyngitis and lead to increased coke colored urine output and periorbital edema d. hence-schonlein purport is typical a childhood disease with nonthrombocytopenic palpable purport and arthritis; IgA nephropathy commonly presents with recurrent hematuria and low grade proteinuria following an upper respiratory tract infection. Both diseases have similar histopathologic findings with IgA deposition in the mesangium. Acute tubular necrosis d/t ischemia (eg, prolonged hypotension), nephrotoxins (eg, antibiotics), or pigment deposition (eg, myoglobinuria) generally presents with rising creatinine and muddy brown granular casts on urinalysis.

11608 A 36 y.o woman has a 4 day history of intermitted fever, abdominal pain and vomiting. Her urine output has decreased associated with skin rash and progressive lethargy. There is catered petechial rash, facial puffiness and 1+ bilateral pedal edema on PE. Labs shows low hemoglobin, elevated reticulocyte count and thrombocytopenia. BT is prolonged, PT AND PTT are normal. The PB smear shows schistocytes and reduced platelets with presence of giant forms. BUN is 56 and serum creatinine is 2.3. Urinalysis is positive for proteinuria and hematuria. Which of the following is most likely to be seen in renal biopsy? a. collapse and sclerosis of glomerular tufts b. crescent shaped mass of cellular proliferation and leukocytes c. diffuse proliferation and sub epithelial immunoglobulin deposits d. mesangial IgA deposition and proliferation e. patchy necrosis of tubular epithelium and loss of basement membrane f. platelet rich thrombi and glomeruli and arterioles.

Most available antidepressant medications affect serotonin or both serotonin and NE at the synapse. SSRIs are used as first line antidepressants that work by blocking the 5HT transporter. This prevents the normal reuptake of serotonin in the presynaptic neuron, resulting in increased availability of serotonin in the synaptic space. The receptor activities are the primary MOA of antipsychotics used mainly to treat schizophrenia and other psychotic disorders and to augment antidepressants int the treatment of mood disorders. Potent antagonism of D2 is the mech of action of first generation antipsychotics such as haloperidol. Second-generation antipsychotics have comparatively less affinity for D2 and additional property of 5HT2 receptor antagonism, which underlies their lower risk of extrapyramidal side effects. An antipsychotic would be appropriate monotherapy for MDD without psychotic feature. Block of Na, resulting in stabilization of neuronal membranes, is the MOA of some anti epileptic drugs (eg, carbamazepine). These drugs also have a role as a mood stabilizers in bipolar disorder but are not a primary treatment of unipolar MDD Inhibition of monoamine oxidase is used to treat resistant to SSRI depression. These drugs are not used as first line therapy d/t their dietary restriction and risk of serious adverse events (eg, HTN crisis, serotonin syndrome) Benz work to potentiate the effect of endogenous GABA, the major inhibitory neurotransmitter in tehCNS, by facilitating the increase frequency of chloride channel opening in the GABA A receptor. They have sedative hypnotic anxiolytic anticonvulsant and muscle relaxant properties but it is not first line treatment for depression

11854 Dude has MDD, A drug with which of the following primary mechanisms of action is most appropriate? a. antagonism of D2 b. antagonism of 5HT2 c. blockade of Na d. blockade of serotonin transporter e. inhibitor of monoamine oxidase f. potentiation of the effects of endogenous GABA

Cholesteatomas are collections of squamous cell debris that form a round, pearly mass behind the tympanic membrane in the middle ear. They can occur congenitally or may develop in adults as either an acquired primary lesion or secondary to infection, trauma, or surgery of the middle ear. Primary cholesteatomas are a result of chronic negative pressure in the middle ear causing retraction pockets in the tympanic membrane that become cystic; as the squamous cell debris accumulates, a cholesteatoma is formed. Secondary cholesteatomas occur after squamous epithelium migrates to or is implanted in the middle ear ("skin in the wrong place"). Cholesteatomas most commonly cause painless otrrhea. They also can produce lytic enzyme and are often discovered when they erode through the auditory ossicles, causing conductive hearing loss. If a mass grows sufficiently large, it can erode into the vestibular apparatus or facial nerve, causing vertigo or facial palsies. Cholesterol granulomas can form in the middle ear after hemorrhage but are much less common than cholesteatomas. They appear as bluish-black gelatinous material behind the tympanic membrane. Despite what the name implies, cholesteatomas do not contain any lipid or cholesterol components. A facial nerve neuroma can grow in the middle ear as the facial nerve courses through this territory. However, it would present with unilateral facial paralysis. Squamous cell carcinoma is the most common malignant tumor of the ar canal. It typically appears as ulcerated plaque or nodule. The most common symptom is local or regional pain. Granulomatous disease of the ear can occur but is uncommon and usually develops in conjunction with a systemic disease, such as sarcoidosis, granulomatosis with polyangiitis, or Langerhans cell histiocytosis.

11628 A 48 y.o woman has intermittent ear discharge over the last 2 years. She has decreased hearing in the right ear recently. PMH is significant for obesity, hyperlipidemia, seasonal allergies, and diet controlled DM. Otoscope shows a small perforation in the right tympanic membrane and a pearly mass behind the membrane. Conduction hearing loss is noted in the right ear. Which of the following is the most likely cause of this patient's mass? a. cholesterol and lipid accumulation b. facial nerve neuroma c. malignant squamous cell neoplasm d. noncaseating granuloma e. squamous cell debris

Initial treatment you would see rapid onset and improvement of core ADHD symptoms. The most common adverse effects are: 1. decreased appetite 2. weight loss 3. insomnia. Most affected children will experience a mild decrease in appetite that can typically be managed by administering the medication after meals and encouraging the child to eat nutrient dense foods. Other psychostimulatne adverse facts that are less common include tics and increase HR and BP. During treatment, patents are typically seen mostly to monitor weight, high, HR and BP. Increase app appetite and weight gain are adverse effects commonly associated with second-generation antipsychotics; these drugs do not have a role in treatment of ADHD The risk of steven johsnon syndrome is associated with lamotrigine, an anticonvulsant used to treat seizure and bipolar disorder. Polyuria an polydipsia are symptoms of nephrogenic diabetes insidious, a potential adverse effect of lithium. Lithium is a mood stabilizer used in treatment of bipolar disorder and is not effective for ADHD.

11809 An 8 y.o boy has ADHD. He is given methylphenidate. Which of the following is a side effect? a. decreased appetite and weight loss b. delayed onset of action c. increased appetite and weight gain d. initial worsening of symptoms e. life threatening rash f. polyuria and polydipsia

Dude has spinal foraminal stenosis causing L5 radiculopathy. Nerve roots in the lumbosacral spine exit below their corresponding vertebral body levels; therefore, a nerve transversing the L5-S11 neural foramen represents the L5 nerve root. Disc herniation at the L5-S1 level classically affects the S1 nerve root, resulting in S1 radiculopathy as opposed to this patient's L5 radiculopathy. L5 radiculopathy clinical presentation: -sensory loss: over the buttocks, lateral thigh, calf and dorsal foot. -motor: weakness on foot dorsiflexion, inversion, eversion and toe extension. a/d: Achilles reflex is mediated by S1 which exits from S1-S2 disk space S1 spinal nerve root also provides motoring innervation to the gluteus max that allow hip extension. c/e: L4 exits between L4-L5 disk space and provides sensory innervation to the medial calf. The L4 spinal nerve root also innervates the quadriceps muscles, which mediates knee extension and patellar reflex

11834 Old dude has back pain. Imaging studies reveals large osteophyte compressing the nerve traversing the right L5-S1 neural foramen. Which of the following is most likely to be seen on right lower extremity examination in this patient? a. loss of ankle reflex b. loss of sensation over the dorsum of the foot c. loss of sensation over the medial calf d. weakness of the gluteus maximus muscle e. weakness of the quadriceps

Risk is the probability of developing a disease or other health outcome over the study period. IN this example, it represents the probability of developing Alzheimer disease over a 20-year period among middle aged men who have a first degree relative with Alzheimer. To calculate the probability in subjects with low carotene levels, divide the numberr of subjects who develop Alzheimer disease in the ow carotene group (20) by the overall number of subjects with low carotene group. So.. 20/(20+20) = 20/40 = 1/2 = 0.50, meaning that amount subject with low carotene levels, there is a 50% probability of developing Alzheimer disease in 20 years. Note this is not relative risk (RR) which is = risk of developing Alzheimer disease among subjects with low carotene levels/ and risk of developing alzheimer disease among subject with normal cortege levels which = .5/(40/160) = .5/.25 = 2. The prevalence of low carotene (exposure) among subjects who do not develop Alzheimer disease is 20/140= 14%. The prevalence of low carotene (exposure) among subjects who develop Alzheimer disease is 20/(60) = 33%. The prevalence of low carotene (exposure) in the entire cohort is (20+20)/200 = 20% As noted in the RR calculation above, the 20-year risk developing Alzheimer disease among subjects with normal carotene levels is 40/160 = 25%

1185 Middle-aged men with first degree relative with Alzheimer disease have their blood carotene concentration measured. they are followed over 20 years for development of Alzheimer disease the results are: this who developed -Those who have Alzheimer disease are 60 total, where 20 of whom have low carotene levels and 40 of them have normal carotene lvl. -Those who do not have Alzheimer disease are 140, where 20 have low Carotene level and 120 have normal carotene level. What is the 20 year risk of developing Alzheimer disease in subject with low carotene level?

Patient has a history of paroxysmal atrial fibrillation and ischemic stroke off anticoagulation is likely suffering from a recurrent embolic stroke complicated by seizure. Stroke is one of the the most common causes of seizure, which occurs d/t to the release of excitotoxic neurotransmitters in the ischemic brain region. The clinical manifestations of partial (focal) seizure depend on the anatomic brain region in which the seizure activity occurs. This patient's intimal right arm numbness/paresthesias were likely caused by a simple partial seizure originating in the left primary somatosensory cortex (post central gyrus). This brian region is responsible for processing all somatic sensory modaiteis (eg, touch, temp/pain, vibration/proprioception) of the contralateral body. The patient subsequently develop right sided convulsion d/t spread of the seizure along the lateral brain convexity to the primary motor strip (pre central gyrus). The seizure then became seoncdarily generalized to involve both cerebral hemispheres, causing tonic clonic convulsions. The frontal eye field is the region of the cerebral cortex that controls horizontal eye movement. Activation of the left frontal eye field during seizure would cause the eyes to deviate horizontally to the right. After the seizure, the patient's eyes may temporarily deviated to the left d/t postictal neuroinhibztion. Broca is located in the posterior inferior frontal gyrus of the dominant (usually left) hemisphere and is responsible for controlling expressive speech/language. A seizure originating from Broca area would likely result in octal speech and vocalizations. Sensory speech (Wernicke) area is located in the posterior superior temporal gyrus of the dominant hemisphere and is responsible for language comprehension. A seizure originating from the temporal lobe of the cerebral cortex would likely result in auditory hallucinations. The primary visual cortex in the occipital lobe is responsible for processing visual information from the contralateral homonymous visual fields. A seizure originating from the left primary visual cortex would likely result in visual hallucinations affecting the right homonymous visual fields.

12008 A 55 y.o man has right arm tingling and numbness for the past several hours. No headache, muscle weakness, slurred speech, or difficulty ambulating. PMH significant for HTN, paroxysmal atrial fibrillation and ischemic stroke without residual symptoms. His anticoagulation was discontinued 2 months ago after he developed a GIT hemorrhage. BP is 150/90. He then develops right arm paresthesias that spreads through the entire right side of the body and started to develop right sided convulsions followed by generalized tonic clonic seizure. Which of the following structures is most likely the origin of this patient's seizures? a. frontal eye field b. Broca area c. pre central gyrus d. post central gyrus e. wernicke area f. occipital lobe

anxiety and insomnia are nonspecific symptoms that can be d/t a bunch of shit. Diagnosis of primary anxiety disorders requires ruling out medical and substance induced etiologies (d/t medication, drugs of abuse, or toxins). Chronic anxiety and insomnia ( as seen in GAD) and panic attacks can be be symptoms of a medical illness or substance intoxication or withdrawal. The ne onset of anxiety in this patient with no prior psychiatric history and prominent PE suggest that anxiety is more likely d/t a medical conditions (i.e., the anxiety is a direct physiological effect of the medical condition) than to a primary anxiety disorder Specifically, this patient's weight loss despite normal appetite; tachycardia; warm, most skin; tremors; frightened start; and restlessness are consistent with hyperthyroidism. Other medical conditions known to present with anxiety symptoms include hypoglycemia, pheochromocytoma, hypercortisolism and cardiac arrhythmia. Common substance induced etiologies include sympathomimetic drugs, caffeine, stimulate intoxication and sedative hypnotic withdrawal. GAD and panic disorder characterized by excessive, uncontrollable worry about multiple issue lasting at least 6 months.

11866 A 40 y.o woman comes to the office d/t persistent anxiety and insomnia over the past 6 months. She says, "I'm really worried that something is wrong with me. I was never a particularly anxious person, but now I feel anxious all the time. Sometimes I feel panicky for no reason and have episodes in which my races and I break out in sweat. The only benefit is that I have lost 5 points without even trying." The patient has no significant medical or psychiatric history. She takes no medication other than an occasional tablet of alprazolam prescribed for insomnia by a previous physician. the patient says, "it helps my anxiety a little but wears off quickly." She drinks 2-3 glasses of wine per week. She doesn't smoke or use illicit drugs. BP is 130/90 and HR is 106. On PE, the patient is relies and has warm, moist skin and mild hand tremor bilaterally. Mental status examination is notable for frightened appearance, anxious mood, and rapid speech. which of the following is the most likely diagnosis? a. anxiety d/t a general medical condition b. bento withdrawal c. GAD d. illness anxiety disorder e. panic disorder f. somatic symptom disorder

Candida gives rise to true hyphae, termed "germ tubes," when incubated at 37 for 3 hours. These things are specific for candida albicans that are not seen with any other candida species. all candida are yeast seen in tissue sections as single cells with pseudohyphae. These guys are the most common opportunistic mycosis. It is also a frequent colonizer of human skin and mucous membrane. Superficial candida infections are associated with antibiotic use, corticosteroid use, DM, HIV and other immunpsuppressing illnesses. these superficial infections include oral thus, vulvovaginitis, and cutaneous candidiasis. Disseminated candidiasis occur in neutropenic patients and most often affects the esophagus, heart, liver and kidney. They don't usually cause lung disease. This patient's pulmonary symptoms might be d/t tuberculosis. He has TB, the candida is just an incidental finding and doesn't contribute to the diagnosis. Aspergillum fumigates is a fungus that colonizes already existing lung cavities. it forms a fungus ball that may cause cough and hemoptysis. many pathogens including bacteria, viruses and fungi can cause inflammation of the lung parenchyma. Candida associated pulmonary disease is very rare. The trachea and large and small bronchi are normally sterile.

119 A 34 y.o immigrant has 3 months history of productive cough, night sweats and low grade fever. Sputum cultures grow budding yeast that form germ tubes at 37 degrees C. The most likely site of this organism before entering the sputum is which of the following? a. fibrous lung cavities b. inflamed lung parenchyma c. bronchioles and small bronchi d. large bronchi and trachea e. oral cavity

The ovaries suspended posterolaterally to the uterus by the utero-ovarian ligament and receives blood supply through the vessels that travers the infundibulopelvic (IP) ligament (eg, suspensory ligament of the ovary). The IP ligament also houses the ovarian nerve plus. Rotation of the ovary around the IP ligament results in ovarian torsion. The main risk factor for torsion is the presence of a large ovarian mass (eg, cyst, neoplasm). the weight of mass causes the ovary to twist and occlude the ovarian vessels and nerves. When blood flow to the ovary is completely blocked, the tossed ovary becomes edematous and ischemic. A patient with ovarian torsion will typically present with sudden onset of unilateral pelvic pain and nausea and sometimes vomiting and fever. Pelvic ultrasound is the first line diagnostic test and may show decreased or no blood flow to the ovaries. a/c: the mesosalpinx is the portion of the board ligament that connects the fallopian tubes to the pelvic sidewall. The broad ligament is a wide sheet of peritoneal tissues that encapsulates the reproductive organs anteriorly and posteriorly and attaches the uterus to the pelvic sidewalls bilaterally, making torsion of the ovary around the ligament (including mesosalpinx) anatomically impossible. d. the round ligament maintains the ante flexion of the uterus. It runs from the lower aspect of the uterus through the inguinal ring to the labia major and is not connected to the ovaries. During pregnancy, a woman may experience intermittent sharp pain d/t irritation from sudden movement of the stretching round ligament. e. the uterosacral ligaments connect the posterior aspect of the uterus to the anterior portion of the sacrum. these ligaments hold the uterus in an anteverted or retroverted position; loss of this support contributes to uterine prolapse into the vagina. they do not involved in the position or vascular supply of the ovaries.

11901 A 29 y.o nulliparous woman comes to the emergency with right sided pelvic pain The pain is sharp and came on suddenly while she was doing some yard work. It is accompanied by nausea and is worse with movement. Temp is 100, BP 100/60, and HR 92. Pelvic US reveals a normal-sized uterus and left ovary, and a right adnexal mass measuring 6 cm with no blood flow to the ovary. Urine B-hCG is negative. The patient's condition most likely involves pathology in which of the following structures? a. broad ligament b. infundibulopelvic ligament c. mesosalpinx d. round ligament e. uterosacral ligament

Dude has trigeminal neuralgia (tic douloureux), a neuropathic disorder the typically presents with episodic, severe, unilateral, electric shock like pain in the distribution of CN V. Symptoms are often triggered by innocuous stimuli such as shaving or washing the face First line treatment: carbamazepine - a neuroleptic medication that inhibit neuronal high-frequency firing by reducing the ability of Na channels to recover from inactivation. Adverse effects are: 1. bone marrow suppression ,which may lead to anemia, agranulocytosis, and thrombocytopenia (thus CBC should be monitored periodically) 2. SIADH Ketamine, felbamate and memantine are examples of medications that block NMDA receptors. Ketamine is used as a general anesthetic agent, felbamate is anticonvulsant and memantine may be effective in treatment of Alzheimer dementia. SSRI (eg, fluoxetine, sertraline) are typically used to treat depression, anxiety and PTSD. Side effects may include headache, wroesenign anxiety/agitaiton, and insomnia Ethosuximide blocks T-type Ca channels and decreases calcium current in thalamic neuron. This anticonvulsant is the agent choice for absence seizures. Major side effects include GI symptoms and drowsiness Various anti seizure medications (eg, bento and barbiturates) act on GABA-A receptor to increase chloride conductance; however, carbamazepine doesn't significantly affect chloride channels. Donepezil is an AChesterase inhibit that can be used to treat Alzheimer dementia. Side effects are generally related to increased cholinergic activity, which includes GIT symptoms and symptomatic bradycardia.

1199 A 64 y.o man has 2 months of episodic facial pain. It is sudden onset, severe and electric shock like pain over the right cheek and ear lasting several seconds. The patient reports that activities like shaving and washing his face can trigger the pain and it is becoming more frequent. He is a retired veteran and has history of PTSD. Labs are normal. He is started on a single drug therapy. On follow-up visits 3 months later his CBC shows low hemoglobin, low platelets and low leukocyte. What is the MOA of the drug he is taking? a. blocks NMDA in hippocampal neurons b. blocks 5-HT reuptake on multiple levels. c. decreases calcium current in thalamic neurons d decreases Na current on multiple levels e. increases Cl current on multiple levels f. inhibits synaptic ACh in cortical neurons

Train of four (TOF) stimulation is used during anesthesia to assess the degree of paralysis induced by NMJ blocking agent. A peripheral nerve is stimulated 4 times in quick succession and the muscular response is recorded. the heigh of each bar represent the strength of each with; higher bars indicate the activation of increasing number of individual muscle fibers (myocytes). When a non depolarizing NMJ blocker (eg, vecuronium) is administered, competitive inhibition of postsynaptic ACh receptors at the motor endplate prevents some of these fibers from activating, decreasing the strength of the twitch. TOF stimulation shows a progressive reduction in each of the 4 responses (fading pattern) as a result of less acetylcholine being released with each subsequent impulse (d/t the additional affect of presynaptic acetylcholine receptor blockade) In contrast, depolarizing blockers (eg, succinylcholine) initially function by preventing depolarization of the motor endplate and show equal reduction of all 4 twitches during TOF stimulation (phase 1 blockade). The responses remain equal because the presynaptic ACh receptor stimulation helps to mobilize presynaptic ACh vesicles for release. Persistent exposure to succinylcholine response in eventual transition to phase II blockade as the ACh receptors become desensitized and inactivated (i.e., functionally similar to non depolarizing blockade). Succinylcholine is commonly administered for reaped-sequence intubation d/t its rapid onset (<1 minute). The duration of action is determined by its metabolism by plasma cholinesterase and is typically <10 mins. However, some patients are homozygous for an atypical plasma cholinesterase, which breaks down succinylcholine more slowly. In these patients, the paralysis can last four hours and they must be painted on mechanical ventilation until spontaneous respiration resumes. a. Dantrolene relaxes skeletal muscle by reducing the release of Ca from the ssarcoplasmsc reticulum. Dnatrolene is used to treat malignant hyperthermia and neuroleptic malignant syndrome. It is not routinely used as an NMJ blocking agent. b. bentos are effective for station, but they have no direct action at the NMJ and do not provide sufficient muscle paralysis to facilitate intubation. c/e. Pancuronium and tubocurarine are non depolarizing NMJ blockers. Unlike depolarizing nmj drugs, these agents do not function in distinct phases and their TOF responses always display a fading pattern. Neostigmine administration reverses non depolarizing NMJ blockade.

1212 Dude has COPD and develops hypercapnia respiratory failure. They gave him drug X and an appropriate sedative as premedication in order for him to be intubated on a mechanical ventilator; however he remains apnea longer than expected. Anesthesiology is consulted, and the patient's NMJ blocker is assessed by electrically stimuatling a peripheral nerve 4 times quick succession and observing the muscular responses reveal a sustained initial response followed by a decreased in response 30 mins later. What is drug X? a. dantrolene b. midazolam c. pancuronium d. succinylcholine e. tubocurarine

Not all interventions end up helping patients. the number needed to harm (NNH) represents the number of people who must be treated before an adverse event occurs. It is calculate din a manner similar to the number needed to treat (NNT) but using the absolute risk increase (sometimes referred to as the attributable risk) instead of the absolute risk reduction: NNH = 1/attributable risk To determine the attributable risk, first calculate the adverse event rates in the treatment and placebo groups. In this case, the adverse event of interest is death at 3 month. There were 80 total people treated with drug X; of those, 60 were dead at 3 months. Similarly, there were 76 total people treated with placebo; of those, 38 were dead at 3 months. -adverse event rate in treatment group = 60/80 = .75 -adverse event rate in the placebo group = 38/76 = .50 The attributable risk can be calculated by subtracting the adverse event rate in the placebo group from the adverse event rate in the treatment group: .75 - .50 = .25, thus the absolute risk increase attributable to drug X is 25%. NNH = 1/.25 = 5, indicating that for every 4 patients treated with drug X, 1 will experience an adverse event (ion this case death).

1231 Drug X is being tested for treatment of stage IV solid tumor malignancies. Part of the drug company's evaluation process is to analyze survival data after 3 months of treatment. Results are given in a table: -Alive at 3 months treated with drug X = 20 -Alive at 3 months treated with placebo = 38 -Dead at 3 months treated with drug X = 60 -Dead at 3 months treated with placebo = 38 Which of the following best represents the number needed to harm for drug X? a. 2 b. 3 c. 4 d. 7 e. 9

Features of drugged induced LE C/Ps: abrupt onset symptoms 9fever/fatigue, arthralgia/arthritis, rash, serosistis). Predilection for *slow acetylators* Labs: anti-histone antibodies (95%) and anti-dsDNA antibodies (rarely seen more specific for SLE) Implicated drugs: *procainamdie, hydrazine, isonizadi,*minocylcine, TNF-alpha inhibitors (eg, etanercept) The drugs above are metabolized via phase II acetylation in the liver. hepatic expression on N-acetyltransferase is genetically determined, and patients with a slow acetylator phenotype are greater risk for developing DILE. Patients who are slow aceytlators are also predisposed to isoniazid induced peripheral neuropathy d/t increased drug concentrations. Liver hydrolysis is a phase I reaction in which a compound is clveaed by adding water (eg, esterase or amides enzymes). Phase I metabolism usually priced phase II conjugation metabolism Liver hydroxylation is a phase I oxidation transformation catalyzed by P450. Addition of the hydroxyl group reduces lipid solidity and facilitates excretion. Liver sulfate conjugation is a phase II metabolic pathway that biotransform drugs into more polar compounds that are more water soluble easily excreted certain drugs are hydrolyzed by plasma esterase, which rapidly cleave ester links after the drug enters the circulation. This rapid inactivation allows for short duration of these medications. many drugs used in anesthesia (eg, succinylcholine,teraciane, remifentanil) are metabolized by plasma hydrolysis. A few drugs are not metabolized in the body. They are excreted in the urine mostly unchanged and are still pharmacologically active.

1253 A 55 y.o nurse was started on multi drug therapy for TB. He comes to his pCP d/t fatigue and low grade fever for several days. He also has had muscle aches and severe joint pain in his elbows, wrists, and knees. CBC shows: low hemoglobin and platelets but high leukocyte BUN is high and creatinine is high. antihistone antibody and anti nuclear antibody are positive. an abnormality in which of the following metabolic processes most likely underlies this patients' current condition? a. liver acetylation b. liver hydrolysis c. liver hydroxylation d. liver sulfate conjugation e. plasma hydrolysis f. unchanged excretion

GIT ulcers are defined as breaches of alimentary tract mucosa that extend through the muscular mucosal into the submucosa (or beyond). Those specifically designated as peptic ulcers are chronic lesions in areas exposed to acid/peptic juices, most commonly in the proximal duodenum, astral stomach and dastroesophageal junction. Patients may present with epigastric discomfort, early satiety, and a positive fecal occult blood test. Almost all peptic ulcers are d/t either Helicobacter pylori infection or use of NSAIDs. The vast majority of ulcers occur within the first part of the duodenum; however, multiple or refractory ulcers beyond the duodenal bulb may be seen in patients with gastronome (non-beta islet cell tumor of the pancreas) Duodenal ulcers are very rarely malignant and do not require biopsy. Conversely, gastric ulcers can be malignant (eg, mucosa-associated lymphoid tissue lymphoma, gastric adenocarcinoma) and must be biopsied for confirmation. Gastric ulcers caused by H pylori infection are associated with an increased risk of gastric cancer, but duodenal ulcers related to this infection do not lead to a higher risk of duodenal carcinoma. Esophageal adenocarcinoma can appear as an ulcerated/exophytic lesion in the lower one third of the esophagus and typically results from Barrett's esophagus, chronic GIT reflux, obesity and/or smoking. Esophageal squamous cell carcinoma appear as plaque like thickening of the mucosa in the upper 2/e of the esophagus that may become ulcerated. Major risk factors include smoking, alcohol consumption and caustic injury. cd: carcinomas of the distal colon tend to be annular lesions that results in "napkin-ring" contraction of the bowel, with heaped-up edges and and ulcerated central region. Risk factors include advanced age, hereditary (eg, FAP) or sporadic colon cancer in the family , and inflammatory bowel disease.

130 Which of the following part of the GIT are ulcers most likely to be non-malignant? a. duodenum b. esophagus c. rectum d. sigmoid colon e. stomach

SSRIs and SNRIs take about 4-6 weeks for response, and their initial activating effects can lead to increased agitation and anxiety during this period. Thus a temporary course of benzodiazepines is used during the time period for SSRI and SNRI to take place. Choice of bento is often based on 2 pharmacokinetic factors: 1. half life - which is related to the desired length of clonal effect 2. hepatic metabolism This patients want to avoid prolonged undesired side effects because of work. It is best to give her a short or intermediate half life drug, while avoid long acting bentos such as chlordiazepoxide, diazepam and flurazepam. Lorazepam, oxazepam and temazepam (mnemonic LOT) are preferred in patients with impaired hepatic metabolism. They undergo glucoridniation directly and are not dependent on hepatic enzymes for their metabolism In clinical practice, lorazepam is commonly used at low doses 2 or 3 times a day as needed for short term anxiety a,c,g: alprazolam and clonazepam have intermediate half life, and triazolam has a short half-life. However these bentos undergo hepatic metabolism and would not be preferred in this patient with hepatic impairment

1352 Lady is GAD. She would like to take a type of benzo that she can have whenever she feels like she needed without prolonged or excessive daytime fatigue and impaired judgment at work. Her PMH shows elevated haptic transaminases. Which of the following is a good for her? a. alprazolam b. chlordiazepoxide c. clonazepam d. diazepam e. flurazepam f. lorazepam g. triazolam

Ammonia is generated from metabolism of amino acids and is converted to urea by the urea cycle. The combination of bicarb, ammonia and ATP is catalyzed by carbamoyl phosphate synthetase (rate limiting enzyme in the urea cycle). Carbamoyl phosphate combines with ornithine to form citrulline in a reaction catalyzed by ornithine transcarbamylase. Citrulline enters the cytosol and is converted to arginosuccinate, and then arginine. The conversion of arginine to ornithine by arginase completes the urea cycle by releasing a urea molecule. N-acteylglutamate serves a regulator of the urea cycle through citation of carbamoyl phosphate syntehatase I. This patient has OTC def, the most common urea cycle disorder. Def results in excess carbamoyl phosphate, which stimulates pyrimidine synthesis. As an intermediate product in the this pathway, orotic acid accumulates and results in increased urinary orotic acid. Patients also have hyper ammonia d/t impaired ammonia excretion, chi is a metabolic emergency. Ammonia is neurotoxic and causes episodes of vomiting and confusion/coma. Tachypnea also occur d/t cerebral edema from ammonia buildup, resulting in central hyperventilation and respiratory alkalosis. Metabolic decompensation is often triggered by illness (eg, viral upper resp. infection, acute otitis media), fasting or increased protein intake. a/c: defects in carbamoyl phosphate synthetase I and N-acetylglutamate synthetase also result in hyperammonemia, levels of carbamoyl phosphate are low and orotic acid is not elevated in the urine. Hypoxanthine-guanine phosphoribosyltransferase def (Lesch-Nyhan syndrome) results in hyperricemia because purines cannot be salvaged from degraded DNA. Rate kidney stones and self mutilation are the classic clinical manifestations. Uridine monophosphate synthetase (UMPS) is part of the pyrimidine synthesis pathway. UMPS def leads to orotic acid buildup (similar to OTC def) but not hyperammonemia. Characteristics influence megaloblastic anemia and delayed growth.

1370 A 2 y.o boy has fever, vomiting and sleepiness. The boy has had mild rhinorrhea and fever for the past 3 days. Since birth, he has had multiple illnesses characterized by vomiting and sleepiness. Prior lab testing revealed increased blood NH3 levels during these episodes and markedly increased orotic acid excretion int eh urine. PE shows a tachypneic boy who is unresponsive to all stimuli. What enzyme is deficient? a. carbamoyl phosphate synthetase I b. hypoxanthine-guanien phosphoribosyltransferase c. N-acetylglutamate synthetase d. ornithine transcarabamylase e. uridine monophosphate synthetase

Clinical features of fibromyalgia: Symptoms - widespread musculoskeletal pain, fatigue, impaired attention and concentration, psychiatric disturbances (eg, depression, anxiety), symptoms lasting for more than 3 months. PE: multiple tender points at characteristic locations. Absence of joint or muscle inflammation Labs: normal acute phase reactants (eg, ESR, CRP) and other inflammatory markers. Fibromyalgia is a chronic disorder that is characterized by widespread musculoskeletal pain in association with fatigue and neuropsychiatric disturbances (eg, paresthesias, poor sleep, depression, difficulty concentrating). It its most common in women age 20-55. Examination often shows tenderness at characteristics locations in the soft tissues and at bony prominences. The diagnosis can be made in patients with chronic pain and fatigue for over 3 months in the absence of physical or lab findings suggestive of an inflammatory etiology (eg, synovial swelling, elevated erythrocyte sedimentation rate or C-reactive protein) or other chronic pain syndrome. The price etiology of fibromyalgia is unknown, but it likely involves abnormal central processing of painful stimuli. Possible contributing actors including genetic predisposition and physical or emotional stressor. Although exercise can temporarily exacerbate the pain of fibromyalgia, gradual incremental aerobic exercise is proven to reduce pain and improve function. Treatments: TCAs and SNRIs modify processing pain signals and can also be considered in patients with more severe or refractory symptoms. Ankylosing spondylitis is a chronic inflammatory disease of the axial skeleton characterized by progressive pain and stiffness of the spine, sacroiliitis, and positive serology for HLA-B27 in the majority of patients. Dermatomyositis is an autoimmune condition the causes bilateral proximal muscle weakness associated with violaceous eruption on the eyelids and knuckles, and elevated creatinine kinase levels. Polymyalgia rheumatic is an inflammatory disorder that affects patients over 50 and causes subacute pain and stiffness in the shoulder and hips, weight loss, fever and malaise. The patient's age, soft tissue tenderness and neuropsychiatric symptoms are not consistent with PMR but are typical of fibromyalgia. RA causes symmetric inflammatory polyarhtirtis. It typically presents with pain and morning stiff ness of the metacarpophalangeal and proximal interphalanageal joints. the affected joints are warm, swollen and tender on palpation.

867 A 45 y.o has 3 month history of persistent shoulder and back pain and easy fatiguability. She also has pain and stiffness in all her muscles that worsens in the morning and evening. Exercising exacerbate the pain. PMH is significant for depression and GIT reflux disease, but she is not currently taking any meds PE shows normal range of motion and 5/5 muscle strength in both upper and lower extremities. Soft tissue tenderness is present at several locations bilaterally above and below the waist. Which of the following is the most likely diagnosis? a. ankylosing spondylitis b. dermatomyositis c. fibromyalgia d. polymyalgia rheumatic e. rheumatoid arthritis

The transition form point 1 to point 2 on the graph shows the loading of O2 into partially deoxygenated hemoglobin. At very low pO2, the Hb molecule is fully deoxygenated and binding of the first O2 is relatively difficult ( as indicated by the early flatness of the curve). As pO2 increases, O2 binds to 1 of the 4 heme moieties on the hemoglobin molecule, causing the oxygen binding affinity of the other Hb subunits to increase (steepening of the curve). Additional O2 molecules bind as the PO2 increases. As Hb become saturated with oxygen, very little additional binding occurs, and the curve levels out. In the peripheral tissue, the release of O2 from Hb is enhanced by increased pCO2 and the resultant decrease in pH (Bohr effect). This effect occurs d/t the histidine side chains found on the alpha and beta hemoglobin subunits. As the tissue release CO2, the majority is converted by erythrocyte carbonic anhydrase to bicarb and Hydrogen ion. While bicarb is shifted out of the erythrocytes in exchange for Cl ions found in the plasma, the hydrogen ions remain within the erythrocytes. The hydrogen ions are buffered by the histidine residues on Hb and in the process, help stabilize the deoxygenated form of Hb and decrease its affinity to oxygen. When deoxygenated blood enters the alveolar capillaries of the lung, the rise of pO2 increases the binding of O2 to Hb and causes the release of H and CO2 from Hb (Haldane effect). As bicarb shifts back into the erythrocytes in exchange for Cl, carbonic anhydrase converts H and bicarb back into the CO2 and water. The CO2 is then excreted through the lungs. Erythrocytes contain a higher concentration of chloride ions in venous blood than in arterial blood d/t the chloride shift. However, neither Cl nor phosphate is transported in Hb. Heme is NOT released from Hb in the context of oxygen loading or dissociation. However, it is released during normal destruction of aged RBCs in the spleen. In the lungs, higher pH, lower temp and decreased concentration of 2,3-DPG shift the oxygen dissociation curve to the left, increasing the affinity of Hb to oxygen.

1386 A graph shows the sigmoidal relationship between %O2 saturation and pO2. Point 1 is located in the exponential portion while point two is at the top where the curve is starting to tapers off. During this transition, hemoglobin molecules are most likely to release which of the following things? a. chloride b. heme c. O2 d. phosphate e. protons

Patient has gas gangrene from Clostridium perfringens (toxin-producing gram positive rods) Lecithinase is the main toxin, aka phospholipase c or alpha toxin, catalyzes the splitting of phospholipids molecules. It hydrolyzes lecithin contaitin lipoprotein complexes in cell membranes, causing cell lysis (including erythrocyte hemolysis), tissue necrosis and edema. In contrast, human phospholipase A2 catalyzes arachidonic acid relate form phsophlipid cell membranes in the first step of leukotriene, thromboxane and prostaglandin synthesis. C perfringens uses carbohydrates for energy. Its rapid metabolism of muscle tissue carbohydrates produces significant amounts of gas, which can be demonstrated on radiographic imaging. a: actin polymerization is from cytotoxin B from C. diff leading to mucosal cell death, necorisi of colonic mucosal surfaces and pseudomembrane formation f. presynaptic Ach release are from clostridium species. c EF-2 ribosylation is MOA of diphtheria toxin and P. aeruginoas e. plasminogen activators such as streptokinase, urokinase and tissue plasminogen activator convert plasminogento plasmin. Of these enzymes, only streptokinase an exotoxin related by Strep pyogenes is a bacterial product g. certain s. aureus toxins act as super antigens by hyper stimulating T cells and causing massive cytokine production. Examples include enterotoxin and TSST.

1395 Dude step on an old rusty nail now he has gas gangrene. Culture from site reveals gram positive rods. Which of the following best describes the MOS of the toxin responsible for the necrotic effects seen in this patient? a. actin depolymerization b. carb degradation c. EF ribosylation d. phospholipid splitting e. plasminogen activation f. presynaptic ACh release g. T cell hyper stimulation

DNA replication: -helicase enzyme and single stranded DNA binding proteins unwind the double helix DNA. This site is call the replication fork, it is bidirectional. -synthesis of the daughter strand simultaneously occurs from both parent strands. -DNA synthesis can occur only in th e5'-3' direction, one daughter strand is synthesize continuously towards the replication fork (leading strand), while the other strand must be synthesized discontinuously in a direction away from the replication fork (lagging strand), with more and more segments being added as the replication fork moves across the DNA double helix. -This results in the formation of Okazaki fragments, short stretches of newly synthesized DNA that are separated by RNA primers. These primers are removed and replaced with DNA, and the Okazaki fragments are subsequently joined together by DNA ligase b/c: -5-3 exonuclease activity removes RNA primers as well as damaged DNA segments -3-5 exonuclease activity performs a proofreading function that removes and replaces mismatched nucleotides on the newly formed daughter strand. -This occur at both daughter strand e: before DNA polymerase can initiate DNA synthesis, RNA primers must first be synthesized by the enzyme primate (DNA-dependent RNA polymerase). Only one primer is needed for synthesis of each leading strand, but the lagging strand requires synthesis of many RNA primers

1418 Dude has ALL and was started on doxorubicin. This agent intercalates between DNA base paris and inhibits DNA replication, a process that normally occurs at the replication forks. As the replication forks move across the DNA molecule, 2 distinct daughter strands are formed. Which of the following is unique to the daughter strand that is synthesized in the opposite direction of the growing replication fork? a. synthesis of multiple, short DNA fragments b. 5-3 exonuclease act of DNA pol c. 3-5 exonuclease act of DNA pol d. 3-5 pol activity of DNA pol e. RNA primer synthesis before DNA strand synthesis

S. pneumonias is the most common cause of CA pneumonia. However, alcoholics has a significantly increased risk of aspiration pneumonia (infection with oral flora) because alcohol intoxication impairs the gag and cough reflexes. Alcoholics with poor oral hygiene have increased numbers of oral bacteria, further increasing this risk. In addition, alcohol may impair the phagocytic and/or bactericidal action of alveolar macrophages, predisposing to infection. Pulmonary infections in alcoholics patients very commonly include anaerobic oral flora (bactericides, prevotella, fusobacterium and peptostreptococcus) admixed with aerobic bacteria. Necrotizing infections and lung abscesses may result. This patient's CT shows a lung abscess that can be identified by presence of air fluid levels. Of antibiotics choices given, CLINDAMYCIN has the most activity against oral anaerobes and also covers aerobic Gram-positive organisms such as S. pneumoniae). Similar to macrolide, lindamycin works primarily by being to 50s ribosomal subunit in bacteria and disrupting protein synthesis. b. vanco is active only against gram positive bacteria and aren't very good against anaerobes. c/e: Klebsiella is a common pathogen in alcoholics with aspiration pneumonia and can also cause mono microbial lung abscess. However, lung abscesses are most recently caused by polymicrobial infection involving anaerobes. Ciprofloxacin and cefazolin have poor coverage against anaerobic organisms. d. metronidazole has excellent coverage against anaerobes but doesn't cover aerobic organisms. Adequate covers against both aerobic strep and anaerobes is essential for successful treatment of lung abscesses.

1446 A 41 y.o man has fever, weigh loss and a productive cough. He drinks 12-18 beers a day. PE shows could smelling sputum and poor dentition. Chest CT shows abscesses. What antibiotics should you give him? a. clindamycin b. vancomycin c. ciprofloxacin d. metronidazole e. cefazolin

Vasospasm occurs in 20-30% of patients with subarachnoid hemorrhage (SAH) caused by ruptured intracerebral aneurysm such as a berry aneurysm. Symptoms of cerebral vasospasm include: -altered mental status (his confusion) -focal neurological deficits (this patient's weakness of right arm and leg) Etiology: substances generated by the degradation of subarachnoid blood clots. Symptoms manifest no earlier than three days after the hemorrhage, and most frequently occur seven to eight days following SAH. DOC: Calcium channel blocker (eg, Nimodipine). This drug is identified as a cerebral selective drug. MOA is unknown. Might be vasodilation or neural protectant. Bet-adrenergic blocker (eg, metoprolol) would decrease HR and PB. No role in treatment of SAH or intrecerebral vasospasm. Central sympatholytics (eg, methyldopa and clonidine) stem alpha-2A receptors to decrease generalized sympathetic outflow. This would decrease blood pressure in addition to decreasing both alpha and beta sympathetic stimulation. They do not play a role in vasospasm following SAH therapy, however it is useful in intracranial hemorrhages such as those caused by HTN. ACE inhibitors (captopril and enalapril) decrease BP and cardiac hypertrophy by inhibiting the cover sion of Angiotensin I to Angiotensin II. They are useful in HTN, prevention of diabetic nephropathy, and in HF with systolic dysfunction. Major side effects are cough and angioedema. Thiazide diuretics are presently the first line of treatment for essential HTN in the outpatient setting. Osmotic diuretics such as mannitol are part of the treatment for increased intracranial pressure, IN SAH, vasospasm, not ICP, is the main problem. Nitrates such as nitroglycerin and isosorbide mononitrate are used as vasodilators primarily for the treatmenet of anti pectoris.

146 Patient has polycystic kidney disease. He has severe headache and confusion. CT shows blood in the subarachnoidspace. On the fifth day after admission, the patient begins to complain weakness in his right arm and leg. Which of the following drugs could most likely have prevented this patient's neurological sequelae. a. beta adrenergic blocker b, central sympatholytic c. ace inhibitor d. thiazide e. osmotic diuretic f. calcium channel blocker g. nitrate

Within the zone of autoregulation, increase in coronary blood flow are primarily mediated by the relative myocardial hypoxia that occurs during times of increased work. At baseline levels of activity, the myocardium extracts over 70% of the available oxygen from the blood it receives. When the oxygen demand increases, increased blood flow is required because the percent oxygen extraction from hemoglobin cannot be significantly increased (i.e., resting O2 extraction is already near maximal). Thus, increased myocardial oxygen requirements during exercise can only be achieved from a corresponding increase in coronary blood flow. Local factors influencing coronary blood flow: -adenosine -prostaglanidns -serotonin -autonomic nervous system neurotransmitter -bradykinin -mechanical shear stress -thrmobin -NO (endothelium derived relaxation factor) There are numerous local factors that are important in influencing coronary blood flow. Of these *adenosine and NO* are the most important factors involved in coronary blood flow auto regulation -Adenosine, a product of ATP metabolism, acts as a vasodilatory element in the *small coronary arterioles.* -NO is created by and released from endothelial cells in the coronary vasculature. It is synthesized from arginine and oxygen by endothelial nitric oxide synthase (eNOS), and is released from coronary endothelium in response to neurotransmitters (eg, ACh and NE), platelet products (i.e., serotonin, adenosine diphosphate), thrombin, histamine, bradykinin and endothelin. NO is also released in response to pulsatile stretch and flow shear stress in the coronary arteries. It is the major regulators flow-mediated vasodilation in *large arteries and pre arteriolar vessels*. -NO acts within the vascular smooth muscle cells via a soluble guanylate cyclase enzyme to increase production of cyclic GMP and cause smooth muscle relaxation. NE is a neurotransmitter released from sympathetic neurons. Alpha-1 receptors cause construction of blood vessels. Alpha 2 are centrally located and cause inhibitor of the sympathetic system. Beta-1 receptors cause excitation of the heart. Beta-2 receptors (not stimulated by NE) cause vasodilation. despite these these effects nervous input generally has very little effect on coronary blood flow. Each is a NT released from parasympathetic neurons. It acts on muscarinic receptors to affect an inhibitory influence on the heart. As stated, nervous input has very little effect on coronary blood flow. Angiotensin II is a very powerful vasoconstrictor that acts to regulate systemic, not local, blood pressure Histamine is a potent vasodilator released primarily from mast cells when there is tissue damage. It increases capillary permeability, leading tot local edema. Serotonin is produced by gut neuroendocrine cells, platelets, and serotonergic neurons in the CNS. Little is known about its vasoactive affects; it has been shown to have both vasoconstrictor and vasodilator effects.

1516 Researchers are observing how coronary blood flow changes in response to progressive increases in MAP. Graph give a sigmoidal relationship between blood flow and MAP. The region where coronary blood flow is relatively insensitive to blood pressure changes (horizontal portion of graph). Within the zone of auto regulation, the metabolic demand of the myocardium are the main determinant of coronary blood flow. Which of the following endogenous factors is most responsible for controlling coronary blood flow within this range? a. NE b. Ach c. NO d. angiotensin II e. histamine f. serotonin

For a person in th upright position, regional differences in ventilation (V) and perfusion (Q) occur vertically in the lungs d/t the effects of gravity. The "slinky effect" on lung ventilation -End expiration: pleural suction fixes lung apex while gravity pulls down. Alveoli are stretched open more at apex than the base. -End inspiration: Lugn elastically expand during inspiration so that alveoli are of similar size. Alveoli expand more at the base than the apex, thus ventilation increases from apex to base. -Thus ventilation increases from the apex to base because gravity acts to stretch the lung downward from the apex, which is effectively fixed to the pleural cavity by the low intrapleural pressure. As a result, at end expiration, alveoli at the apex are expanded more than those at the base. During inspiration, a smaller amount of air id directed to the l uno apex since the alveoli there are more distended and less compliant. Regional variance in perfusion of the lungs is also determined by gravity and can be dived into 3 zones based on the continuity of blood flow: Zone 1) doesn't occur in the lung under normal physiologic conditions but would be found at the apex. IN this zone, *alveolar pressure is greater than arterial pressure which is greater than venous pressure.* The arterial pressure is low in this region as the heart must pump blood uphill against the force of gravity. Because arterial pressure is low than alveolar pressure, the pulmonary capillaries are collapsed and there is no blood flow (i.e., zone 1 represents alveolar dead space). These regions can form when there is low pulmonary arterial pressure (eg, hemorrhage) or high alveolar pressure (eg, positive pressure ventilation). Zone 2) is found in the higher areas of the lung. Here the *arterial pressure > alveolar pressure > venous pressure,* because alveolar pressure is greater than venous pressure, the pulmonary capillaries are initially obstructed near the venous end of the capillary bed. However, as arterial pressure rises during systole, capillary pressure becomes high enough to overcome alveolar pressure. For this reason, blood flows through zone 2 in a pulsatile fashion. Zone 3) is found in the lower areas of the lung. Here, the *arterial pressure > venous pressure > alveolar pressure.* Arterial venous pressure are both greater than alveolar pressure, and therefore blood flows continuously through the pulmonary capillaries. When a person lies supine, the differences in lung perfusion are negated as gravity then affected the lung equally form apex to base. In spin individual, the lung is entirely composed of zone 3 blood flow. Although both ventilation and perfusion increases from apex to base, perfusion increases to a MUCH greater degree. Thus, as one moves down the lung from the apex to the base, the V/Q ratio progressively decreases from around 2.5 to about 0.6.

1541 Describe the V/Q ratio and blood flow along the lung from base to apex for a normal person standing in the upright position.

Salicylate intoxication causes different acid base abnormalities simultaneously: 1) Respiratory alkalosis: is the first disturbance to occur, as salicylate directly stimulate the medullary respiratory center. The resulting increase in ventilation leads to increased loss of CO2 in the expired air. 2) Anion gap metabolic acidosis: begins to develop shortly afterwards,as high concentrations of salicytlates increase lipolysis, uncouple oxidative phosphorylation and inhibit the citric acid cycle. This results in the accumulation of organic acids in the blood (eg, ketoacids, lactate, and pyruvate). Because presenting approximately 4-5 hours after salicylate ingest, it is likely that the initial primary respiratory alkalosis has processed to mixed respiratory alkalosis/metabolic acidosis. The ABG in salicylate toxicity often shows a pH in normal ranges as the 2 priory acid base disturbances shift the pH in opposite directions. In addition, theABG will show low PaCO2 (t/t respiratory alkalosis and compensation for metabolic acidosis) and low HCO3 (d/t metabolic acidosis). Not that the PaCO2 will be lower than the predicted respiratory compensation as estimated by Winter's formula formula (in this case, expected PaCO2 = (1.5 x 12) + 8 +/- 2 = 26 +/- 2= 25-28 mm Hg, indicating that the low PaCO2 is not d/t to compensation alone and that there is a concurrent primary respiratory alkalosis.

1544 Bitch drank a whole bottle of aspirin. Four hours later, what will her pH, PaCO2 and plasma HCO3 are going to be like?

Water deprivation results in ADH release from the posterior pit. ADH stimulates V2 receptors on principal cells in the renal collection ducts, activating a cAMP second-messenger G-protein system that leads to insertion of endosomes containing aquaporin 2 into the apical cell membrane. Aquaporin 2 is a water channel that spans the luminal membrane, enhancing the water permeability of the principal cells. When ADH feels are low, aquaporin 2 accumulates within pits along the luminal membrane and is returned to cell cytoplasm through endocytosis. In the presence of high ADH, the tubular fluid osmolarity follows this patter. -in the proximal tubule (choice A), water is reabsorbed along with electrolytes. The tubular fluid in this segment remains isotonic with plasma (300) whether the final urine is concentrate or diluted -in the descending limb of the loop of Henle, free water is drawn out of the tubules into the renal interstititum and the tubular fluid becomes hypertonic (>300, typically reaching 122 when ADH is high -electrolytes, but not water, are reabsorbed in the thin ascending limb of the loop of Henle. The tubular fluid becomes hypotonic (<300). the thick and think ascending limbs ar primary region of urine dilution, achieved mainly through NaCl reabsorption -the distal convoluted tubule is relatively impermeable to water, so the tubular fluid remains hypotonic. Reabsorption of solutes continues to occur; thus, fluid in the distal tubule is the most dilute (lowest osmolarity, approaching 100) - in the presence of ADH, the collecting duct is highly permeable tow water. Water leaves the tubular fluid driven by the high osmolarity of the medullary interstitum and hypertonic urine is formed (up to 1200 mOsm/L). The CD system is the primary region of urine concentration through the mechanism of DH mediated water absorption.

1608 At what part of the nephron does tubular fluid osmolarity is 110 mOsm/L from a healthy person after 12 hours of water deprivation?

B-cell precursors proliferate and mature in the bone marrow. They then leave and go to the lymphoid organs and peripheral tissues, where they are exposed to antigens. On first exposure to a new antigen, a clone of B-cells becomes activated. Some activated B-cells differentiated into short lived plasma cells that release antigen specific IgM through a T-cell independent process. However, most activated B cells migrate to lymphoid follicles located in the lymph node cortex where they form germinal centers that are the site of B-cell proliferation during the immune response. A portion of these activated B cell form long lived memory cells that remain dormant in the lymph node until the next encounter with the same antigen, but the majority transform into antibody secretin plasma cells. Isotope switching (from IgM to other types of immunoglobulins) also occur in the germinal centers later in the primary response, providing activated B-cells the ability to produce antigen-specific antibodies of differing isotopes. Heavy chain constant region are isotope-specific and distinguish the 5 isotopes (IgM, G, A, E and D), while the variable regions are antigen specific. Light chains are antigen-specific and do not determine isotope. Isotope switching first requires interaction of the CD40 receptors on activated B-cells with the CD40 ligand (CD154) expressed by activated T-cells. Afterward, isotope switching can occur through genetic rearrangement of the heavy chain constant regions. This process is regulated by T-cell cytokines such as IL-2, 4, 5, 6 and IFN-y. After the primary immune response, subsequent encounter with the same antigen generate a predominantly IgG response (or IgA in the case of mucosal response). Negative selection and tolerance refers to the deletion of T-cell clones that strongly bind to self-MHC antigens such that an immunologic unresponsiveness to self antigen is selected. This process occurs in the fetal thymus. Peripheral tolerance develops by means of T-cell anergy, which is the functional inactivation of T-cells that a re reactive to self antigens. VDJ = heavy chains. VJ = light chain occur via DNA rearrangement. After undergoing Ig gene rearrangement, each B-cell makes antibodies of a single specificity. an enormous variety of different Ig molecules can potentially be produced through rearrangement. Recombination of these regions occurs during B-cell maturation within the bone marrow. Later on, during the primary immune response, affinity maturation occurs in the germinal centers through the process of somatic hypermutation. The primary immune response to a new antigen initially results in plasma cells that only produce IgM. Isotope switching later occurs in the germinal center of lymphondes and requires interaction of the CD40 receptor on B cells with the CD154 expressed by activated T-cells. IgG is the main serum Ig of the secondary response.

1614 What is going on in a germinal center? a. isotope switching b. negative selection c. tolerance development d. VDJ recombination e. VJ recombination

The diagram described illustrates a set of vessels arranged in parallel. Systemic circulation is often simplified in the sway for the purposes of calculating total peripheral resistance (TPR). Each branch of the the aorta can be visualized as one of the vessel branched connecting the inflow and outflow vessels. For example, the four vessels can represent coronary, splanchnic, renal and cerebral circulation. The formula for calculating the total resistance for parallel circuits is as follows. RT represent total peripheral resistance, and the circuits represent blood vessels: 1/Rt = 1/R1 + 1/R2 + 1/R3 + .... 1/Rn In this calculation, the inverse of TPR (or Rt) is equal to the sum of the inverse of each vessels or system that lies in parallel. so 1/TPR = 1/2 + 1/2 + 1/2 + 1/2 = 4(1/2) = 2 so 1/2 = .5 a. 8 would be the resistance if each of the avovve vessels were arranged in series, rather than nparallele. A series arrangement does not estimate total body circle well, but is a good way to describe blood flow in an individual organ. to calculate resistance in organ just simply add up R-artery + R-arteriole + R-cap + R-venule + R-vein

1616 A circuit shows inflow pressure of 120 mmHg and outflow pressure of 40 mmHg, with four vessels in between. R1=R2=R3=R4 =2. What is the total peripheral resistance of the circuit? a. 8 b. 4 c. 2 3. 1 e. .5

The GFR depends on interplay of hydrostatic and oncotic pressures in the glomerular capillaries and Bowman's space. The GFR increases with high glomerular hydrostatic pressure and decrease with increasing Bowman's capsule hydrostatic pressure or high glomerular capillary oncotic pressure. Acute ureteral obstruction increases hydrostatic pressure proximal to the constriction. This pressure rise is transmitted back to the Bowman's space, resulting in decreased GFR. FF is portion of the RPF that is filtered from the glomerular capillaries into the Bowman's space (i.e., the GFR:RPF ratio). With acute ureteral obstruction (first 12 hours), the RPF may transiently increase; however, with time, efferent arteriolar constriction (in response to reduced GFR) will decrease RPF. Even at later stages, though, the GFR remains depressed to greater extent than the RPF, resulting in reduced FF.

1651 A 48 y.o woman is being evaluated for postcoital bleeding and vaginal discharge. Pelvic exam shows a viable mass at the cervix that bleeds easily on touch. Cervical biopsy shows invasive SCC confined to the cervix and uterus. Lymph node metastases are not seen. A radical hysterectomy is performed during which the right ureter is accidentally injured but then repaired. Imaging studies performed after the surgery show a partial obstruction of the right ureter with mild dilation of the proximal collecting system. Which of the following changes are to be seen? a. GFR = no change: FF increase b GFR increase: FF decrease c. GFR decrease: FF increase d. GFR decrease: FF no change e. GFR decrease: FF decrease

Mallory-Weiss tear Vs. Boerhaave syndrome MW: caused by upper GI mucosal tear. caused by forceful vomiting (increase intragastric pressure). submucosal arterial or venous plexus bleeding. C/Ps: vomiting, retching, hematemesis and epigastric pain Boearhaave: caused by esophageal transmural tear. Caused by formula vomitng (increase intragastric pressure). Esophageal air/fluid leakage into the mediastinum and pleura. C/Ps: vomiting, retching, chest and upper abdominal pain, fever, dyspnea, septic shock rapidly ensure. Patient has Mallory Weiss tear of gastric mucosa near the gastroesphageal junction. Repetitive vomiting leads to *metabolic alkalosis* d/t loss of acidic gastrin secretions. a. unlike methanol or ethylene glycol, acute ethanol intoxication doesn't usually cause high anion gap metabolic acidosis, but chronic alcoholism can cause ketoacidosis in malnourished patients. Ethanol abuse is a common predisposing factor for Mallory-Weiss tears; however, it is the repetitive vomitng and retching the tdierectly cause the tears. c. normal anion gap metabolic acidosis is usually caused by loss of bicarb, which can occur with prolonged diarrhea. recurrent coming causes metabolic alkalosis, not acidosis. Respiratory acidosis usually occurs in patients with COPD or CNS depression (eg, narcotic overdose). Respiratory alkalosis occurs with hyperventilation (eg, panic attacks, pulmonary emboli)

1662 a 34 y.o man has upper GI bleeding. He is an alcoholic. Gastric lavage fluid initially contains blood but clear quickly. Upper endoscopy shows a linear mucosal tear at the gastroesophageal junction. The process directly responsible for causing this patient's mucosal tear will most likely result in which of the following acid-base disturbances? a. high anion gap metabolic acidosis b. metabolic alkalosis c. normal anion gap metabolic acidosis d. respiratory acidosis e. respiratory alkalosis

Patient has acute gouty arthritis from treatment with niacin (nicotinic acid). Niacin improves hyperlipidemia by decreasing hepatic synthesis of TAGS and VLDL and reducing clearance of HDL. *It is the most effective agent for rating HDL levels.* However, as it has not been shown to reduce the risk of cardiac events, it is primarily used in patients who have ailed other lipid lowering drugs. The use of niacin is limited by several significant side effects including: -flushing -hyperglycemia -hepatoxicity -decrease renal excretion of uric acid, leading to elevated blood levels and an increased risk of acute gouty arthritis a. atovastatin: hepatitis and myalgia b. cholestyramine: bile acid binding resins (eg, cholestyramine) are GIT upset, impaired absorption of nutrients and drugs and hyperTAGS. c. Ezetimibe selectively inhibits intestinal absorption of cholesterol. It is primarily used in conjunction with statin therapy for the treatment of hypercholesterolemia. However, ezetimbie can increase the hepatoxocity seen with statins. Fabric acid derivatives (eg, gemfibrozil, fenofibrate) are used to lower TAGs. Vibrates can cause severe myopathy when given with statins and can increase the risk of cholesterol gallstones.

167 A 56 y.o has severe right foot pain. Pain began acutely without history of trauma. No redness, swelling or fever. PMH is significant for mixed hyperlipidemia treated with lifestyle modifications and sever medications. PE shows a swollen, tender first metatarsophalangeal joint. Aspiration reveals a high leukocyte count, negative Gram stain, and numbers needle-shaped negatively birefringent crystals. Which of the following drugs most likely caused this? a. atovastatin b. cholesteryramine c. ezetimibe d. gemfibrozil e. niacin

Patient has a "clasp-knife" spasticity, which is characterized by initial resistance to passive extension followed by a sudden release of resistance. This is seen with UMN lesion and results from lack of UMN inhibition on the spinal stretch reflex arc. Upper motor neuron lesions can affect any part of the pyramidal motor system (including the corticospinal tract of the spinal cord; the medulla, pons and midbrain; the internal capsule a; and the precentral gyrus (primary motor cortex). Patients with an internal capsule stroke commonly have a premotor weakness affecting the contralateral arm, leg and lower face. Contralateral spasticity or increased tone, hyperreflexia and positive Babinski sign are also present. Caudate nucleus can lead to extrapyramidal symptoms such as chorea and athetosis. Insult is involved with integrating body states with emotions (limbic system), autonomous nervous system control and conscious experience of visceral sensation. The putamen is part of the basal ganglia ( extrapyramidal motor system) and plays a role in initiation of movement. Lesion in the putamen can cause contralateral tremors, bradykinesia and rigidity. Globus pallidus is also part of the basal ganglia/extrapyramidal motor system. It consists of an external and internal segment. Damage to the external will cause decreased motion/movement; damage to the internal segment will result in excessive motion/movement.

1687 A 62 y.o man has left arm clumsiness. PE shows motor weakness involving the left arm and leg. His speech is slurred and there is drooping of the left lower face. On passive extension of the left arm, there is initial resistance followed by a sudden release of tension as extension is continued. A lesion affecting which of the following brain structures is most likely responsible fore this patient's condition? a. caudate nucleus b. internal capsule c. insular cortex )insult) d. putamen e. globes pallidus

At lower doses, the quantity of drug glucuronide formed is directly proportional to the dose administered (i.e., as more of the drug is administered, more drug-glucuronide is produced). Because a fixed proportion of drug is converted to the metabolite, this early portion of the graph represents first-order kinetics. As the active sites of glucuronosyltrransferase become saturated, the drug-glucuronide rate doesn't continue to rise with increasing doses of the substrate drug. At this point (2 on the graph), the kinetics change from first to zero order kinetics and the graph levels out to a zero slope. With zero order kinetics, a constant amount of drug is metabolized an eliminated per unit of time regardless of its concentration or dose Zero- and first-order kinetics can also be represented graphically by showing the decrease in drug concentration over time after a single dose. Zero order metabolism is indicated by a straight line with negative slope (fixed amount eliminated per unit of time), whereas first order kinetics manifest as an exponential decay curve (fixed proportion eliminated per unit of time). a. After point 3, the curve remains flat, indicating that a constant amount of drug is metabolized per unit of time regardless of the dose. As higher doses are administered, the proportion that is converted to the metabolite becomes smaller. b. bioavailability (the fraction of unchanged drug that reaches the systemic circulation) usually decreases with oral administration d/t incomplete absorption and first pass metabolism. It can be calculated using graphs of plasma drug concentration versus time; however, this information is not provided. c. at point 2, biotransformation (i.e., metabolism) of drug begins to switch from first order to zero order kinetics d/t enzyme saturation. However, drug metabolism doesn't stop. e. before point 1, drug metabolism proceeds via first order kinetics. Therefore, the drug metabolization rate increases as higher doses are administered.

1716 A large, multinational drug corporation conducts a phase I clinical trial to evaluate the safety profile and pharmacokinetic properties of a new drug designed to treat refractory epilepsy. Initial studies in animals showed that the drug undergoes extensive metabolism by the liver into glucuronidation byproducts that are primarily excreted by the kidneys. The curve below demonstrates the glucuronidation rate of the drug over a wide range of doses. A graph represents the relationship between drug dose and glucoronidation rate where there are three points on a curve that started out as first order kinetic then turns into zero order kinetic, which begins at point 2. Which of the following is the most accurate statement about this drug's metabolism? a. constant proportion of the drug is metabolized past point 3 b. bioavialabiility of the drug is highest at point 3 c. transformation of the drug ceases shortly after point 2 d. metabolism begins to switch to zero order kinetic near point 2 e the rate of drug metabolism is not dependent on dose before point 1

Drug resistance is associated with prolonged use of highly active antiretroviral therapy (HAART), a regimen that includes inhibitors of HIV RT and protease. The development of such drug resistance has been attributed to high mutation rate of the HIV genome and the selective pressure exerted by antiretroviral drugs. Pol gene mutations are responsible for the emergence of HIV protease variants that are resistant to standard protease inhibitors. The pol gene mutations are also transcriptase that render the enzyme resistant to standard nucleoside and non-nucleoside RTIs. a. The humoral immune response against HIV is comprised of neutralizing Abs directed against the epitopes of folded viral envelop glycoproteins. Frequently emerging after the primary infection are new HIV-1 "escape mutants" that are no longer susceptible to host antibody neutralization. Because structural glycoproteins are encoded for the HIV-2 env gene, viral evasion of humoral immunity is more likely to occur secondary to a mutation of the env gene. b. activation induced apoptosis can occur in uninfected CD4 cells as they respond to MHC class II-associated HIV peptides on the surfaces of infected cells. Abnormal intracellular signals transducer by HIV might also prime the infected CD4 T cells for apoptosis. Hypothetically, a reduction in CD4 T cell apoptosis could result if mutations in the HIV genome either changed the MHC class II antigenic peptides on the infected cells or reduced the host intracellular caspase activation. However, it is unclear to what extent these alterations actually occur. Moreover, no direct association between these alterations and pol gene mutations has been established. c. reduced viral replication is not likely to be associated with high rate of genomic mutations in HIV, as the replication rate is generally positively correlated with the mutation rate. d. HIV-1 possesses the intrinsic ability to mutate rapidly. During both intimal and chronic infection mutations can develop in the HIV-1 epitopes recognized by CD8+ cytotoxic T cells. As a result, cell-mediated immune (CMI) responses may quickly become inadequate to control HIV. However, a specific linkage between such CMI epitope evolution an pol gene mutations has not been established.

1722 A 54 y.o male who has been HIV-positive for more than five years presents to clinic for evaluation. the viral strains isolated from blood samples drawn at this visit demonstrate a significant increase in pol gene mutations over the last year. Which of the following is this observed finding most likely related to? a. evasion of humoral immune response b. prevention of CD4 cell apoptosis c. reduced viral replication d. ineffective CD8 cytotoxicity e. antiretroviral chemotherapy

The lip and palate form doing the fifth-sixth week of embryonic development through a series of scions: 1. the first pharyngeal arch splits into the upper maxillary province and the lower mandibular prominence 2. fusion of the 2 medial nasal prominences form the midline inter maxillary segment. The inter maxillary segment will beomce the philtrum of the upper lip, the 4 medial maxillary teeth and primary palate. 3. the left and right maxillary prominences then fuse with the midline inter maxillary segment to form the upper lip and primary palate. If one of the maxillary prominences fails to fuse with inter maxillary segment, a unilateral cleft lip will occur. If both maxillary prominences fail to fuse with the inter maxillary segment, bilateral cleft lip results. a. failure of the medial nasal prominence to fuse and form the inter maxillary segment is associated with severe midline defects, such as holoprosencephaly c. during the 7th-8th week of embryonic development, the maxillary prominences give rise to palatine shelves. The thin sheets of tissue that comprise the palatine shelves grow medially and fuse into the secondary plate. Fusion of the secondary palate with the posterior aspect of the primary palate form the complete palate. Failure of any of these process can lead to clef palate. The presence of cleft lip increases the risk of cleft palate, but either can occur in isolation d. the orbits normally arises from the sides of the face and rotate medially. However, excessive tissue from the frontal nasal prominence could result in orbital hypertelorism (wide-set eyes) in addition to a broad forehead and wide nasal bridge e. micrognathia is characterized by hypoplasias of the mandibular prominence. In Pierre-Robin sequence, the sever micrognathia results in the posterior display meant of the tongue (glossoptosis) and prevents fusion of the secondary palate (cleft palate).

1740 A new born has intact palate and a unilateral cleft lip on the left side. This is resulted to from a failure of which of the following intrauterine processes? a. fusion of the 2 medial nasal prominences b. fusion of the maxillary prominence and inter maxillary segment c. fusion of the palatal processes d. .hyperplasia of the frontonasal prominence e. hypoplasia of the mandibular prominence

This patient most likely have a patent ductus arterioles (PDA). The ducts arterioles is an embryonic derivative of the 6th aortic arch that allow sfetal blood to pass directly from the pulmonary artery to the proximal descending aorta (bypassing the pulmonary circulation). This vessels usually closes shortly after birth secondary to decreased prostaglandin E2 (PGE2) levels and increased oxygen concentration. potency of the ducts after birth results in a left-right shunt that can cause left ventricler volume overload and symptoms of heart failure (eg failure to thrive, respiratory distress). PE will show continuous machinery like murmur and palpable thrill over the left infraclavicular region d/t to turbulent blood flow through the PDA. Pharmacologic closure of PDA can be achieved by PGE2 synthesis inhibitor (eg, indomethacin) in premature infants. However, older patients usually require surgical ligation or percutaneous PDA occlusion. Bulbus cordis forms the beginning of the ventricular outflow tract in the embryonic heart. This structure forms the smooth portion of the left and right ventricles adjacent to the aorta and pulmonary artery, respectively. The primitive atrium receives blood from the sinus venous in the embryonic heart and transmits it to the primitive ventricle. The primitive atrium forms the rough portions of the left and right atria. The sinus venous is an embryologic structure within the heart that receives blood from the venue cave. IN adults, this structure forms the smooth portion of the right atrium, known as the sinus venarum. 1st aortic arch: part of maxillary artery 2nd aortic arch: hyoid artery and stapedial artery 3rd aortic arch: common carotid artery, proximal internal carotid artery 4th aortic arch: on left (aortic arch) on right (proximal right subclavian artery) 6th: proximal pulmonary arteries. on left (ductus arterioles).

1751 A 4 y.o boy is being evaluated for failure to thrive, shortness of breath and exercise intolerance. PE of CVS shows bounding peripheral pulses and palpable thrills over the left upper sternal border. A continuous murmur best heard over the same region is noticed upon auscultation. If surgery is planned, the surgeon should intervene on a derivative of which of the follow embryologic structures? a. bulbus cordis b. fourth aortic arch c. primitive atrium d. sinus venosus e. sixth aortic arch

Raphe nuclei of the brain stem is where serotonergic neurons are primarily found. The nuclei are located in the midbrain, pons and medulla, and axons are from these cells project widely throughout the CNS to synapse on structures such as the cerebral cortex, thalamus, hypothalamus, cerebellum, hippocampus and spinal cord. They play an important role in sleep wake cycle, anxiety, mood, psychosis, sexuality, eating behavior and impulsivity. Medications targeting these neurons are used to treat antidepressants and anxiety disorders. SSRIs and TCA work by prolonging the amount of time that serotonin spends in the synaptic clefts before they are taken up by the presynaptic neurons. a. Caudate nucleus and putamen form the striatum, which functions in motor activities. In Huntington, there is a loss cholinergic an dGABA-releasing neuron in the striatum b. locus ceruleus houses the NE secretion neurons that participate in the "fight or flight" response to physical and emotional stress. It is located in the dorsal pons. These neurons are implicated in some anxiety symptoms, but they are not serotonergic and not affected by SSRIs. c. the nucleus basalis of Meynert houses the cell bodies of cholinergic neurons. In Alzheimer disease, these neurons secrete decreased amounts of Ach. e. red nucleus is located in the anterior midbrain. Its neurons participate in motor coordination of the upper extremities. f. the substantial nigra is Dopamine

1834 Where are serotonergic neurons located? a caudate nucleus b. locus ceruleus c. nucleus basalis of Meynert d. Raphe nuclei e. Red nucleus f. substantial nigra

Dude has MEN 1 which are your 3 P's Pancreatic Parathyroid Pituitary MEN 2a Pheochromocytoma Medullary thyroid cancer parathyroid MEN 2b pheochromocytoma medullary thyroid cancer marfinoid/mucosal neuromas

1845 A 34 y.o man comes to clinic d/t frequent accidents. He has been walking into doorways and stationary objects. PMH is significant for renal stones, for which he underwent definitive treatment with neck surgery 3 years ago. PE shows bitemporal visual field defects. In addition not brain imaging, this patient should also be screened for which of the following tumors? a. bone b. breast c. colonic d. pancreatic e pheochromocytoma

This patient is having extra medullary hematopoiesis, a condition characterized by erythropoietin-stimulated, hyper plastic marrow cell invasion of extra medullary organs. Extra medullary hematopoiesis is most frequently caused by every chronic hemolytic anemias, such as beta-thalassemia (which the child depicted here may well have had). Extramedullary hematopoiesis can cause a range of skeletal abnormalities. The expanding mass of progenitor cells in the BM thins the bony cortex an impairs bone growth. Pathologic fractures are common in the most symptomatic children. Maxillae overgrowth and frontal bossing are associated with the characteristic "chipmunk facies" observed in the pediatric population. a. RBC transfusions limit extra medullary hematopoiesis by reducing hypoxia b. immune def are not frequently associated with RBC hemolysis c. erythropoietin def is observed in patients with chronic renal disease and would be associated with anemia, not extramdeullary hematopoiesis. e. iron def results in a relative depression of erythropoietic activity and is not associated with extra medullar hematopoiesis f. cyanocobalaim (B12) def causes a megaloblastic anemia that is not associated with extra medullary hematopoiesis. g. intrahepatic extra medullary hematopoiesis can cause portal hypertension, and portal hypertension can cause hepatosplenomegaly. However, the reverse is not true.

1859 a 4 y.o male dies in hospital from overwhelming infection. Autopsy findings include bone deformities and hepatospelnemegaly. Clumps of erythroid precursor cells are found in the liver and the spleen. The presence of these precursor cells is most likely related to which of the following conditions? a. frequent transfusion b. immune def c. erythropoietin def d. chronic hemolysis e. iron def f. cyanocobalamin def g. portal HTN

Neutrophil myeloperoxidase is responsible for green color and pus and sputum in bacterial infections. It is a blue green heme based pigment molecules contained within the azurophilic granules of neutrophils and catalyzes the production of hypochlorous acid from chloride and hydrogen peroxide during the phagocytic respiratory burst. a/b: currant jelly sputum seen in pneumonia from Klebsiella and the rusty color occasionally seen in pneumococcal pneumonia are d/t extravasation of RBCs and Hb into sputum caused by extensive inflammation and necrosis c. Pseudomonas pneumonia can cause blue green pigment d/t production of pyocyanin, but Pseudomonas is not common cause of CAP in otherwise healthy adults. d. respiratory epithelium sloughing and mucopolysaccharide production can contribute to the formation of sputum in patients with respiratory infection but doesn't contribute to sputum's green color.

1910 A 44 y.o man has 3 day history of fever, chills, malaise, dyspnea, and a cough productive of "greenish" sputum. The patient has no prior medical problems and has never been hospitalized. He has a 25 pack year smoking history and drinks 4-5 beers a week. His temperature is 103, bp is 130/80 and rr 20. PE shows dullness to percussion, crackles, and egophony are present at the right lung base. X-ray shows a dense infiltrate occupying the entire right lower lobe. Which of the following most likely accounts for the color of this patient's sputum. a. epithelial necroses b. hemolysis c. high bacterial load d. mucopolysaccharides e. myeloperoxidase

This patient has follicular lymphoma, a non-Hodgkin lymphoma of the cleaved and non cleaved B-lymphocytes of the follicular center. The classic cytogenetic abnormality in follicular lymphoma is t(14:18) translocation, which moves the Bcl-2 (B-cell lymphoma-2) protooncogene from chromosome 18 to chromosome 14, near the site of Ig heavy chain enhancer element. Bcl-2 is considered a protooncogene because it has anti-apoptotic effects (prevents the release of pro-apoptotic factors within the affected ells). When this protooncognee is positioned near the Ig enhance element, the resultant Bcl-2 over expression allows for cell immortality. b. Bcr-abl hybrid formed as a result of a reciprocal translocation between chromosome 9 and 22 (the Philadelphia chromosome). Thesis the characteristic cytogenetic abnormality in chronic myelogenous leukemia (CML). c. C-myc over expression occurs in many malignancies. The mic oncogene is located on chromosome 8. Translocations between chromosome 8 with Ig heavy chain on 14, kappa light chain on 2, or lambda light chain on 22 may result in Burkitt lymphoma. ErbB2, HER2 and new are all names for same epidermal growth factor receptor, which is over expressed in many cases of breast cancer. p53 gene inactivation occurs as one of the two "hits" in many human malignancies. Normally, the p53 protein functions as a tumor suppressor. Li-Fraumeni syndrome is an autosomal dominant cancer syndrome caused by an inherited mutation in p53.

1911 Dude has a lump in his neck that has grown slowly over the past several months. He smokes a pack a day and drinks occasionally. ROS, he denies dysphagia, chest pain, weight loss or fever. Biopsy of the mass reveals abnormal cells with the t(14:18) chromosomal trnaslcoation. This chromosomal change is mot likely to cause which of the following abnormalities in gene expression? a. Bcl 2 over expression b. Bcr-abl hybrid formation c. C-myc over expression d. Erb-B2 over expression e. p53 inactivation

Clinical features of melanoma (ABCDEs) Asymmetry: when bisected, the 2 sides are no identical. Border irregularities: uneven edges, pigment fading off Color variegation: variable mixtures of brown, tan, black and red Diameter over 6 mm Evolving: lessons chaining in size, shape or color; new lesion -Patient has FH of skin cancer. -She has a melanocytic lesion seen on skin examination. -She has seizure resulting from metastasis to CNS (common places being GIT, bone, liver and lungs). It is a malignancy of malocytes, which are of neural crest origin. a. The endoderm gives rise to all structures derived from the inner lining of the primitive gut tube. These include the: -thyroid follicular cells -epithelial surfaces of the trachea, bronchi and lungs -liver and biliary tree -pancreas -GIT and bladder epithelium. b. mesoderm gives rise to the dermis as well as most bones, muscles, blood vessels and visceral tissue d. the neuroectoderm gives rise to the CNS, preganglionic autonomic neurons, retina and posterior pituitary. e. the surface ectoderm gives rise to the epidermis and its appendages, mammary glands (modified what glands), lens of the eye and adenohypophysis.

1960 A 45 y.o woman has tonic clonic seizure and right arm weakness. PMH is insignificant for seizure. FH reveals a mother that died of skin cancer. PE shows a black skin lesion of 13mm in her back.. This lesion most likely originated form which of the following embryologic derivatives? a. endoderm b. mesoderm c. neural crest d. neuroectoderm e. surface ectoderm

The electrical impulses in the myocardium are normally generated by the SA node at the rate of 60-100 bpm. These impulses are then transmitted through the conduction system to the ventricles, which depolarize from apex to base and from endocardium to epicardium. cells throughout the conduction system possess pacemaker ability. The SA node, however, normally stifles impulse generation by other cells in the conduction systems it is able to fire more rapidly. Should impulse conduction from the SA node be interrupted, the pacemaker activity of cells in other regions of the conduction system will be unmasked. The location of the pacemaker can be identified on ECG, which reflects the sequence of cardiac depolarization: -P: atrial depolarization -QRS: ventricular depolarization -T: ventricular depolarization AV nodal cells an become pacemakers when conduction between the SA and AV nodes is impaired. This can occur in complete (3rd degree) AV block , in which SA node impulses cause atrial contraction while impulses generated by the AV node cause ventricular contraction. on ECG, the atria and ventricles depolarizes independently of each other (AV dissociation). QRS complexes are narrow since ventricular depol proceeds normally. The AV node produces HR of 45-55 bpm. a. when electrical impulses are generated by the SA node, the HR is normal. The ECG shows P waves synchronized with each QRS complex and normal ventricular depol c,d,e. When electrical impulses are generated below the AV node and His bundle, the HR can slow to as few as 20 ppm. The ECG also shows prolonged, abnormal shaped QRS complexes d/t aberrant impulse conduction through the ventricles.

1976 A 67 y.o man has syncopal episode. ECG shows bradycardia with regular rhythm and narrow QRS complexes. However, there is complete desynchronization between the Pwaves and QRS complexes. Which of the following locations is most likely responsible for pacing this patient's ventricles? a. Sa node b. AV node c. left bundle branch d. purkinje system e. left ventricular muscle

HIV is an RNA virus that coverts RNA genome into cDNA with the help of a viral RT. Once in cDNA, the viral genome can be incorporated into the host cell genome by viral integrase. The HIV cDNA genome is referred to as a provirus once it has been integrated into the host cell genome. Zidovudine (AZT) is a nucleoside RTase inhibitor used to treat HIV infection. It competitively binds to RTase and is incorporated into the viral genome as a thymidine analog. However, AZT has an aside group in place of the Oh group normally found on the 3' end of thymidine. Because of a free 3' hydroxyl group is required for new nucleotides to be added to replicating DNA, the aside group on AZT prevents DNA chain elongation. a. inhibition of nucleoside phosphorylation is not an effect of AZT. In fact, nucleoside reverse transcriptase inhibitors (including AZT) must be phosphorylated within the host cell to form the functional triphosphate moiety. Non-nucleoside reverse transcriptase inhibitors can allosterically bind to and inactivate reverse transcriptase without being phosphorylated. b. the enzyme thymidylate synthase catalyzes the conversion of deoxyuridine monophosphate (dUMP) to deoxythymidine monophosphate (dTMP). N5, N10-methylene THF is methyl donor in this reaction and is converted to DHF in the process. THF is then regenerated from DHF by dihydrofolate reductase. Methotrexate is a chemotherapeutic agent the inhibits DNA synthesis by inhibiting DHF reductase (i.e., folate dependent uracil methylation). d. in dsDNA, A goes with T and G goes C. These bases bind via H-bond. Some chemotherapeutic agents, like the anthracyclines daunorubicin and doxorubicin, intercalate between the bases, causing defective base pairing and splitting of the DNA strands. Transcription and replication failure and mutagenesis ultimately result. e. integration of viral DNA into the host genome is mediated by integrate, a retroviral enzyme that is inhibited by *raltegravir* an integrate inhibitor.

2019 AIDS patient was prescribed zidovudine, which of the following components of viral genome replication does this drug messes with? a. nucleoside phosphorylation b. folate dependent uracil methylation c. e'-5' phsphodiester bond formation d. proper base pairing and hydrogen bond formation e. integration of viral DNA into the host genome

Primary structures: covalent peptide bonds. Secondary structures (alpha helix or beta sheet) from subsequent H-bond. Tertiary: single polypeptide chain assumes following compact folding of the secondary structure. Many forces combine to stabilize the tertiary structure including: ionic, hydrophobic, hydrogen and disulfide bonds. Keep in mind the disulfide bonds are very strong covalent bonds between two cysteine residues within the same polypeptide chain that enhance a protein's ability to withstand denaturation. In Alzheimer, beta amyloid protein loses its alpha-helical configuration and forms beta-sheets, which are less soluble and therefore prone to aggregating. Aggregations of beta sheets are primary component of the extracellular senile (neuritic) plaques found in Alzheimer patients. The conversion of alpha helices to beta sheets involves the breaking and reforming of hydrogen bonds.

2026 A group of researchers is studying the pathogenesis of Alzheimer disease. A protein isolated from the brain has cofonrmation consisting of mostly beta-pleated sheets. A sample of new medication is applied to the protein, and the prevailing structure changes to an alpha-helical structure. The conformation is result of reorganization of which of the following? a. ionic b. hydrophobic c. peptide bonds d. H bonds e. disulfide bonds

Hb electrophoresis is used to determine if the patient's new husband also has sickle cell trait. Abnormal Hb moves at a slower speed than normal d/t to the replacement of the glutamic acid by valine. The husband's test result combines with the maternal family history will determine the risk that future offspring will inherit sickle cell anemia a. chorionic villi are part of the placenta and as such are fetal tissue. Once fetal DNA is obtained, DNA sequencing can be performed to evaluate for sickle cell mutation. This can be performed at 10-14 weeks gestation. However, it is relatively invasive and carries a risk of miscarriage, spotting/bleeding, infection, and other complications. CVS is not helpful to a patient who wishes to determine her risk prior to conception b. it is not necessary to perform maternal Hb electrophoresis as it has been establish from this patient's history that she has sickle cell trait c. Northern blot use electrophoresis to separate RNA by size, followed by the application of a DNA probe to detect a specific RNA molecule. This allows for the quantification of the RNA expression of specific genes. Hemoglobin electrophoresis is a more appropriate test for SCD as it involves the separation of protein (eg, hemoglobin) rather than RNA e. Karyotype is a visual analysis of all the chromosome in a cell ordered by size and stained to reveal chromosomal banding. SCD results form a single nucleotide mutation, not form an abnormality in overall chromosome structure. Disorders d/t chromosomal abnormalities include Klinefelter syndrome (47, XXY) and Turner syndrome (45, XO) and trisomy disorder.

2040 A 24 y.o African American woman comes to the office with her husband for prenatal counseling. She has a 3 y.o son that has sickle cell anemia from a previous marriage. She and her husband were worried about their future children having sickle cell anemia. A urine pregnancy test is negative. Which of the following is the best initial test that can be offered to this couple? a. chorionic villous sampling during future pregnancy b. maternal hemoglobin electrophoresis c. northern blot analysis of paternal blood sample d. paternal hemoglobin electrophoresis. e. paternal karyotype analysis

Schizoaffective: mania and psychotic features together, and a lifetime history of at least 2 weeks of psychotic symptoms without significant mood disturbance is the key requirement. Differentiating schizoaffective disorder from bipolar disorder or MDD with psychotic features requires determine the temporal relationship between psychotic symptoms and mood symptoms. In bipolar disorder and MDD with psychotic features, the psychotic symptoms occur exclusively during manic or depressive episodes; when the patient's mood is euthymic, there are no psychotic symptoms. Schizophrenia has no mood symptoms Schizophreniform last over one month but no more than 6.

2047 A patient has psychotic and manic episodes. During his last admission he was hospitalized 3 weeks for hearing voices an beleiving that he was being monitored by secret cameras. At that time, no other mood symptoms except for being scared where found. Which of the following is the most likely diagnosis? a. bipolar disorder with psychotic feature b. delusional disorder c. MDD with psychotic features d. schizoaffective e. schizophrenia f. schizophreniform disorder

This patient was given an SSRI. History suggests an intentional overdose, and she has serotonin syndrome that gives the following clinical features: -neuromuscular excitation (hyperreflexia, clonus, myoclonus, rigidity and terror) -autonomic stimulation (hyperthermia, tachycardia, diaphoresis and vomiting/diarrhea) -altered mental status (agitation and confusion) This syndrome most commonly occurs when SSRIs are given with other serotonergic agents such as MAOIs or triptans. It may also occur with a single agent if an excess dose is taken. However, a multitude of other meds have been implicated in precipating serotonin syndrome, and it may be difficult to obtain an adequate medicate history in patients with altered mental status. Therefore, it is important to maintain a high index of suspicion. TRYPTOPHAN is a precursor of serotonin, and metabolism occurs by the enzymes TRYPTOPHAN HYDROXYLASE and AMINO ACID DECARBOXYLASE a. Glutamic acid is the precursor for GABA b. histidine is precursor for histamine that plays a role in allergic (atopic) reaction c. methionine is a precursor or intermediate in the synthesis of cysteine, carnitine, taurine and lecithin. d. tyrosine is precursor for thyroxine, dopamine, epinephrine, NE and melanin. TREATMENTs: -supportive care including airway maintenance, hydration and temp maintenance. -Pharmacologic therapy: serotonin receptor antagonists such as CYPROPHEPTADINE, which is the first generation histamine antagonist with nonspecific 5-HT, and 5-HT2 receptor antagonistic properties a. short-acting antiHTNs (eg, esmolol, nitroprusside) can be used to treat HTN associated with serotonin syndrome. Antihypertensive agents with longer half-lives, such as propranolol, should be avoid d/t the risk of developing hypotension and shock b. haloperidolis a anti-dopaminergic agent used primarily for treatment of psychosis. An adverse reaction associated with antipsychotic meds is neuroleptic malignant syndrome, which presents with symptoms similar to serotonin syndrome c. Flumazenil is th antidote for benzo overdose, not serotonin syndrome. Bento themselves are frequently used for treating serotonin syndrome, and can help relieve agitation as well as mild reduce HR and BP e. Naloxone is the antidote for narcotic overdose

2088/2089 a 32 year old woman is feeling sad for the past three months and often finds herself crying for no obvious reason. Her sleep has been fragmented, and that she wakes up early in the morning and then has difficulty falling back to sleep. She has decreased appetite and been experiencing weight loss. She denies chest pain, abdominal pain, headaches, palpitations or excessive sweating. Labs for TSH is normal. She started on first line pharmacologic treatment for her condition. Two days later, she is brought to the hospital after her sister found her to be agitated and confused. The sister had rushed to the patient's home after she received a voicemail in which the patient seemed particularly sad, and while there, she found an empty pill bottle. The patient is tremulous, and has abdominal cramps and diarrhea. Her temp is 102, BP is 210/120 and HR 120. PE shows dilated pupils and diaphoretic. Neurologic examination reveals bilateral hyperreflexia and inducible ankle clonus. Increased central nervous system activity caused by derivative of which of the following amino acids is responsible for this patient's condition? a. glutamic acid b. histidine c. methionine d. tyrosine e. tryptophan The patient is admitted to the hospital and supportive therapy is initiated. However, she remains agitated with only limited improvement of her vital signs. Which of the following is the antidote for this patent's condition? a. propranolol b. haloperidol c. flumazenil d. cyproheptadine e. naloxone

Dude has blunt aorta injury, which is most commonly caused by motor vehicle collisions. Mechanical injury involves sudden deceleration that results in extremism stretching and torsional forces affecting the heart and aorta. Injury occurs most often at the *aortic isthmus,* which is tethered by the ligmentum arteriosum and is relatively fixed and immobile compared to the adjacent descending aorta. The majority of patient die form aortic rupture before reaching the hospital. those who survive the intimal injury have nonspecific finding such as chest pain,b ack pain or shortness of breath. A *widened mediastinum* may also be seen on chest X-ray.

2130 a dude was involved in high speed motor vehicle accident. He complains of chest pain, abdominal pain and difficulty breathing. PMH is HTN, asthma, and type 2 DM. BP is 98/54 and HR 121. Dude died. What part of the thoracic aorta is messed up?

Bitch has panhypopituitarism with failure of lactation, central hypothyroidism and adrenal insufficiency. In setting of a recent delivery, this is most likely d/t ischemic necrosis of the pituitary gland (Sheehan syndrome). During pregnancy, the pituitary enlarges d/t estrogen induced hyperplasia of the alctotrophs. However, the blood supply to the pituitary doesn't increase proportionally. As a result, the enlarged pituitary is vulnerable to ischemia in case of systemic hypotension d/t permpartum hemorrhage (which this patient likely experienced given her low hemoglobin). The most common manifestation of Sheehan syndrome is failure of lactation d/t prolactin def. It also commonly causes hypocortisolism and hypothyroidism. Manifestations of thyroid deficiency may take a few weeks to develop d/t the long circulation half-life thyroxine (5-7 days) and peripheral conversation of the T4 into T3. Cortisol def manifests rapidly however, with nausea, postural hypotension , fatigue and weight loss. Pituitary apoplexy is d/t sudden hemorrhage into the pituitary, usually in setting of a preexisting pituitary adenoma. It usually presents with acute severe headache, opthalmoplagia and altered sensorium. Lymphocytic hypphsiitsi is the most common inflammatory ocndiotn of the pituitary and typically occurs during late pregnancy or the early postpartum prediod. In contrast to Sheehan syndrome, the presentation is acute with severe headaches and visual field defects. Primary pituitary cancer is extremely rare, although the pituitary is prone to metastases d/t its rich vascular supply. These patients typically present with tumor mass effects (eg, headache, bitemporal hemianopsia). Non-malignant infiltrative lesions such as sarcoidosis and histiocytosis X daily involve the supresellar region, where they compress the hypothalamus and pituitary stalk. This disrupt the normal hypothalamic dopaminergic suppression of of prolactin secretion, leading to increased prolactin levels and possible galactorrhea. Central diabetes insidious may also be seen with resulting hypernatremia.

220 a 24 y.o woman, gravid 1 para 1, is not lactating. She feels excessive tired and has significant weight loss. She also has nausea, loss of appetite, and postural dizziness. Patient has no headaches, visual problems, cold intolerance, constipation or polyuria. Lab test are at 8 AM shows slow hemoglobin. sodium, cortisol, free T4 and TSH. What is going on? a. glandular hemorrhage b. inflammation c. ischemic necrosis d. malignant infiltration e. non-malignant infiltration

Lab findings: 1. decrease C3, maybe C4 2. increase creatinine 3. increase anti-DNase B and increase AHase 4. increase ASO 5. increase anti-NAD (preceding pharyngitis) PSGN most commonly affects children age 5-12, and over 95% recover completely with anti-hypertensives as well as salt/water restriction and diuretics for edema. However, increased age is the most important poor prognostic factor; only 60% of adult cases resolve complete, whearase the rest develop chronic HTN, recurrent proteinuria, chronic renal insufficiency, or rapid progressive GN. Pre-existing kidney disease is an additional poor prognostic factor. (BEF) ASO titers and low C3 levels are present in the vas majority of PSGN patients. ASO titers are diagnostic of a recent streptococcal infection, most often post-pharyngeal infection. C3 decreased d/t alternative complement pathway activation leading to complement deposition in the glomerulus. Initial urinalysis may show hematuria, proteinuria, and RBC case, but none of the findings affect prognostic (CD) following diagnosis of streptococcal pharyngitis, antibiotic therapy is sued to prevent the nonsuppurative complication of acute RH fever, but it doesn't prevent or treat PSGN. Corticosteroid are used in minimal chain disease (most common cause of nephritic syndrome in children) but not PSGN

25 A few adults and children hadePSGN. None of these patients have pre-existing renal disease. Which of the following patient characteristics is most likely indicate a poor long term prognosis? a. adult onset b. decreased C3 c. delay in antibiotic administration d. delay in corticosteroid administration e. elevated anti-streptolysin titer f. gross hematuria

Levodopa have 2 of the same outcomes in the periphery and brain. 1. Gets metabolized into 3-OMD by Catechol-O-methyl transferase 2. Gets metabolized into dopamine by DOPA decarboxylase These two outcomes are targets for Parkinson treatments. -Entacapone & Tolcapone inhibits Catechol-O-methyl transferase to inhibit the conversion of Levodopa into 3-OMD. The ladder drug (eg, tolcapone) can cross the BBB and work on the COMT in the brain -Carbidopa inhibits DOPA decarboxylase PERIPHERALLY because it cannot cross the BBB. Levodopa is the immediate precursor of dopamine, a NT that is absent in nigrostriatum of patients with Parkinson disease. Dopamine itself cannot be administered directly because it cannot cross the BBB, but levodopa can. It is rapidly metabolized in the peripheral tissues to dopamine (via DOPA decarboxylase) and 3-O-methyldopa (via COMT). Thus, only a small percentage of levodopa reaches the brain. Therefore, levodopa is typically administered with carbidopa, a peripheral DOPA decarboxylase inhibitor, but even then only about 5-10% of levodopa actually reaches the brain Entacapone is COMT inhibitor that increases bioavailiabity of levodopa by inhibiting its peripheral methylation. Combining levodopa with entacapaone helps increase the plasma half-life of levodopa, producing a more stable levodopa plasma concentrations and prolonging the therapeutic effect of each dose. COMT inhibitors are currently used to treat Parkinson disease patients with motor fluctuations who are experiencing end of dose "wearing off" periods with levdopa/carbidopa therapy, as they are otherwise ineffective. b. dopamine agonist (eg, bromocriptine, pramipexole, ropinirole) preferentially stimulate dopa 2 receptors c. amantadine likely enhances the effect of endogenous dopamine by increasing dopamine synthesis/relase and inhibiting the reuptake of dopamine d. anticholinergics such as trihexyphenidyl and benztropine inhibit central muscarinic receptors. these drugs are mainly used for drug induced parkinsonism, but patients with tremor as the predominant symptom also respond well. e. Acetylcholinesterase inhibitors (eg, donepezil, rivastigmine) increase the amount of acetylcholine in the CNS. These drugs are commonly used in patients with dementia to increase cholinergic activity in the prefrontal cortex. However, they should be used cautiously in patients with Parkinson disease as higher levels of Ach in the basal ganglia can exacerbate symptoms.

265 a 72 y.o man has Parkinson. He has been taking levodopa/carbidopa for 5 years. He is currently on a maximum dose, but for the past several months has experienced bothersome motor fluctuations. The patient also presorts worsening stiffness and difficulty moving before his next scheduled dose, which improves after taking the medications. Entacapone is added to his treatment regimen. This drug most likely improves this patients symptoms by which of the following mechanism? a. decreasing peripheral levodopa degradation b. directly stimulating dopa receptors c. enhancing the effect of endogenous dopamine d. inhibits central muscarinic receptors e. inhibiting centrally active acetylcholinesterase

Kid has PSGN. LM: enlarge and hyper cellular glomeruli IF: granular deposits of IgG, IgM an dC3 along the basement membrane and in the mesangium of glomeruli (lumpy-bumpy appearance) EM: sup epithelial hump of immune complexes. A. albumin may be lost in urine d/t increase permeability of the glomerular capillary wall in PSGN. However, albumin does't deposit within the glomerulus or renal tubules b. T-lymp, monocyte and eosinophilic infiltration may be seen in acute interstitial nephritic secondary to some antibiotics (eg, penicillins, cephalosporins). However, AIN rarely causes HTN, edema or hematuria and usually resolve when the offending agent is discontinued c. fibrin deposition = rapidly progressive glomerulonephritis d. hyaline, acellular deposits composed of plasma proteins, can be seen in Kimmelstiel-Wilson nodules of diabetic nephropathy f. lipid droplets in renal tubules may be seen in conditions causing nephrotic syndrome; however, this is uncommon in PSGN g. neutrophils and monocytes infiltrate the glomerular mesangium in PSGN, contributing to the hyper cellular appearance on light microscopy. Enzymatic granules are small intracellular structures that do not appear homogeneous on EM

27 Young patient has facial and periorbital edema. Moderate HTN and hematuria. Sub epithelial hump in the glomerulus. What is inside of it? a. albumin leak b. eosinophil enzymes c. fibrin d. Hyaline accumulation e. immune complex deposits f. lipid droplet g. neutrophil enzymes

anti fungal targets 1. Griseofulvin: mitosis 2. DNA and RNA synthesis: Pyrimidines (eg, flucytosine) the only agent in this calls of anti fungal, is converted to 5-fluorouracil within the fungal cell wall and interferes with fungalRNA and protein synthesis. 3 Cell wall: echinocandins (eg, caspofungin, micafungin) inhbit the synthesis of glucan, a component of the fungal cell wall 4. Cell membrane: amphotericin B & nystatin bind ergosterol (polyenes) in fungal cell membrane,s creating pores and causing cell lysis (via leakage of K). Azoles (eg, ketoconazole, fluconazole, itraconazole, voriconazole) inhibit synthesis of of ergosterol. Of these agents, only polyenes (amphotericin B, nystatin) depend on the amount of ergosterol incorporated into fungal cell membrane in order for them to work. An organism that decreases the amount of ergosterol in its cell membrane would likely become resistant to polyenes. -amphotericin B is a systemic anti fungal active against Candidia, Aspergillus, Cryptococcus, Histoplasma, Blastomyces and Coccidiodes. -nystatin is used topically; oral candidiasis is the main indication for its use ("swish and swallow") cyclosporin is an immunosuppressant used in transplant recipients to prevent rejection. It decreases the synthesis of IL-2, thereby inhaling T cell proliferation. Flu cytosine is used in combination with amphoteric B primarily for the treatment of cryptococcal meningitis in patients with HIV> It is a nucleotide analog that competitively inhibits RNA synthesis in fungal cells. Griseofulvin interacts with fungal cell microtubules, inhibit mitosis. It accumulates in keratin-containing tissues and is used to treat dermatophyte infection (eg, microsporum, epidermophyton and trichophyton)

271 Investigation reveals that the mechanism of resistance to drug X by candida was a decrease in ergosterol incorporation into the cell membrane. Base on this date, drug X is likely to be most closely related which of the following anti fungal agents? a. caspofungin b. cyclosporin c. flucytosine d. griseofulvin e. nystatin

This patient has CREST syndrome: Calcinosis Raynaud phenomenon Esophageal dysmotility Sclerodactyly Telangiectasia aka spider angioma is a limited variant of systemic sclerosis with skin disease that primarily affects the face, forearms and fingers. The pathogenesis of systemic sclerosis involves chronic autoimmune inflammation, vascular endothelial injury resulting in chronic ischemic tissue damage (eg, fingertip ulcers), and excessive activation of fibroblasts leading to progressive tissue fibrosis. Esophageal dysmotility is a result of atrophy and fibrous replacement of the muscular in the lower esophagus. The esophageal body and the lower esophageal sphincter become atonic and dilated, resulting symptoms of GERD (eg, heartburn, regard and dysphagia). This increase the risk of Barrett's esophagus and fibrous stricture formation. a. sliding hiatal hernia is characterized by herniation of gastroesophageal junction and a portion of the stomach through the esophageal hiatus, which predisposes patients to GI reflux. c. increased gastric acid production occurs with gastronomes, gastrin secretin gneuroendocirne tumors of the small intestine/pancreas that often present with multiple or medically refractory peptic ulcers. d. Achalasia is an esophageal motility disorder characterized by failed relaxation of the lower esophageal sphincter resulting in food retain, dilation of the esophageal body and symptoms of solid liquid dysphagia e. diffuse esophageal spasm is characterized by periodic, uncoordinated, simultaneous contractions of the lower esophagus d/t impaired inhibitory innervation within the esophageal myenteric plexus. Patients typically have liquid/solid dysphagia and chest pain d/t inefficient propagation of food into the stomach.

284 A 35 y.o woman has heartburn that is resistant to over the counter antacids. PE shows scattered telangiectasia on the face, several ulcers at the tips of the fingers and small calcium deposits in the soft tissues of the hands and elbows. Which of the following processes is the most likely cause of this patient's heartburn? a. abnormal location of gastroesophageal junction b. fibrous replacement of the muscular in the lower esophagus c. increased gastric acid production d. increase in resting lower esophageal sphincter tone e. uncoordinated, simultaneous muscle contractions in the lower esophagus.

Patient's valve is d/t dystrophic calcification affecting an aging aortic valve. Dystrophic calcification is considered a hallmark of cell injury an death, occurring in all types of necrosis (eg, coagulative, fat, caseous, liquefactive) in the setting of normal calcium levels. Grossly, dystrophic calcium deposits are seen as fine, gritty, white granules or clumps. On hematoxylin and eosin staining, these deposits typically appear as dark-purple, sharp-edged aggregates. Deposits that develop lamellate outer layers are described as psammoma bodies. Dystrophic calcification in aged (or damaged) cardiac valves is thought to be the result of endothelial and fibroblast death secondary to chronic hemodynamic stress ( can be accentuated by valvular abnormalities) or atherosclerotic inflammation. Subsequent release of cellular degradation products into the valvular interstitial then promotes calcification and thickening of the valve leaflets and annulus. These changes are often benign in elderly adults (aortic sclerosis); however, overtime, progressive valvular stiffening can lead to outflow obstruction (calcific aortic stenosis). b. extracellular amyloid deposition occurs in patients with amyloidosis. Amyloidosis can cause restrictive cardiomyopathy and HF, but it is not directly associated with calcification of the aortic valve. c. hypercalcemia is associated with metastatic calcification affecting normal tissues and organs. IN this process, calcium deposition typically occurs in more alkaline tissues involved in acid excretion, such as kidneys, lungs, systemic arteries and gastric mucosa. d. Intracellular hemosiderin accumulations common in patients who have hemolytic anemia or who undergo frequent blood transfusions. It is not directly associated with calcification of the aortic valve. Pathologic cell hypertrophy of left ventricular cardiomyocytes (i.e., left ventricular hypertrophy) can occur as the result of severe aortic stenosis.

296 A 75 y.o man is in a high speed motor vehicle accident and died. Everything else was normal before. At autopsy, heavy calcifications of the aortic valve are seen. Which of the following most likely preceded the aortic valve changes observed in this patient? a. cell necrosis b. extracellular amyloid deposition c. hypercalcemia d. intracellular hemosiderin accumulation e. pathologic cell hypertrophy

4 things you check: AFP, estriol, beta-hCG and inhibin A. -Things that have low AFP is trisomy. -Things have high AFP are neural tube defect, ventral wall defect and multiple gestation. AFP is synthesized by the fetal liver, GIT and yolk sac (early gestation only), and levels increase with gestational age. For this reason, accreting dating is very important in determining the risk status. In this patient, the gestational age is likely to be incorrect d/t to the patient's history of irregular menses. Women who have irregular periods ovulate less predictably. The next step should be an ultrasound to estimate gestational age. b/e: Fetal neural tube defect and abdominal wall defect are associated with high AFP. Tobacco use can inc erase the risk of fetal growth restriction and placental insufficiency. Estriol levels would be low, not AFP. The risk of fetal heart defects is increased d/t DM and FH of major congenital heart disease. Markedly high beta-hCG are seen in hydatidiform mole and other gestational trophoblastic diseases. Patients may present with vaginal bleeding and a significantly enlarged uterus.

324 A 28 y.o, gravida 1/para 0, says her fetus haven't moved. Age of menarche occurred at 13. Periods last 3-4 days, and has been irregular. She estimated her gestational age is about 16 weeks. PMH is significant for type 1 DM. She smokes a pack a day. A second trimester maternal serum quadruple screen is performed and shows a decreased AFP level. Which of the following is the most likely etiology for the decreased in AFP? a. dating error b. fetal abdominal wall defect c. fetal growth restriction d. fetal heart defect e. fetal neural tube defect f. hydatidiform mole.

Ectopic pregnancy occurs when a fertilized ovum implants outside of the uterus. Most common site is the ampulla of the fallopian tube, which appear as an adnexal mass on ultrasound. Ask factors include tubal pathology such as previous infection or surgery (eg, tubal ligation). Pregnancy after permanent sterilization is extremely rare, but one third cases are ectopic if implantation occurs An ectopic pregnancy may become life threatening as the embryo and trophoblastic tissue proliferate. This growth will compromise the blood supply to the surrounding tissues, which can result in rupture and profuse intra abdominal bleeding. A ruptured ectopic pregnancy is managed surgically by removing the pregnancy an achieving homeostasis. Dilation and curettage of the uterus may be performed either to stop uterine bleeding or confirm whether the pregnancy is intrauterine or ectopic. IN an ectopic pregnancy, the uterine specimen will reveal *decidualized endometrium ONLY, consistent with dilated coiled endometrial glands and vascularized edematous storm.* these changes occur in the luteal phase of the menstrual cycle, under influence of progesterone, as the endometrium prepares for implantation. Embryonic trophoblastic tissue will e absent from the uterus. The finding of atypical endometrial cells that form glands would suggest endometrial adenocarcinoma, a malignancy that typically occurs in postmenopausal women and manifests with vaginal bleeding. A molar pregnancy or spontaneous aboriton may present with vaginal bleeding. Uterine curettage may show enlarged chorionic villi and avascular edematous storm. IN contrast, intrauterine chorionic villi are absent in ectopic pregnancy. An inflammatory endometrial infiltrate would suggest endometritis, an infection of the decidua, which presents with uterine tenderness, fever, and tachycardia. An ectopic pregnancy is not an infectious process Straight, short endometrial glands and compact storm are found in the early proliferative phase of the menstrual cycle. This microscopic appearance would be observed 4-7 days after the onset of menses.

334 A 28 y.o woman, gravid 3 para 3, comes to the emergency department with severe abdominal pain in the left lower quadrant and vaginal bleeding. Past surgical history is significant for 3 cesarean deliveries and permanent sterilization via a bilateral tubal ligation. A ruing pregnancy test is positive, and ultrasound shows a 2 cm mass in the left adnexa adjacent to the ovary and thickened endometrial stripe. If a uterine curettage is performed, which of the following findings would be expected? a. atypical endometrial cells, disorganized glands and multiple mitoses b. dilated, coiled endometrial glands and edematous storm c. enlarged chorionic villi and avascular edematous storm d. inflammatory infiltration of endometrial glands e. straight,s hoo endometrial glands and compact storm.

Patient is having pancytopenia secondary to treatment of her bacterial meningitis with chloramphenicol. Chloramphenicol suppresses bacterial protein synthesis by binding to the ribosomal 50S subunit and inhibits the peptide transferase enzyme. Adverse effects: dose related anemia, leukopenia and/or thrombocytopenia that are reversible by withdrawing the medication. -It also causes dose independent (idiosyncratic) aplastic anemia, which is usually severe and fatal without treatment (eg, bone marrow transplantation). Clindamycin is active against anaerobic gram negative rods (eg, bactericides) and aerobic gram positive cocci. Most significant side effect is pseudomembranous colitis. Side effects of gentamicin are vestibular and cochlear ototoxicity, nephrotoxicity and neuromuscular paralysis (with large dose and intrapleural administration) The common side effect of metronidazole are GI (eg, nausea, vomiting, cramps and abdominal pain) and neurologic (paresthesias, dizziness). When taken with alcohol, metronidazole induces a disulfiram like reaction Rapid administration of vancomycin can cause histamine release, which rustles in flushing (red man syndrome). this antibiotic may also cause dose released nephrotoxicity and ototoxicity.

359 A 6 y.o has fever, headache, and confusion. She had a severe allergic reaction to penicillin that required hospitalization 3 years ago. She started on the appropriate antibiotics and recovers. Two weeks later the kid has pancytopenia. What was she treated with? a. chloramphenicol b. clindamycin c. gentamicin d. metronidazole e. vancomycin

retroperitoneal abdominal organs can be remembered via the mnemonic: SAD PUCKER Suprarenal (adrenal) glands Aorta and inferior vena cava Duodenum* (except first part) Pancreas* (head and body) Ureters and bladder Colon* (ascending and descending) Kidneys Esophagus Rectum (mid-distal) Retroperitoneal hematoma are commonly associated with abdominal and pelvic trauma. Pancreatic injury is a frequent cause of retroperitoneal hematoma. It can occur following severe blunt or penetrating abdominal trauma, such as a malpositioned seat belts or steering wheels during accident. Pancreatic injury with retroperitoneal hematoma may initially cause only mild symptoms or be asymptomatic, but it can lead to life threatening blood loss. Therefore an abdominal CT scan is frequently performed in patients with blunt abdominal trauma to rule out retroperitoneal hematoma. d. although the abdominal aorta is a retroperitoneal structure, rupture would most likely result in massive and rapid blood loss leading to hemorrhagic shock. All other options are not in sad pucker

838 Patient was in an accident. BP 115/55 and HR 96. CT scan shows retroperitoneal hematoma. This patient most likely experienced which of the following injuries? a. contusion of the body of the pancreas b. laceration of the inferior border of the spleen. c. laceration of the right hepatic love d. rupture of the suprarenal aorta e. tear at the lesser curvature of the stomach f. tear at the proximal transverse colon

Hep B infection progresses through two phases 1. proliferative: the entire vision and all related antigens of the episomal HBV DNA are present. ON the hepatocyte cell surface, viral HBsAg and HBcAGg are expressed in conjunction with MHC class I molecules. This expression serves to activate the cytotoxic CD 8 T lymph, which respond by destroying the infected hepatocytes. Note that the vision itself doesn't have a cytopathic effect 2. integrative : the HBV DNA is incorporated into the host genome of those hepatocytes that survived the immune response. Infectivity ceases and liver damage tapers off when the antiviral antibodies appear and viral replications stops. The risk of hepatocellular carcinoma, however, remains elevated because ofHBV dan that has been integrated into the host genome a. HBV has no direct cytotoxic effect b. host antibody HBsAb neutralizes HBV infecitivty by binding to the viral surface antigens HBsAg and preventing the antigen from interacting with hepatocyte receptors. Note that this neutralization occurs before the virus enter the hepatocyte, and therefore wouldn't be associated with hepatocellular damage. c. antigen antibody complexes cause some of the early symptoms of HBV infections (eg, arthralgia, arthritis and urticaria) as well as some of the choir complications (eg, immune complex glomerulonephritis, cryoglobulinmeia, vasculitis). These complexes are not responsible for hepatocellular damage. d. the pathogenesis of autoimmune hepatitis not hep B infection ivvloves antigen mimicry with genratio not self antigen receogzign CD 4 T lymph that damage hepatocytes

375 Patient has anorexia and fatigue. HIs ALT and AST are high. He is positive for HBsAg. Which of the following best describes the mechanism of liver injury in this patient? a. viral cytotoxic effect and hepatocyte necrosis with following secondary inflammation. b. antibody response to viral antigens on the cell surface with antibody-mediated cell damage c. circulating viral antigen antibody complexes with small vessel deposition and immunologic vessel injury. d. antigen mimicry with generation of self antigen recognizing CD4 t lymphocytes that damage hepatocytes e. CD8 T lymph response to viral antigens on the cell surface resulting in hepatocyte damage

Dude has AR inherited Wilson disease Normally about 60% of ingested copper is absorbed in the stomach and duodenum, loosely bound to circulating albumin and then transported to the liver. Within the liver, copper is incorporated into an alpha-2-globulin to form ceruloplasmin. The ceruloplasmin is then secreted into plasma, where it accounts for 90-95% of circulating copper. Normal total body copper is estimated at 50-150 mg. Senescent ceruloplasmin and the reminder of ingested, unabsorbed copper are secreted into bile and excreted in stool, which are primary route o upper elimination. Renal losses represent 5-15% of daily copper excretion.

396 Brain autopsy reveals extensive atrophy of basal ganglia and increased brain copper content. How does the body remove excess copper in the body? a. glomerular filtration b. renal tubular secretion c. hepatic excretion into bile d. intestinal epithelial accumulation e. colonic active secretion

Most colon adenocarcinomas develop from preexisting adenomatous polyps (adenomas). Colon adenomas usually occur in patients 50-0 y.o and are considered premalignant. It is hypothesized that 10 years on average, is required for malignant transformation of adenoma. Early detection and decision of adenomatous polyps is, therefore, an effective prophylaxis for colon adenocarcinoma. The malignant potential of adenomatous polyps determined by the following: 1. size of the polyp: <1cm - unlikely to undergo malignant transformation; >4cm = 40% risk of malignancy 2. histologic appearance: villous adenomas are more prone to be malignant than tubular adenomas 3. degree of dysplasia. The transformation of normal mucosal cells into malignant ones is caused by a series of gene mutations called the adenoma to carcinoma sequence: 1. progression from normal mucosa to small polyp. The intimal appearance of small adenomatous polyps is attributed to mutation of the APC tumor suppressor gene 2. increase the size of the polyps. Mutation of K-ras protooncognee is thought to facilitate this sep by leading to uncontrolled cell proliferation. 3. Malignant transformation of adenoma into carcinoma requires mutation of two genes: p53 and DCC. Increase the size of adenomatous polyps (and therefore, increase in their malignant potential) is attributed to K-ras protoconcogene mutation. Tai gene normally encodes for a protein that regulates cell cycle by stimulating and inhibiting it as necessary. Mutation of K-ras cauaews it to become the oncogene K-ras, which encodes for a protein that has lost its ability to inhibit the cell cycle, but can still stimulate it. Thus, unregulated cell proliferation ensues.

421 Describe the progression of adenocarcinoma in a stepwise accumulation of various mutations.

Midline fusion of neural folds creates the neural tube during the 3rd week of fetal life. The neural tube is a structure that is connected to the amniotic cavity by opening at the ends known as anterior and posterior neuropores. Failure of neuropore closure by 4 weeks gestation leads to neural tube defects (NTDs). -Anterior NTDs include anencephaly (complete absence of the brain) and encephalocele (protrusion of neural tissue through the cranial defect). -Posterior NTDs include 1. spina bifida occult 2. meningocele - present as cystic mass at the lower spine that is covered with skin and sometimes a tuft of hair. 3. meningomyelocele - is a cystic mass where portions of the spinal cord and caudal equine within the protruding meningeal sac. Folate (B9) supplementation reduces risk of NTDs and is recommended for all women of childbearing age because the neural tube develop during the time when most women are unaware of their pregnancy. Prenatal valproate use for bipolar or epilepsy is associated with a 10 to 20 fold increased risk of NTDs d/t impaired folate metabolism. Other risk factors include maternal use of folate antagonists (eg, methotrexate, TMP-SMX) a. antenatal steroid therapy (eg, betamethasone) reduces the incidence of neonatal respiratory distress syndrome in premature infants. b. Although chronic alcohol consumption can result in folate consumption (and therefore NTDs), it is more commonly associated with fetal alcohol syndrome. However, fetal alcohol syndrome is unlikely in the absence of short palpebral fissure, a thin vermilion border and smooth philtrum. c. Lithium an valproate are first line drugs for acute mania. If valproate cannot be substituted by another drug to control a mood or epilepsy disorder, the lowest effective dose should be prescribed during pregnancy. Lithium use in pregnancy is also not recommended d/t toits association with increased incidence of Ebstein anomaly (ASD, atrialized ventricles and messed up valves) d/e. Uncontrolled HTN and tobacco use can cause fetal growth restriction, premature delivery and placental abruption. Tobacco use, HTN and antiHTN medicates are not associated with NTDs

496 Please describe neural tube formation. If a patient with bipolar and chronic HTN has a baby with neural tube defect. Which of the following interventions in the mother could have most likely prevented this neonate's abnormalities? a. steroid therapy b. decreased alcohol consumption c. drugs that treat bipolar d. lowering dose of HTN e. smoking cessation f. vitamin supplementation

GLUT-1: contributes to basal glucose transport in erythrocytes and at the BBB GLUT-2: presents in hepatocytes, pancreatic B cells, and the basolateral membrane of renal tubules and small intestinal mucosa. It plays a role in absorption of dietary glucose, reabsorption of glucose from the renal tubules, hepatic glucose regulation and sensitivity of beta cells to circulating glucose GLUT-3: involved in placental and neuronal glucose transport GLUT-5: fructose transporter found in the spermatocytes and GIT tract ONLY GLUT-4 is repsonvie to insulin, found predominantly in skeletal muscle cells and adipocytes.

847 Describe all of the GLUT, especially where they are located and their affects by insulin.

Clozapine indications: Treatment-resistant schizophrenia. Schizophrenia associated with suicidal tendency. Adverse effects: -Agranulocytosis -Seizures -Myocarditis -Metabolic syndrome The second generation antipsychotic clozapine is the only antipsychotic that has consistently shown superior efficacy in treatment resistant schizophrenia and schizophrenia associated with persistent suicidality. Clozapine has affinity for multiple dopamine and serotonin receptors, but the precise pharmacological mech responsible for its superiority is unknown. Clozapine binds loosely and transiently to D2 receptors, causing significant fewer extrapyramidal symptoms compared to first generation antipsychotics. NEUTROPENIA and the potential for life threatening agrunolcytosis are the major adverse effects of clozapine, which happens in about 1% of patients. Thus guidelines require enrollment in a centralized program that regularly monitors the patient's absolute neutrophil count. Treatment should be interrupted if neutropenia is suspected to be clozapine induced and the absolute neutrophil count is <1000. Seizures, myocarditis and metabolic syndrome are other important adverse effects that require provider vigilance. Clozapine plasma levels can be checked after an initial target dose is reached, but further dosage adjustments are usually based on clinical response. Clozapine levels are not regularly monitored. c/g: Thyroid function tests and creatinine should be monitored in patients taking lithium d/t this drug's potential to adversely affecting the thyroid and kidney. d. although physicians should be alert to the development of CVS symptoms suggestive of myocarditis, routine ECGs are not required. Among the second generation antipsychotic, ziprasidone is most often noted for causing a prolonged QT interval. e. LFT may be mild elevated with use of many psychotropic medications, inclusion antipsychotics and anticonvulsants; however, routine monitoring of LFT are not required with clozapine. Prolactin levels are not routinely monitored. Among the second generation antipsychotics, risperidone has been associated with a greater risk of prolactin elevation.

516 Patient is schizophrenic and suicidal that are refractory to treatments by haloperidol, risperidone and olanzapine. What do you give her and what should be regularly monitored? a. absolute neutrophil count b. clozapine blood level c. creatinine d. ECG e. Liver function test f. prolactin g. thyroid function tests

This patients weight gain, dry skin, hair loss and constipation accompanied by bradycardia are most likely d/t hypothyroidism. Lithium, a mood stabilizer used in treatment of bipolar disorder and unipolar depression ,interferes with normal thyroid functioning through multiple mechanisms, including decreasing thyroid hormone synthesis and release. Hypothyroidism and goiter are the most common thyroid abnormalities in lithium treated patients. Periodic measurement of the serum TSH level is recommended for patients on long term lithium therapy. When hypothyroidism develops, it can be treated with T4 and doesn't necessarily require lithium discontinuation. Long term lithium therapy is also associated with adverse effects on renal function (nephrogenic diabetes insidious and less frequently, chronic tubulointerstitial neprhopathy), requiring regular monitoring of blood lithium levels and renal function (i.e., BUN and creatine). Lithium use during pregnancy has ben associated with Ebstein anomaly of the tricuspid valve, but the absolute risk is small. a. carbamazepine is used to treat manic episodes and maintenance. It can also be use to treat partial and generalized seizures as well as its most famous indication TRIGEMINAL NEURALGIA. Side effects: agranulocytosis, hyponatremia (SIADH) and neural tube defects. b. citalopram is an SSRI used as an antidepressant. side effects is sexual dysfunction. c. clozapine is a second generation antipsychotic indicated for treatment-refractory schizophrenia. It is associated with agranulocytosis that requires regular monitoring of the absolute neutrophil count. Although clozapine can cause weight gain and constipation, it is not an initial treatment for mood disorders and would not explain all of this patient's symptoms (eg, dry skin and hair loss) d. lamotrgine is indicated for depressive episodes, maintenance. Focal seizure. serious side effect is the deadly steven johnson syndrome. f. risperidone is second generation antipsychotic that can cause weight gain and symptoms of hyperprolactinemia (eg, galactorrhea, amenorrhea). It is not associated with hypothyroidism. g. Trazodone, an antidepressant, is most commonly sued to treat depression related insomnia du/t its sedating effects. It is associate wth the are but serious side effects of priapism (prolonged painful erection). h. valproate indicated for depressive episodes and their maintenance. Main is absence, generalized tonic clonic and myoclonic seizure. Side effects are hepatoxcity and neural tube defects.

520 A 36 y.o woman has mood swings and sleep problems. Her symptoms responded well to intimal therapy and she has remained on the same regimen for 2 years. However, the patient now has new onset constipation, dry skin and hair loss. She is also concurred about a weight gain of 5 pounds over the last couple of months despite trying to eat healthy, low calorie foods. BP is 110/70 and pulse is 55. Which of the following meds are responsible for her symptoms? a. carbamazepine b. citalopram c. clozapine d. lamotrigine e. lithium f. riperidone g. trazodone h. valproate

HBV and HCV increase the risk of hepatocellular carcinoma (HCC). Active hepatitis infection causes chronic live cell injury and regenerative hyperplasia, which results in an increased number of hepatocyte cell division. This increases the chance for genetic mutations that can induce malignant transformation. Infectivity ceases and liver damage tapers off when the host's antiviral antibodies appear and viral replication stops. However, in HBV infection, the risk of HCC remains elevated d/t the persistence of HBV DNA in host genome. This allows continued transcription of oncogenic viral proteins, such as Hex protein. This protein disrupts cell cycle control by activating multiple growth promoting genes and activating the p53 tumor suppressor protein. abcf) C. trachoma's, CMV, EBV and HPV do not cause cancer through the incorporation of their DNA into the hepatocyte genome e) similar to HBV, HCV also causes chronic liver cell injury and regeneration that can predispose to malignancy. However, HCV is an RNA virus that lacks reverse transcriptase and is therefore unable to integrate into the host genome.

58 Postmortem liver has masses that have hepatocytes with foreign DNA fragments integrated into their genome. These fragment most likely belong to which of the following organisms? a. chlamydia trachoma's b. cytomegalovirus c. epstein barr virus d. hep b e. hep c f. HPV

Aldosterone is a steroid hormone. In HF, aldosterone is produced in the myocardium and acts locally, leading to fibrosis and myocardial hypertrophy. The resulting cardiac remodeling worsens left ventricular dysfunction in HF patients. Spironolactone is an aldosterone antagonist with mild diuretics effects. By competitively inhibiting aldosterone, spironolactone effectively blocks aldosterone's detrimental cardiac effects. It has a potassium-sparing effects, which can lead to hyperkalemia. It is structurally similar to androgens and exerts an anti-androgenic effect by acting as an androgen receptor antagonist and inhibiting testosterone synthesis. The resulting effects can include unilateral or bilateral gynecomastia, decreased libido, and impotence. c. eplerenone is a new and more selective aldosterone antagonist that produces fewer endocrine side effects. a. carvedilol is a nonselective beta/alpha1 blocker used to treat HF. S/Es are fatigue, hypotension and bradycardia. b. dofetilide is a class III antiarrhthmic used to maintain sinus rhythm in patients with afib. Risks include serious ventricular arrhythmias and conduction disturbances. d/e. hydrazine is a direct vasodilator often used in combination with isosorbide denigrate for treatment of HF. Common side effects include tachycardia and orthostatic hypotension. f/i: metolazone and torsemide are diuretics used to control edema in patients with HF. Neither has a significant antiandrogenic effect. G. Rivaroxaban is a factor Xa inhibitor used to reduce the risk of cardioembolic stroke in patients atrial fibrillation. Excess bleeding is the primary risk.

687 A 72 y.o man is requesting for mammography. He is concerned that he has breast cancer because his right breast has been getting bigger for the past 2 months. In addition, the patient's sister recently developed breast cancer at age 70. PMH is significant for CAD, CHF with LV systolic dysfunction, persistent atrial fibrillation, HTN and type 2 DM> The patient had a myocardial infarction 2 years ago. His extensive medication list is reviews to identify a possible adverse drug affect. Which of the following medications is mot likely responsible for this patients symptoms? a. carvedilol b. dofetilide c. eplerenone d. hydrazine e. isosorbide denigrate f. metolazone g. rivaroxaban h. spironolactone i. torsemide

regulation of RAAs involves 3 major components: 1. macula dense (distal table sodium sensory) 2. intrarenlal baroreceptors 3. beta adrenergic receptors - is mediated through sympathetic stimulation of beta-1 receptors located on that JG cells,w chi stimulate the release of renin beta-blocker inhibit renin release, which in turn reduces the conversion angiotensiongen into angiotensin I and reduces the levels of angiotensin II and aldosterone. This effect on the RAAS is only partially responsible for beta blocker effects on BP, and antihypertensive efficacy of beta blocker weakly correlates with plasma renin levels. beta blockers have no effect on ACE activity, and there for do not effect brady kinin levels. Direct renin inhibitor alskiren blocks the conversion of angiotensinogen to angiotensin I, which leads to reduced level of angiotensin I and II and aldosterone. Palma concentration is increased via suppression of the inhibitory feedback loop. ARBs block the action of angiotensin II on AT1 receptors, which leads to increased levels of renin and angiotens I and II. ARBs have no effect on bradykinin levels. Aldosterone antagonists (spironolactone, eplerenone) compete with aldosterone for the receptor sites in distal tubules. they raise the levels of renin, angiotensin II and I, and aldosterone via inhibitor of negative feedback

692 Patient has essential hypertension. she has followed dietary and exercise recommendation since then. At follow up visit 3 months ago, she reported episodic headaches d/t to migraines. Her BP is 145/90. Antihypertensive therapy with a beta blocker was started d/t its beneficial effect on migrant prophylaxis. Now 3 months later, the patients BP is 12/80. She complaint with her medication and has had no serious averse effect. Which of the following is most likely combination of changes in response to the patients treatment? describe the RAAS, aldosterone and bradykinin level.

Reactive arthritis: preceding infection: Genitourinary infection (Chlamydia). Enteritis (Salmonella, Shigella, Yersinia, Campylobacter, C. diff) Musculoskeletal: asymmetric oligoarthritis, sacroiliiitis enthesitis (tendon) & dactylitis (sausage fingers). Extraarticular symptoms: -Ocular: conjunctivitis, anterior uveitis -Genital: urethritis, cervicitis, prostatitis -Dermal: Keratoderma blennorrhagicum (hyperkeratotic vesicles on the palms and soles) , circinate balanitis (serpiginous annular dermatitis of the glans penis) -Oral ulcers The classic triad of reactive arthritis (ReA) are: 1. urethritis 2. conjunctivitis 3. mono-or oligoarticular arthritis. Seronegative for RA spondyloarthropathy most commonly affects patients age between 20-40 and is associated with HLA-B27. Symptoms often manifest 1-4 weeks following a primary infection. It is caused by an autoimmune reaction initiated by the infecting pathogen. a. esophageal dysmotility is a classic finding in scleroderma b. hyperPTH are associated with the MEN syndromes c. Polymyositis may occur in the setting of dermatomyositis or a separate entity e. tabes dorsalis is a manifestation of tertiary syphilis. Secondary phials causes maculopapular or pustular (but not vesicular) rash that can involve palms and soles; it is generally associated with poly arthritis (mono-arthritis is rare).

743 A 34 y.o dude has painful urination. PE shows watery penile discharge. Urethral swabs obtained from the patient are neg for gonococcal infection. He is treated with appropriately and his symptoms resolve. Two weeks later, he develops acute conjunctivitis, right knee pain, and vesicular rash on his palms and soles. This patient's condition is most likely associated with which of the following> a. esophageal dysmotility b. hyperparathyroidism c. polymyositis d. scroiliiitis e. tabes dorsalis

Henoch-Scholein purport Path: IgA immune complexes-mediated (type III hypersensitivity) vasculitis. Generally follows upper respiratory or minor infections. Clinical manifestation: 1. palpable purpura 2. arhtralgias 3. abdominal pain, intussusceptions 4. renal disease similar to IgA nphropathy (hematuria) Diagnosis: -usually made clinically confirmed by skin biopsy showing IgA deposition in blood vessels. deposition of these complexes in the walls of small vessels and the renal mesangium leads to recruitment of neutrophils and lymphocytes as well as activation of complement via the alternative/lectin pathway. resulting inflammation leads to the organ dysfunction and palpable purport found in HSP The condition is self limited and resolves as the circulating immune complexes clear. Treatment is supportive unless specific complications (eg, intussusception) occur. a. antibody dependent cellular cytotoxicity type II hypersensitive is part of the body's defense against viral and parasitic infections. Antibodies bound to antigens on the surface of infected cells are recognized by th eFc receptors on the effector cells (eg, NK, neutrophils and eosinophils) that then destroy the infected cells by releasing cytolytic granules delayed hypersensitivity type IV are T-cell and macrophage mediated response. They occur in response to Mycobacterium TB infections and in certain allergic reactions, such as contact dermatitis and transplant rejection a palpable skin rash is commonly seen with disseminated Nessiera infections (miningococcemia or disseminated gonococcus). Unlike the purport of HSP, which is generally limited to the lower extremities, the rash of disseminated Neisseria begins with petechiae on the trunk and spreads over the entire body. patients also have a fever, hypotension and tachycardia IgE depended degranulation occurs in atopic and anaphylactic reactions type I hypersensitivity. IgE on the surface of mast cells and basophils binds the offering allergen and triggers degranulation with relate of histamine, serotonin and other vasoactive substances.

758 A kid has abdominal pain and athralgias. He had a cough and runny nose lsat week but otherwise has been in good health. PE shows palpable purport over his buttocks and things. Auscultation of the lungs and heart is normal. His abdomen is diffusely tender to palpation without rebound or guarding. Both knees are tender but do no appear warm or swollen. A stool occult blood test is positive. Urinalysis reveal proteinuria, mild hematuria, traces of leukocyte esterase, negative for nitrates and RBC casts. Which of the following mechanism is the most likely underlying cause of this patient's condition? a. antibody dependent cellular cytotoxicity b. circulating immune complexes c. delayed hypersensitivity reaction d. disseminated bacterial infection e. IgE dependent degranualtion

Acute cholecystitis is caused by gallstone obstruction of the cystic duct in more than 90% of cases. Ingestion of fatty foods then stimulates contraction of the gallbladder against the impacted stone, resulting in severe cocky pain. Mechanical disruption of the gallbladder mucosa and release of inflammatory mediators (eg, lysolecithin, prostaglandins) cause the obstructed gallbladder to become inflamed and edematous. As blood supply to the distended organ becomes compromised, secondary bacterial infection frequently develops. Potential complications include gangrene and perforation, with subsequent formation of a pericholecystic abscess or generalized peritonitis. US is the PREFERRED intimal imaging test of the diagnosis of acute cholecystitis; however, nuclear medicine hepatobiliary scanning (i.e., cholescintigraphy) can be an alternate means when US is INCONCLUSIVE. During a hepatobiliary scan, a radio tracer is administered IV and is preferentially taken up by hepatocyte and excreted into bile. IN patients with a patent cystic duct, the gallbladder will be seen as the radio tracer accumulates and concentrates within. In acute or chronic cholecystitis, the radio tracer will be taken up by the liver with progressive excretion into the common bile duct and proximal small bowel, but the gallbladder will not be visualized d/t the obstruction. a. distended duodenal upper GIT series would be suggestive of small-bowel obstruction (as seen with gallstone ileum). b. the presence of echogenic structures within the gallbladder on US can be suggestive of acute cholecystitis in the setting of fever and abdominal pain, but it is not DIAGNOSTIC. Cholelithiasis can also cause more benign biliary colic, or be an incidental asymptomatic finding in the setting of other abdominal pathology. US findings more specific for acute cholecystitis include gallbladder wall thickening, pericholecystic fluid, and a positive sonographic Murphy sign. d. mild increases in serum aspartate and alanine aminotransferase levels can occur in acute cholecystitis, but they are not specific and do not aid in the diagnosis. e. most patients have insufficient calcium in their gallstones to be visualized on an abdominal X-ray.

80 A 50 y.o woman has severe abdominal pain. It started suddenly and it is sharp and colicky. She also omitted several times throughout the day. She describe prior episodes of similar abdominal pain that resolved spontaneously without treatment. Her PMH include complicated appendicitis when she was 22. Temp is 100, HR is 112. Abdominal examination shows cessation of inspiration with deep palpation of the right upper quadrant. Lab assessment shows serum bilirubin of 0.8 mg/dL and serum alkaline phosphatase of 100. Which of the following diagnostic test results would be most specific for acute cholecystitis a. distended duodenal on upper GIT series b. echogenic structures inside the gallbladder on abdominal ultrasound c. failed gallbladder visualization on radionuclide biliary scan. d. increased serum aspartame and alanine aminotransferase levels. e. opacities in the right subcostal area on abdominal x-ray

Cd is caused by muttons affecting the CF transmembrane conductance regulator (CFTR), a chloride channel that is activated by cAMP-mediated phosphorylation and subsequently gated by ATP. Most common mutton is a 3 base pair delation that removes a phenylalanine at amino acid position 508. This causes impaired postranslational processing (eg, improper folding and glycosylation) of CFTR, which is detected by the ER. As a result, the abnormal protein is targeted for proteasomal degradation, preventing it form reaching the cell surface. (BCDE) less common mutations can also cause CF. Mutations causing premature termination of transmembrane protein (eg, nonsense, frameshift) also lead to complete absence of membrane bound CFTR. Mutations that cause defective ATP binding by CFTR can result in a membrane-bound, nonfunctional protein. Milder disease may cause by mutation impairing Cl conduction through CFTR or those cousin decreased production of functionally normal CFTR (eg, reduce dmRNA or protein stability).

802 Patient has CD. Which of the following abnormalities is most likely to be seen in this patient? a. abnormal post translational processing of a transmembrane protein b. an abnormally truncated transmembrane protein c. decreased transcription of a transmembrane protein d. impaired ion conduction through a transmembrane protein e. poor ATP binding by transmembrane protein

Patient has CF because: -recurrent pneumonia -digital clubbing -azoospermia -bilateral absence of vas deference CF patients have serious lung and pancreatic complications depending on the severity of the mutation. Regardless of lung and pancreas function, virtually all adult men with CF have azoospermia and infertility. Although spermatogenesis is usually normal, almost all males with CF are unable to secrete semen d/t congenital bilateral absence of vas deferens. CFTR mutations are likely responsible for abnormal development of Wolffian structures, resulting in vagal genesis and defective sperm transport. A diagnosis of CF can be based on elevated sweat chloride levels. If the sweat chloride test is equivocal, measurement of nasal transepithelial potential difference and genetic testing for CFTR mutations should be performed to confirm the diagnosis. b. although primary ciliary dyskinesia (eg, Kartagener syndrome) can cause recurrent pulmonary infections and digital clubbing, infertility in Kartagener syndrome is usually d/t immotile spermatozoa. Abnormal nasociliary motility is a nonspecific finding that is seen most commonly in patients with primary ciliary dyskinesesia and in some CF patients (from abnormally thick mucus). Low serum alpha-1 antitrypsin is associated with AAT def and not with CF. AAT def is typically associated with panacinar emphysema and chronic liver disease. Infertility is not seen in these patients d/f: FSH, LH and testosterone levels are suavely normal in patients with CF. A low testosterone level in setting of decreased FSH and LH is seen in hypogonadotropic hypogonadism (eg, Kallmann syndrome) e. primary humoral deficiencies usually manifest as recurrent upper and lower respiratory tract infections d/t impaired antibody production. However, most patients with selective IgA def are asymptomatic.

807 A 25 y.o white man is being evaluated for azoospermia. PMH is significant for recurrent pneumonia with frequent hospitalization for antibiotic treatment. He was adopted. PE shows digital clubbing. A transrectal ultrasound shows bilateral absence of the vas deferent. Which of the following tests would most likely confirm the underlying diagnosis of this patient's condition? a. Cl level in sweat b. cilia motility of the nasal epithelium c. serum alpha-1 antritrypsin level d. serum FASH and LH e. serum IgA f. serum testosterone levels.

Patient has scarlet fever, caused by Group A step that produced pyrogenic exotoxins. Scarlet fever is most often associated with step pharyngitis, which begins acutely after an incubation period of 1-5 days. Clinical symptoms are: -fever, malaise, abdominal pain and sore throat -pharynx is typically erythematous, swollen and possibly covered with gray white exudate -tongue have inflamed red papillae with an appearance similar to that of a red strawberry. After 1-2 days, a rash appears on the neck, armpit an groin that subsequently generalized the rest of the body. Rash begins as scarlet spots or blotches, giving a boiled lobster appearance. As the rash progresses and becomes more widespread, it begins to resemble a sunburn with goose pimples ("sandpaper-like" rash). The cheeks commonly appear flushed, giving the area around the mouth a pale appearance in comparison (circumoral pallor). Toward the end of the first week, desquamation begins and is most pronounced in the armpits, groin and tips of the fingers ad toes. As with any strep upper resp. infection, scarlet fever can predispose to acute rheumatic fever and glomerulonephritis. Treatment with penicillin V can prevent development of rheumatic fever, although its role in preventing glomerulonephritis is uncertain. Aplastic anemia can complicate parvovirus B19 infection in patients with sickle cell anemia or immunocompromised. Coronary artery aneurysm its the most dreaded complication of Kawasaki disease, which can present with fever, strawberry tongue, and a rash. However, tonsillar exudates are typical of bacterial disease, and this patient lack of other manifestation of Kawasaki disease (eg, bilateral conjunctival injection) Encephalitis can be cause by measles (acutely), acute disseminated encephalomyelitis (during recovery), and subacute sclerosing panencephalitis (years later). Orchitis is one of the most common complicates of mumps. Mumps characteristically presents with fever malaise, headaches, and myalgia followed by the development of parotitis within 48 hours.

8565 12 y.o has fever, chills and rash that appeared this morning. Two days ago he had a sore throat. Temp is 101. PE shows diffuse erythematous rash on his chest and abdomen that blanches with pressure along with numerous 1 to 2mm papule. The threat is erythematous with gray white tonsillar exudates and the tongue is bright red. Which of the following is the most serious complication that can develop from this disease? a. aplastic anemia b. coronary artery aneurysm c. encephalitis d. orchitis e. rheumatic fever

HPV (1-4): skin warts (verruca vulgarism) HPV (6, 11): genital warts (condylomata acuminatum) HPV (16,18): cervical, vaginal, vulvar and anal neoplasia SCC most common type of cervical cancer, arises from the squamocolumnar junction of the endocervix. CIN is carcinoma in situ and precedes SCC. CIN represents a wide spectrum of dysplastic changes of the cervical epithelium that occur under the influence of HPV infection. CIN is usually transient in young healthy women. However, patients with persistent HPV infection or untreated high grade CIN are at risk of invasive cervical cancer. DNA from high risk strains (i.e., 16, 18 and 31_ is integrated into the human cell genome, leading to over expression of viral oncogene E6 and E7. -E6 binds to protein p53 and increases its degradation -E7 binds to RB1 and displaces transcript factors normally bound by pRB, the tumor suppressor protein product of RB Risk factors are related to risk of acquiring HPV infection: -skin to skin contact -sexual contact -multiple sex partners -unprotective sex (lack of barrier contraceptive) BFG:null parity, early menarche and obesity increase the risk endometrial cancer. These factors are associated with increased estrogen stimulation of the endometrium. C: FH of breast cancer is a risk factor for developing breast malignancy. Breast cancer is not associated with cervical cancer, as the latter is usually d/t a persistent, oncogenic HPV infection. D: plants wart is usually caused by HPV 1-4; these strains are not typically carcinogenic and are not associated with cervical dysplasia and invasive cervical cancer. H. douching is a risk factor for arterial vaginosis by disturbing the vaginal ecosystem, resulting in overgrowth of Gardnereall vaginalis.

869 A nulliparous, mild obese woman is diagnosed with cevical intraepithelial neoplasia III. Menarche occurred at age 9, and her menses recur every 28 days and last 6 days. She had a plantar wart a few years ago but otherwise normal. The patient's mother had breast cancer at age 42. Which of the following is the most significant factor that predisposed this patient to cervical abnormalities? a. alcohol use b. early menarche c. family history of breast cancer d. history of plantar wart e. lack of barrier contraception f. nulliparity g. obesity h. practice of douching

This patient has xeroderma pigmentosum, which literally means pigmented dry skin. -AR -decreased ability to repaired DNA following damaged by UV light. Clinical manifestation: -erythema, scaling and subsequent hyper pigmentation and lentigo formation on light-exposed areas. -Later, the skin of affected areas shows atrophy, telangiectasia, and intermingling areas of hypo and hyperpigmentations. -skin malignancies, including SCC, basal cell carcinoma and malignant melanoma, develop as early as 5-6 years of life. Normally the region of DNA damaged by UV radiation are excised and replaced. In xero pigmentosum, the genes that code for various DNA repair enzymes are abnormal. IMparimtn of DNa repair results d/t defects in excision of abnormal nucleotides or defects in replacement of nucleotides following excision other diseases associated with impaired DNA repair include: -Fanconi (AR, hypersensitivity to DNA cross linking agents) -Bloom syndrome (AR, hypersensitivity to UV damage and chemotherapeutic agents) a. The Rb gene is responsible for regulation of the cell cycle. The abnormal Rb protein loses its ability to arrest the cell cycle in the G1 phase. Mutation is associated with retinoblastoma and osteosarcoma b. mutation of fas protein is found in many types of cancer. Fas codes for a G-protein that regulates signal transduction. Abnormal was stimulates, signal transduction leading to unregulated cell division, inhibited apoptosis and decreased cell adhesion. These predispose to malignancy and metastasis d. abnormalities in mismatch repair are found in patients with HNPCC aka Lynch syndrome.

883 A 5 y.o girl's skin become s red and scaling with only minimal sun exposure. She began to notice these phenomenon when the chidd was 7 month old. Now the girl's skin is thin and hyperpgimented. The patient has a few nevi on her hands that have been rapidly enlarging. The defective gene in this patient is responsible for what? a. regulation of cell cycle b. signal transduction c. DNA excision repair d. DNA mismatch repair e. prevention of micro deletions f. regulation of apoptosis

this patient is experiencing vertigo d/t vestibular dysfunction. Vertigo is a sensation of motion when no motion is present or an exaggerated sense of motion for a given bodily movement. It must be distinguished from other sensations that are often described by patients using similar terminology, such as imbalance, light headedness and syncope. Vertigo resulting from vestibular dysfunction tends to be sudden onset, interfere with waking and cause nausea and vomiting. Typical causes of vestibulopathy include Meziere's disease, perilymphatic fistulas, benign positional verge, labyrinthitis, and acoustic neuromas. a. disease of the posterior column of the spinal cord can occur in syphilis (i.e., tabes dorsalis) and vitamin B12 def. Typical symptoms include ataxia, decreased proprioception and vibratory sense and hyporeflexia. this patent's normal neurologic exam makes this unlikely b. vagal nerve damage can present as hoarseness, dysphagia, abnormal GI motility and tachycardia. However, dysfunction would not cause vertigo. c. Optic tract dysfunction will result in visual problems. Vision deficits do not typically cause vertigo, but may cause people to feel unsteady on their feet. e. cerebellar dysfunction presents as ataxia, imbalance, incoordination, and nystagmus. Patients with cerebellar dysfunction will be unable to perform tandem walk d/t their imbalance. f. the frontal cortex controls executive functions and response inhibitor. Damage to the frontal cortex results in frontal release symptoms, such as personality change, inability to organize or plan and disinhibition.

8859 A 32 year old woman has five episodes of intractable vomiting over the last year associated with spinning and tilting of room with balance difficulty. Negative Romberg and tandem walk. a. posterior columns of the spinal cord b. vagal nerve c. optic tract d. inner ear e. cerebellum f. frontal cortex

Dude has renal cell carcinoma. Large number is sporadic. Small number is hereditary, such as the von Hippel-Lindau (VHL) disease. VHL disease is an AD disorder caused by a deletion or mutation of the VHL tumor suppressor gene on chromosome 3p. It is characterized by cerebellar hemangioblastomas, clear cell renal carcinomas and pheocrhomocytomas . Renal cell carcinoma develops in nearly 40% of these patients and is the major cause of death. Although VHL disease is rare, alterations of the VHL gene and the crhomome 3p are found in the majority of patients with sporadic renal cell carcinoma. c-MYC is oncogene located on chromosome 8. Mutations are associated with Bruit lymphoma and diffuse large B cell lymphoma. NF-1 is a tumor suppressor gene located on chromosome 17 Mutations of this gene cause NF type 1 RB is an anti-oncogene (tumor suppressor) located on 13. Mutations of this gene lead to the development of retinoblastoma and osteosarcoma. Mutations of WT-1, a tumor suppressor gene, are associated with Wilms tumor. This gene is located on chromosome 11.

904 A 60 y.o man has dark, rusty-colored urine for the last 2 weeks. Dude smokes half a pack of cigs for 10 years and quit 30 years ago. Father had HTN and mom has Alzheimer. Urinalysis shows large number of red blood cells. Renal ultrasound reveals a mass in the right kidney. Cytologic evaluation of the mass shows malignant cells with chromosome 3p deletion. The deletion most likely involves which of the following genes? a. c-MYC b NF-1 c. RB d. VHL e. WT-1 What other tumor would you have an increase risk in?

This patient's symptoms and biopsy finding are suggestive of acute serum sickness, a condition caused by tissue deposition of circulating immune complexes (type III hypersensitivity). Most common manifestations include fever, pruritic skin rash and arthralgia that begin 7-14 days after exposure to an antigen. Lymphadenopathy and proteinuria may also occur in some patients. Histology: small vessel vasculitis with fibrinoid necrosis and intense neutrophil infiltration. Deposition of IgG and/or IgM complement fixing antibodies result in localized complement consumption and hypocomplementemia (decrease serum C3 levels). Serum sickness can occur following administration of antigenic heterologous proteins such as chimeric monoclonal antibodies (eg, rituimab and infliximab) or nonhuman IGs (eg, venom antitoxins). A serum sickness like reaction is also associated with the use of certain nonprotein drugs (eg, penicillin, cofactor, and TMP-SMX). a. anergy to cutaneously applied Candida antigens would be indicative of depressed cell mediated immunity. Cell mediated immunity is involved in the pathogenesis of type IV (delayed) hypersensitivity. c. serum IgE levels are typically found in atopic individuals prone to IgE mediated (type I) hypersensitivity. Type I reactions cause vasodilation and tissue edema and inflammatory infiltration; NOT vasculitis with fibrinoid necrosis Deposition of IgA containing immune complexes is involved in pathogenesis of Henoch-Schonlein purpura in pediatric patients. IgA doesn't play an important role type III hypersensitivity Serum sickness causes release of C5a (a neutrophil chemoattractant) complement fragment at sites of immune complex deposition. This lead to neutropenia d/t extensive neutrophilic marginalization and tissue infiltration. In addition, infliximab and other TNF-alpha inhibitors can also cause neutropenia. Mild thrombocytopenia can be associated with serum sickness d/t platelet consumption at the site of active vascular inflammation. However,rsevere thrombocytopenia is unlikely.

A 32 y.o man is started on infliximab for treatment of refractor Crohn disease. Ten days later he develops joint pain and pruritic skin rash. Skin biopsy shows scattered areas of fibrinoid necrosis and neutrophil infiltration involving his small blood vessels. Which of the following findings is most likely to accompany this patient's condition? a. candida antigen anergy b. decreased serum C3 level c. increased serum IgE level d. low serum IgA level e. neutrophilic f. severe thrombocytopenia


Conjuntos de estudio relacionados

Taylor Chapter-31 skin integrity and wound care

View Set

Modern Database Management - Chapter 3

View Set

Los meses y las estaciones del año (Tarea due 11/02)

View Set

Physiology Unit 4: Endocrine System

View Set

computer science chapter 6&7 test

View Set